Solution Manual for Project Management Achieving Competitive Advantage, 5th Edition By Jeffrey K. Pi

Page 1


Instructor’s Solutions Manual For Project Management: Achieving Competitive Advantage Fifth Edition Jeffrey K. Pinto, Pennsylvania State University Prepared by Jeffrey K. Pinto, Pennsylvania State University

Boston

Columbus

Indianapolis

New York

San Francisco

Hoboken

Amsterdam Cape Town Dubai London Madrid Milan Munich Paris Montreal Toronto Delhi Mexico City Sao Paolo Sydney Hong Kong Seoul Singapore

Taipei

Tokyo


______________________________________________________________________________

Copyright © 2019, 2016, 2013 by Pearson Education, Inc. or its affiliates. All Rights Reserved. Printed in the United States of America. This publication is protected by copyright, and permission should be obtained from the publisher prior to any prohibited reproduction, storage in a retrieval system, or transmission in any form or by any means, electronic, mechanical, photocopying, recording, or otherwise. For information regarding permissions, request forms and the appropriate contacts within the Pearson Education Global Rights & Permissions Department, please visit www.pearsoned.com/permissions/.

Instructors of classes using Project Management: Achieving Competitive Advantage, 5th Edition, Jeffrey K. Pinto, may reproduce material from the Instructor’s Resource Manual and Test Bank for classroom use.

10 9 8 7 6 5 4 3 2 1

ISBN-10: 0-13-473045-3 ISBN-13: 978-0-13-473045-5

www.pearsonhighered.com


CHAPTER FOURTEEN Project Closeout and Termination Chapter Outline PROJECT PROFILE Amazon’s Golden Touch Fails with a High-Tech Gadget INTRODUCTION 14.1 TYPES OF PROJECT TERMINATION PROJECT MANAGERS IN PRACTICE James J Devine, PMP - Engineering Research and Consulting, Inc. 14.2 NATURAL TERMINATION—THE CLOSEOUT PROCESS Finishing the Work Handing Over the Project Gaining Acceptance for the Project Harvesting the Benefits Reviewing How It All Went Putting It All to Bed Disbanding the Team What Prevents Effective Project Closeouts? 14.3 EARLY TERMINATION FOR PROJECTS Making the Early Termination Decision PROJECT PROFILE Case—Aftermath of a “Feeding Frenzy”: Dubai and Cancelled Construction Projects Shutting Down the Project Allowing for Claims and Disputes PROJECT MANAGEMENT RESEARCH IN BRIEF Project Termination in the IT Industry 14.4 PREPARING THE FINAL PROJECT REPORT Conclusion Summary Key Terms Discussion Questions Case Study 14.1 New Jersey Kills Hudson River Tunnel Project Case Study 14.2 The Project That Wouldn’t Die Case Study 14.3 The Navy Struggles to Avoid Cancellation of its Littoral Combat Ship Program Internet Exercises PMP Certification Sample Questions Appendix 14.1—Sample Pages from Project Sign-off Document Notes

341 Copyright 2019 Pearson Education, Inc.


TRANSPARENCIES Note for Instructors: To present transparencies in class, please download the PowerPoint Presentations (available on www.pearsonhighered.com) that accompanies this product. The PPT ISBN is 9780134730479.

14.1 TYPES OF PROJECT TERMINATION 1. TERMINATION BY EXTINCTION 2. TERMINATION BY ADDITION 3. TERMINATION BY INTEGRATION 4. TERMINATION BY STARVATION

14.2 THE SEVEN ELEMENTS OF PROJECT CLOSEOUT MANAGEMENT 1. Finishing the Work 2. Handing Over the Product 3. Gaining Acceptance for the Product 4. Harvesting the Benefits 5. Reviewing How in All Went 6. Putting it All to Bed 7. Disbanding the Team

14.3 EARLY WARNING SIGNS OF PROJECT FAILURE 1. LACK OF VIABLE COMMERCIAL OBJECTIVES 2. LACK OF SUFFICIENT DECISION-MAKING AUTHORITY 3. MARKET VOLATILITY 4. LOW PRIORITY ASSIGNED TO THE PROJECT

Source: Green, Welch, and Dehler, 1993.

14.4 WHEN SHOULD WE CONSIDER CANCELLING THE PROJECT?

1. WHEN COSTS EXCEED BUSINESS BENEFITS 2. WHEN THE PROJECT NO LONGER MEETS STRATEGIC FIT CRITERIA 3. WHEN DEADLINES CONTINUE TO BE MISSED 4. WHEN TECHNOLOGY EVOLVES BEYOND THE PROJECT’S SCOPE 342 Copyright 2019 Pearson Education, Inc.


14.5 WORK BREAKDOWN FOR PROJECT TERMINATION ISSUES Project Termination Issues

Emotional

Staff

Intellectual

Client

Internal

External

Fear of no future work

Change in attitude

Identification of remaining deliverables

Agreement with client on remaining deliverables

Loss of interest in remaining tasks

Loss of interest in project

Certification needs

Agreement with suppliers on outstanding commitments

Loss of projectderived motivation

Change in personnel dealing with project

Loss of team identity

Unavailability of key personnel

Selection of personnel to be reassigned

Diversion of effort

Identification of outstanding commitments

Communicating closure

Control of charges to project

Closing down facilities

Screening of partially completed tasks

Determination of requirements for audit trail data

Closure of work orders and work packages Disposal of unused material

Source: Spirer (1983)

14.6 1. 2. 3. 4. 5. 6.

ELEMENTS TO BE INCLUDED IN THE FINAL PROJECT REPORT PROJECT PERFORMANCE ADMINISTRATIVE PERFORMANCE ORGANIZATIONAL STRUCTURE TEAM PERFORMANCE TECHNIQUES OF PROJECT MANAGEMENT BENEFITS TO THE ORGANIZATION AND THE CUSTOMER

343 Copyright 2019 Pearson Education, Inc.


DISCUSSION QUESTIONS 14.1 Why is the decision to terminate a project often as much an emotional one as an intellectual one? When termination occurs without completing objectives, there are several emotional elements involved for those connected to the project. The most obvious is that the termination represents a failure for those involved. For the visionary or originator of the project concept, their personal vision has to be relinquished (at least for the time being). Additionally, the ego, pride, and reputation of project team members may be injured. Lastly, there may be anxiety over job loss or transfers when the team is dismantled.

14.2 Comment on the different methods for project termination. How have you seen an example of one of these methods, through either your school or work experience? This is a question that requires students to respond to individually. Some with extensive project management experience will easily recognize how to answer this, while other students will need to reflect on a variety of their activities that could be considered examples of “projects” before they can respond.

14.3 Why do so many projects end up terminated as a result of termination through starvation? Discuss the role that ego, power, and politics in this form of termination. Termination through starvation, or deliberate lack of resources, may occur for several reasons causing projects to be terminated before completion. One reason for starvation results from changes in upper management priorities or organizational goals. When priorities change, what once may have been a top project may become insignificant. Therefore, funding is allocated elsewhere leaving the other project to starve. There may be a decrease in funding (for a multitude of reasons) causing insufficient resources for existing projects. Another cause of starvation relates to power and influence. Those who hold powerful positions in organizations, but may not necessary act functionally within the organization, often have “pet projects.” In the event that those who allocate resources do not see these projects as highly desirable, the projects may be allowed to exist on paper, but in reality they are not giving the resources they need to survive. Over time, they starve and eventually are terminated. In these cases, power struggles may ensue between those who sponsor the projects and those who have to create budgets and project timelines. Politics may also influence which projects are starved. Outside of personal politics involving the pet projects, “community friendly” projects may be allowed to exist with minimal funding in order to create harmony between the company and the surrounding community. Again, these projects tend to exist on paper but are allocated few to no resources. 344 Copyright 2019 Pearson Education, Inc.


14.4 Refer back to Chapter 2. How does the concept of escalation of commitment factor into decisions of whether or not to terminate projects? Remember that escalation of commitment is an emotional, psychological perspective that supports the continuation of a bad decision once it has been made. That is, people tend to continue to pursue poor choices in spite of contradictory evidence that suggests a better course would be to stop. This same concept holds true to termination decisions because egos are often involved, key organizational members may be supporting the decision and they are loath to be proved wrong, and so forth. In short, escalation of commitment demonstrates why many projects are pursued against all available evidence of their pending failure. 14.5 Consider the case of the Navy’s Littoral Combat Ship in Case 14.3. Take the position that terminating this project after having invested so much in research and development represented a good or bad decision by the Navy. Argue your case. This question requires students to take either a pro or con position on the Navy’s decision to terminate the Littoral Combat Ship program. Instructors should encourage students to research the topic via the Internet to develop a comprehensive understanding not just of the Littoral Combat Ship, but of the problems inherent in large defense programs. The costs of new ship classes are so high that it requires a strong case be made for the need for such a platform. In this case, there are so many counter-arguments to each of the original needs that were cited that it makes for an interesting in-class discussion. Clearly, the decision to pursue additional ships of this class is open to question, given their cost and the quality problems that have plagued them, the obvious flaws in the strategic thinking that drove their development, and current economic conditions. As a result, a reasonable case can be made that the ship class does not support an obvious and immediate critical need in the Navy’s inventory, leading to its cancellation. Alternatively, students can argue that quality issues are common in new programs and once they get worked out, the underlying value will become evident. One final point to ask students to consider is the political question that emerges when powerful senators from the states where the shipbuilding occurs (e.g., Maine and Mississippi) work to get additional funding for the program—not for additional ships, per se, but to keep people at the shipbuilding yards employed.

14.6 Of the seven elements in project closeout management, which do you view as being most important? Why? This question is designed to have students consider the challenges of each element of a project closeout, recognizing that they all have associated risks and require careful consideration. In other words, there are no “simple” elements in the closeout process. For example, handing over the project to its users can be a difficult process for many IT projects, as users may have preconceived ideas about what they are expecting versus 345 Copyright 2019 Pearson Education, Inc.


what they will actually receive. Likewise, reviewing how it went may be difficult because project team members have moved on to their new assignments and there may be no energy or motivation to take an in-depth look at the project development process, especially if it did not go according to plans.

14.7

What are four principles of effective postproject reviews?

The four principles are: objectivity, internal consistency, replicability, and fairness. Objectivity refers to the need for an unbiased, critical review of the project from the perspective of someone without an agenda or “axe to grind.” Internal Consistency recognizes that there must be a logical and well-constructed procedure to be followed when conducting reviews. Replicability refers to the need for a standardized review process that should yield similar findings regardless of who conducts the evaluation. Finally, fairness is a principle whereby members of the project team must perceive that the review was conducted fairly, without agendas, and intended to highlight both successes and failures.

14.8 What are some of the reasons why objective project evaluation may be difficult to achieve? The text suggests four reasons why objectivity is difficult to obtain, including: a. Outside reviewers are unfamiliar with the circumstances the project team was dealing with while implementing the project. The evaluator may not understand the project’s technical challenges, specific instructions received from top management or clients, or problems encountered while working on the project. The “outside view” here does not allow for a knowledgeable perspective on the part of the reviewer and may unfairly influence their response. b. Project teams may be suspicious about who selected the reviewers and the instructions they may have received. The worst thing that can happen is for the project team members to assume that the reviewers have an agenda they are working from when reviewing a project. If the reviewer holds a particular ideological or political perspective, it will likely color their evaluation. For example, the Affordable Healthcare Act, commonly referred to as Obamacare, has been a political lightning rod for several years now. Were a Republican Congressman to serve as a program reviewer, the argument could reasonably be raised that his perspective is biased. c. Outside evaluators may feel compelled to find problems. Human nature suggests that when individuals are assigned to serve as project reviewers, they will naturally dig for problems, with the potential for elevating minor issues into much larger problems than they really are. As one project management writer put it: Evaluators should be on the lookout for problems but they should “guard against behaving like traffic police who have a quota of tickets to issue each day.”i 346 Copyright 2019 Pearson Education, Inc.


d. Outside reviewers are not competent. It may be the case that the evaluator assigned to review the project is not technically or otherwise competent to make an accurate assessment. 14.9 Why do “lessons learned” programs often fail to capture meaningful information that could help guide future projects? Lessons learned programs are aimed at documenting problems from past projects to be used as problem predictors for future projects. However, those in charge of closing out the project often fail to properly document. This sometimes occurs during closeout. For instance, members performing the closeout assume that a problem was a one-time, nonrecurring event. Therefore, there is no need to document it for future reference. Other times, the source of the error is misinterpreted making it difficult to learn from the experience. Or, there may be the belief that the project is too unique to be beneficial for future use. Another problem that makes lessons learned inefficient is that future PMs do not look at the documents prior to beginning new projects. This may be due to a lack of procedure, time, or care for performing research of past projects. 14.10 Comment on the following statement: “In deciding on whether or not to kill a project, it is critical to continually monitor the environment for signs it may no longer be viable.” There are internal and external reasons that may render a project no longer viable. When considering external reasons, it is vital to scan the environment for potential obsolescence or other changes in the operating environment that signal a necessary shut down. Signs such as reduced market opportunity, changes in consumer demands, and rapid technological advancements may allow call for project termination. Failing to heed these warnings could result in the failure of the project upon completion or continued investment into an unattainable project goal.

14.11 Refer to the Project Management Research in Brief box in this chapter. In your opinion, why is it so difficult to bring IT projects to successful completion? In other words, identify some reasons why the cancellation rate for IT projects is 40%. Information technology is subject to a great deal of change associated with technological advancements. Many projects run the risk of being obsolete if they do not operate under extremely short-time frames. In addition to potential obsolescence, there are additional risks that run high in IT project management. For the sake of time, projects are rushed causing oversights and inadequate planning which would use up extra time, a luxury these projects do not seem to have. The lack of planning leads to poor definition of customers needs and project scope. Furthermore, reviewing past projects for “lessons learned” is often skipped, leading to redundant problems. 347 Copyright 2019 Pearson Education, Inc.


14.12 Imagine you are a project team member on a project that has missed deadlines, has not produced the hoped-for technological results, and has been a source of problems between your team and the customer. You have just been informed that the project is being canceled. In what ways is this good news? How would you view it as bad news? Missing deadlines and milestones are warning signs the project needs to be terminated. As a member of the project team, the project’s cancelation would therefore be good news, since it would be a waste of time and talent to continue pouring resources into a project that likely will not meet its objectives. However, personally the project failure may be detrimental. The dismantling of the project may mean you will be transferred (possibly to a position you do not desire) or you may no longer be needed. Moreover, depending on the reasons for the failure, the termination may reflect badly on your own reputation and impede the possibility of promotion or reassignment.

CASE STUDIES Case Study 14.1—New Jersey Kills Hudson River Tunnel Project This case illustrates the challenges in making an early termination decision. Often, particularly in the case of public projects, there is a real difficulty in stopping a project once it has gotten “on the books.” As a result, projects with huge cost overruns, like Boston’s “Big Dig” are allowed to continue almost indefinitely. New Jersey Governor Chris Christie made a tough call in deciding to cancel the Hudson River Tunnel project because he was given a number of rosy projections that did not match the actual costs incurred to that date. This is a great case to let students pick sides: should Christie have cancelled the project or not? There are arguments to be made that the need was strong (although others could argue that the need was over-sold). On the other hand, the history of the project to date and the uncertainty about future federal funding made it a real gamble, especially during a time of economic recession. Questions 1. How would you respond to the argument that it is impossible to judge how successful a project like this one would have been unless you actually do it? This question has also been phrased more elegantly as Hirschman’s “Hiding Hand” theory that projects may show benefit down the road, even if their current costs and advantages are not lining up with economic realities. The problem is to try and reasonably determine when a project’s value should be assessed. The Sydney Opera House was a financial disaster but is now, decades later, an Australian landmark. The Channel Tunnel (Fixed Link) project has never made money and lost its investors a fortune. When should that be assessed? It is an interesting question without an obvious right or wrong answer. 2. Take a position, either pro or con, on Christie’s decision to kill the ARC. Develop 348 Copyright 2019 Pearson Education, Inc.


arguments to support your point of view. As discussed, answers to this question are not clear cut. There are valid arguments on both sides of the coin, which is exactly why Christie’s decision has been so controversial. Students can be put into “pro” and “con” groups for this debate and should be encouraged to bring in outside information (of which there is plenty on the internet). One suggestion: ask students to compare and contrast the decision to terminate this project with the California High-Speed Rail project that is still staggering along, seeking additional funding and public support. 3. In your opinion, how clearly must a large infrastructure project like ARC have determined its need, costs, and so forth before being approved? If the criteria are too stringent, what is the implication for future projects of this type? Would any ever be built? This question gets to the heart of mega-project investment. Obviously, the financial pros and cons for almost any mega-project must be investigated as stringently as possible but there are limits as to how well we are ever likely to project future benefits. On the other hand, as the “Building Megadams” Case Study 8.1 suggested, many assumptions about future benefits are actually wildly inaccurate and lead to long-term financial sinkholes for countries making these investments. Case Study 14.2—The Project That Wouldn’t Die This case tells the story of a powerful individual’s pet project that has been allowed to continue, despite a complete lack of productive results, for the past 20 months. Alice is explaining to Ben, recently appointed to the project, some of the reasons why the Regency Project has been kept alive and how Ben can make the best of the situation he faces. Questions 1. What termination method does it appear to company is using with the Regency Project? It appears that they are using Termination by Starvation. No one wants to be in the position to kill the pet project of a Vice-President, but on the other hand, the case notes that top management has noticed the lack of productive results coming from the project and appears to have decided that the politically smart thing to do is kill it, but slowly so as not to offend its original project manager and current sponsor.

2. What are the problems with motivation when project team members perceive that a project is earmarked for termination? It is extremely difficult to get personnel motivated to serve on a project that is either rumored to be ripe for termination or in fact, is being slowly killed, as in the case of the 349 Copyright 2019 Pearson Education, Inc.


Regency Project. Personnel do not perceive service on such a project as a good career step and often, will work hard to get themselves removed from these projects so they are not associated with the projects termination and perceived failure. Motivation in this instance is very hard to maintain because of natural feelings of self-preservation. Alice’s advice to Ben is good, when she suggests that because the project is one of high visibility, his efforts to do his best may get him noticed, despite the likely Termination by Starvation that the project faces. 3. Why would you suspect Harry Shapiro has a role in keeping the project alive? The simplest reason is often the most powerful—human ego. Harry was the originator of the project and served as its first project manager before getting promoted; thus, the project will always have great visibility to him and a strong emotional attachment. Top management must be aware of this fact because rather than killing the project directly, they are opting for a less-direct approach that slowly whittles away at its funding to the point where, they hope, it simply dies of this neglect.

Case Study 14.3—The Navy Struggles to Avoid Cancellation of its Littoral Combat Ship Program This is the true story of a recent project sponsored by the U.S. Navy that is generating a great deal of controversy. The Navy determined that the next generation of warfare was likely to occur in the “littoral” (coastal) areas, around land masses, rather than deeper into the ocean. Consequently, they created a competition between two firms to develop the Littoral Combat Ship and beyond giving broad guidance, they allowed the two firms to develop different versions of the craft. One key emphasis coming from the Navy was that the ships in the new class must be cost-effective; that is, they cannot allow major cost overruns to sour Congress’s perceptions of the new class. A total of 13 have been built, as of 2017, and the ships are currently experiencing a number of technical and mechanical problems. Part of the problem is that the Navy is intent on continuous upgrades to the ships, as new technologies become available, so rather than use a “spec freeze” approach that allows the builders to develop learning curves and improve the technical problems, they keep changing the requirements. Finally, in an effort to comply with some of the Navy’s requirements for speed, the ship builders are deliberately sacrificing critical materials and substituting aluminum and other alloys, which will be much more hazardous to the ships’ crews in the event of actual combat. Over-budget, too complicated, and uncertain mission capabilities make this a very questionable program to support.

350 Copyright 2019 Pearson Education, Inc.


Questions 1. The U.S. Department of Defense has a long history of sponsoring projects that have questionable usefulness. If you were assigned as a member of a project review team for a defense project, what criteria would you insist such a project have in order to be supported? In other words, what are the bare essentials needed to support such a project? This question does not have any obvious answers but requires students to suggest that one of the reasons for runaway projects is a failure to adequately freeze specifications and mission parameters early in the development process. This error results in “the engineering disease” of continual upgrades, tinkering, and modifications for the sake of “newness,” rather than to fulfill the project’s core mission. 2. Why, in your opinion, is there such a long history of defense projects overshooting their budgets or failing some critical performance metrics? (In your assessment, make sure to consider a wide variety of criteria, from Congressional support to changing political realities.) This offers a nice discussion point of departure by comparing the multiple examples of defense projects that have overrun, going back to the Strategic Defense Initiative and beyond. The essential problem is that we are often dealing with cutting-edge technologies that have not been fully tested nor integrated with other leading-edge systems. This lack of field testing and integration causes all manner of technical problems and unforeseeable issues that must be addressed one at a time. The cumulative effect is to lead to huge overruns of budget and schedule. It is often useful to assign students to research examples of defense project failures such as the Sergeant York antiaircraft gun, the A-12 attack airplane, or the Bradley Fighting Vehicle. 3. Google “criticisms of the Littoral Combat Ship” and identify some of the several problems that critics have listed. In light of these problems, why do you think the Navy has pressed ahead with the development of the LCS? Students will find a large amount of information on this new Littoral Combat Ship, particularly in Navy blogs and from former servicemen and women. The evidence suggests that the LCS is a bad conceptual idea that is being built because it has funding, much as past weapon systems were developed beyond the point they were either needed or fulfilled their original mission (e.g., Bradley Fighting Vehicle).

i

Frame, J.D. (2004), as cited, p. 1202.

351 Copyright 2019 Pearson Education, Inc.


CHAPTER THIRTEEN Project Evaluation and Control PROJECT PROFILE U.S. Army Can’t Track Spending on its Spending Tracker Project Introduction 13.1 CONTROL CYCLES—A GENERAL MODEL 13.2 MONITORING PROJECT PERFORMANCE The Project S-Curve: A Basic Tool S-Curve Drawbacks Milestone Analysis Problems with Milestones The Tracking Gantt Chart Benefits and Drawbacks of Tracking Gantt Charts 13.3 EARNED VALUE MANAGEMENT Terminology for Earned Value Creating Project Baselines Why Use Earned Value? Steps in Earned Value Management Assessing a Project’s Earned Value 13.4 USING EARNED VALUE TO MANAGE A PORTFOLIO OF PROJECTS PROJECT PROFILE Earned Value Management at Northrop Grumman 13.5 ISSUES IN THE EFFECTIVE USE OF EARNED VALUE ANALYSIS 13.6 HUMAN FACTORS IN PROJECT EVALUATION AND CONTROL Critical Success Factor Definitions Conclusions Summary Key Terms Solved Problems Discussion Questions Problems Case 13.1 The IT Department at Kimble College Case 13.2 The Superconducting Supercollider Case 13.3 “Dear Mr. President—Please Cancel our Project!”: The Honolulu Elevated Rail Project Internet Exercises MS Project Exercises PMP Certification Sample Questions Answers Appendix 13.1 Earned Schedule Notes

301 Copyright © 2019 Pearson Education, Inc.


TRANSPARENCIES Note for Instructors: To present transparencies in class, please download the PowerPoint Presentations (available on www.pearsonhighered.com) that accompanies this product. The PPT ISBN is 9780134730479.

13.1 A GENERAL MODEL OF THE PROJECT CONTROL CYCLE

The Project Control Cycle

302 Copyright © 2019 Pearson Education, Inc.


13.2 BUDGETED COSTS FOR SAMPLE PROJECT

Duration (in weeks)

Design

5

10

6

2

Engineer

4

15

20

25

8

8

8

4

20

Install Test

30

35

40

45

2

6

4

2

Total

6

Total

6

6

8

12

28

8

6

4

2

Cumul.

6

12

20

32

60

68

74

78

80

80

303 Copyright © 2019 Pearson Education, Inc.


13.3 PROJECT S-CURVE

Cumulative Cost ($ in thousands) 80

60

40

20

5

10

15 20 25 30 Elapsed Time (in weeks)

35

40

45

304 Copyright © 2019 Pearson Education, Inc.


13.4

PROJECT S-CURVE SHOWING NEGATIVE VARIANCE

Cumulative Cost ($ in thousands) 80

60

$10,000 Negative Var. 40

20

5

10

15 20 25 30 Elapsed Time (in weeks)

35

40

45

Cumulative Budgeted Cost Cumulative Actual Cost

305 Copyright © 2019 Pearson Education, Inc.


13.5

GANTT CHART WITH MILESTONES

Microsoft product screen shot(s) reprinted with permission from Microsoft Corporation.

306 Copyright © 2019 Pearson Education, Inc.


13.6

TRACKING GANTT WITH PROJECT ACTIVITY DEVIATION

Microsoft product screen shot(s) reprinted with permission from Microsoft Corporation.

307 Copyright © 2019 Pearson Education, Inc.


13.7 EARNED VALUE TABLE WITH PERCENTAGE OF TASKS COMPLETE

Design

5

10

6

2

Engineer

4

15

Duration (in weeks) 20 25 30

35

40

45

% Comp. 100

8

Install

8

8

4

20

Test

100 6

50

2

6

4

2

Total

6

6

8

12

28

8

6

4

2

Cumul.

6

12

20

32

60

68

74

78

80

0

308 Copyright © 2019 Pearson Education, Inc.


13.8

CALCULATING EARNED VALUE

(All numbers are in thousands $)

Planned

% Comp.

Earned Value

Design

8

100

8

Engineer

28

100

28

Install

30

50

15

Test

14

0

0

Cumul. Earned Value

51

309 Copyright © 2019 Pearson Education, Inc.


13.9 PROJECT BASELINE, USING EARNED VALUE

Cumulative Cost ($ in thousands) 80 Project Baseline 60 Earned Value 40

20

5

10

15 20 25 30 Elapsed Time (in weeks)

35

40

45

310 Copyright © 2019 Pearson Education, Inc.


13.10

EARNED VALUE METRICS

311 Copyright © 2019 Pearson Education, Inc.


13.11 EARNED VALUE TABLE

Activity Value

Jan

Feb

Staffing 15

8

7

Blueprinting

4

Mar

Apr

May

Jun

6

Jul

10

Plan

%C

15

100

80

8

10

60

Prototype Devel 6

2

8

Full Design

3

8

10

21

33

7

2

30

32

25

8

10

10

0

20

0

0

Construction Transfer 0 Punch List

15

5

Σ=

118

Monthly Plan 8

7

6

17

10

55

15

Cumulative

8

15

21

38

48

103

118

Monthly Actual

8

11

8

11

10

30

Cumulative Actual

19

27

38

48

78

8

44

0

312 Copyright © 2019 Pearson Education, Inc.


13.12 SCHEDULE VARIANCES FOR EVM

Schedule Variances Planned Value (PV)

103

Earned Value (EV)

44

Schedule Performance Index

EV/PV = 44/103 = .43

Estimated Time to Completion

(1/.43 × 7) = 16.3 months

313 Copyright © 2019 Pearson Education, Inc.


13.13 COST VARIANCES FOR EVM

Cost Variances Actual Cost of Work (AC)

78

Earned Value (EV)

44

Cost Performance Index

EV/AC = 44/78 = .56

Estimated Cost to Completion

(1/.56 × $118,000) = $210,714

314 Copyright © 2019 Pearson Education, Inc.


13.14 CRITICAL SUCCESS FACTORS IN PROJECT IMPLEMENTATION 1. Project Mission

2. Top Management Support

3. Plans and Schedules

4. Client Consultation

5. Personnel

6. Technical Tasks

7. Client Acceptance

8. Monitoring and Feedback

9. Communication

10. Troubleshooting

DISCUSSION QUESTIONS 13.1 Why is the generic four-stage control cycle useful for understanding how to monitor and control projects? One of the more difficult challenges of project control is finding a way to accurately measure progress. The four-stage cycle breaks project down into specific goals that can be measured against the project baseline. Deviations from the planned budget or time line can be identified and corrected swiftly. The fact that it is a cycle, implying repetition of the process, demonstrates the constant need for project monitoring and control measures. The final step in the cycle is to recycle the process resulting in continuous project control.

315 Copyright © 2019 Pearson Education, Inc.


13.2 Why was one of the earliest project tracking devices referred to as an S-curve? Do you see value in the desire to link budget and schedule to view project performance? This early device compared project time and cost graphically. The nature of project time and costs creates an S when the points are plotted on a graph, hence the term “S-curve.” There is value in linking the budget and schedule as an indicator of project performance. Following the S-curve, managers can get a rough depiction of expected progress. They can also see deviations of their own project from the typically expected progression.

13.3

What are some of the key drawbacks with S-curve analysis?

The cause of S-curve drawbacks lies mainly in its lack of tying the schedule and budget to actual project progress. S-curves give little indication as to the cause of variations from projections. The S-curve simply points out deviation of cost in relation to time. It does not relate task completion to time or cost. Therefore, when a deviation is discovered, it is unknown whether the project is on target so far as physical progress (whether work is being completed on, ahead, or behind the anticipated time and budget). Without knowing the cause of the variance, managers may make incorrect assumptions about the status of the project.

13.4 What are the benefits and drawbacks with the use of milestone analysis as a monitoring device? Milestone analysis is beneficial in signaling the completion of important project stages and in creating distinctions between work packages. This increases the team’s ability to respond to change and create logical review points. Milestones also provide periodic goals that keep team members motivated. They represent significant accomplishments within the larger picture of the project. It also draws the team’s attention to the project’s status. Overall, the analysis provides a clear picture of project development. However, this form of analysis only allows for reaction to problems, not foresight or prevention. Problems are then able to compound and grow to the point of unmanageable resulting in a significant over-budget/schedule project.

13.5 It has been said that Earned Value Management (EVM) came about because the Federal Government often used “Cost-plus” contractors with project organizations. Costplus contracting allows the contractor to recover full project development costs plus accumulate profit from these contracts. Why would requiring contractor firms to employ earned value management help the government hold the line against project cost overruns? Earned Value Management goes beyond reporting costs and progress. It links costs incurred to the time and budget baseline as well as to measurable performance 316 Copyright © 2019 Pearson Education, Inc.


milestones. By using EVM, the government is requiring that costs incurred during the project be directly tied to performance or progress of the project. The cost of the project is based on the budgeted cost of work performed. Therefore, negative or positive variances in performance are measured by the value of the work performed, not by the costs spent to complete the work. This allows companies to have a better understanding of what variances mean and their impact on the overall project. By understanding why the meaning of variances, managers are in a better position to take corrective action and to keep the project on schedule.

13.6 What are the major advantages of using EVM as a project control mechanism? What do you perceive are its disadvantages? The major advantages of EVM are that it is a comprehensive approach to measuring progress (links cost, time, and completion), its use of objective criteria, and it enables more accurate information for decision making. Disadvantages may include the time consuming nature of analysis in large-scale projects, mathematical formulas used for efficiency do not take into account unique problems that stall the project or spike costs in one area (which may not lead to overall poor efficiency), and a lack of information regarding what type of corrective action may need to be taken.

13.7 Consider the major findings of the research on human factors in project implementation. What common themes seem to emerge from the research on behavioral issues as a critical element in determining project status? The overarching theme is that in order to understand why a project is progressing the way it is the project must be evaluated on human performance criteria. There are several human factors proven to be influential in project success. Some of the main areas that need to be measured are motivation, leadership, expertise and top management support. The problem with this area of assessment is that it lacks a straightforward, objective system for measurement.

13.8 The 10 critical success factors have been applied in a variety of settings and project types. Consider a project with which you have been involved. Did any of these factors emerge clearly as being the most important for the project’s success? Why?? This question requires students to give a specific answer based on their own experience with projects. Responses will vary depending upon the project they select to respond to with the critical success factor model.

13.9 Identify the following terms: PV, EV, and AC. Why are these terms important? How do they relate to one another? 317 Copyright © 2019 Pearson Education, Inc.


PV refers to Planned Value. This is the expected (planned) budget for all project activities that are planned to occur within a specific time period. Planned Value is compared with Earned Value to determine the “real” progress that has been made on a project. EV refers to Earned Value. Earned value is the budgeted cost of the work performed. This is important in establishing the true progress of the project and in understanding the meaning of variances from the project baseline. AC stands for Actual Cost. These are the total costs incurred to complete project work.

13.10 What do the Schedule Performance Index and the Cost Performance Index demonstrate? How can a project manager use this information to estimate future project performance?? The indexes compare the planned value and actual cost of the project with the earned value (EV) measure to assess “true” project performance. The goal for an organization is to maintain SPI and BPI of 1.0 or higher, indicating that the project’s progress is ahead of schedule. The lower the project’s SPI and CPI are from 1.0, the less progress is being made on the project and the higher the likely overruns on schedule and budget we can anticipate.

13.11 Suppose the SPI is calculated as less than 1.0. Is this good news for the project or bad news? Why? This would probably be viewed as bad news. A performance index of less than 1.0 indicates that the project, based on current EV and PV information, is not progressing at the planned rate. Depending upon how much less than 1.0 the SPI is, the project’s schedule could either be marginally or significantly delayed.

318 Copyright © 2019 Pearson Education, Inc.


CASE STUDIES Case Study 13.1 The IT Department at Kimble College This case identifies some of the serious problems and challenges involved in accurately tracking and determining the status of ongoing projects. In this case, there is no clear method for tracking and identifying project performance midstream. Either it succeeds, or (more often) it comes in very late and over budget. Dan Gray, the new head of the IT department, is not helping the process because he himself has a tendency to paint a rosy picture of his projects. Questions 1. As a consultant monitoring this problem, what solutions will you propose? To what degree has Dan’s management style contributed to the problems? This department needs to develop a monitoring and control system that allows project managers and administrators the ability to get real-time information on project development so there are no end-game surprises, when a project is “suddenly” late and over budget. The use of earned value, milestones, or some other tracking mechanism is critical. 2. What are some types of project status information you could suggest the project team leaders begin to collect in order to assess the status of their projects? Use of standard monitoring and control metrics such as milestones would begin to give some interim updates on project status. The problem with milestones is that they are a reactive measure (you know you missed one only when you miss one). On the other hand, earned value, combined with frequent updates regarding project activity development, can provide real-time information and well as the ability to make reasonable projections into the future to avoid any surprises regarding how projects are performing. 3. How would you blend “hard data” and “managerial or behavioral” information to create a comprehensive view of the status of ongoing projects in the IT department at Kimble College?? Using concepts such as earned value, coupled with “softer” information provided by tools such as critical success factor analysis, will give project managers and top management a more comprehensive assessment or how projects are performing, how effectively project teams are functioning, and early-warning signs in cases where behavioral issues may be poised to negatively affect the project’s performance. “Hard data” and “soft data” each serve a purpose in detailing a clear view of the project’s current status as well as the status of project team performance, which is critical to the ability to successfully complete the project.

319 Copyright © 2019 Pearson Education, Inc.


Case Study 13.2 The Superconducting Supercollider A famous example of a project that started with great fanfare and was quietly shut down was the Superconducting Supercollider. A particle physics structure as it was conceived, the project received funding after an intense (and some would argue, divisive) competition among various communities seeking to house the complex. A combination of incremental funding coupled with very poor project oversight led to allegations of slipshod work, inflated costs, and unnecessary expenses. All these problems contributed to a rapid decline in the attitude of the Federal Government toward keeping the project alive and it was finally killed through withdrawal of funding. This case also makes an excellent discussion point for that argument that good project management also requires good stakeholder management; that is, keeping all the powerful project stakeholders happy and supportive of the project. Questions 1. Suppose you were a consultant called into the project by the federal government in 1990, when it still seemed viable. Given the start to the project, what steps would you have taken to reintroduce some positive “spin” on the Superconducting Supercollider? This question asks students to think about developing stakeholder management strategies for the project to enhance its reputation. Early warning signs were already emerging about poor cost control and slow, expensive development. However, there was still a window of time in which a canny project manager could have worked to reestablish support for the project from the key funding agencies and powerful congressional members. Students should consider steps to re-engage these crucial supporters. 2. What were the warning signs of impending failure as the project progressed? Could these signs have been recognized so that problems could have been foreseen and addressed or, in your opinion, was the project simply impossible to achieve? Take a position and argue its merits. There are several points of departure that students can adopt in answering this question. First, the divisive nature of the competition for the location of the Superconducting Supercollider was guaranteed to ensure that losing communities, and their federal representatives, would be upset and unlikely to give the project the benefit of the doubt downstream. Second, the way that project funding was initially doled out at a slow pace (due to Federal budget deficit concerns) made it difficult for the project to kick off strongly; in fact, they had to begin slowly and never were able to gain much momentum. Third, the project also was sold on the basis of European financial support, which never materialized. When this lack of funding became evident, it gave the project’s enemies powerful ammunition to move to kill the program. The larger question regarding how much of these problems were foreseeable is a debatable issue and one that can generate a lot of in-class discussion as students take one position or the other. The ultimate goal of this component of the case is for them to develop some guidelines for their own careers in projects, in terms of how to uncover 320 Copyright © 2019 Pearson Education, Inc.


warning signs of project difficulties and what positive steps can be taken to address them before they become debilitating to the project. 3. Search for “Superconducting supercollider” on the Internet. How do the majority of stories about the project present it? Given the negative perspective, what are the top three lessons to be learned from this project? This is a summary question that asks students to consider the lessons to be learned from this disaster. Most Internet sites that address just the science underlying the Superconducting Supercollider offer a mixed view of it and are supportive of the particle physics science that drove its development. Federal watchdog groups, on the other hand, view the project as a classic case of governmental waste with nothing to show for it. Case 13.3 “Dear Mr. President—Please Cancel our Project!”: The Honolulu Elevated Rail Project This case is a great current example of a very expensive project that was kicked off because of an assumed need—to relieve congestion in downtown Honolulu through an elevated urban rail system. Critics argue that in addition to having a ballooning cost, the actual planning was poorly conceived, leaving Honolulu with an intrusive and ugly rail system through the downtown area, ruining panoramic views, and impeding traffic. Additionally, advocates underestimated the power needs for the rail system, requiring the transport authority to renegotiate electricity fees for the system. Finally, the original costs that were assumed for the project were calculated during an economic downturn and with the economy booming again, the costs of the project have gone up dramatically. All of these elements points to a state Governor who is anxious to be rid of the project and hoping that President Trump will deny additional federal funding, in which case the project will likely be cancelled. Questions 1. Why are public works projects like the Honolulu Rail project nearly impossible to stop once they have been approved, even if later cost estimates skyrocket? Public works projects often have the backing of powerful politicians who love the idea of having their name attached to a major project and loath the thought of having it cancelled. Based on the ideas of “escalation of commitment,” it is common for these projects to continue, in the thought that stopping it once it has started would be worse than “cutting the cord” and ending the financial drain. There are many examples of projects that, even in the face of obvious failure, will be continued to the finish in hopes that the final result “won’t be too bad.” 2. Project Management researchers have charged that many large infrastructure projects, like this one, suffer from “delusion” and “deception” on the parts of their advocates. Explain how “delusion” might be a cause of ballooning budgets in this project. How does “deception” affect the final project budget overruns? 321 Copyright © 2019 Pearson Education, Inc.


Faculty are encouraged to draw on the work of Dr. Bent Flyvbjerg here in formulating and discussing this issue, as he made the “delusion” and “deception” observations. His argument is the that delusional optimism, first observed through the Prospect Theory work of Daniel Kahneman, is common with project planning, as planners and politicians always assume the most optimistic times when estimating project durations. They also assume the most favorable conditions and research (and common sense) suggests that this is never the case in real life. As a result, decision-makers enter into project planning with a delusional optimism that inevitably leads to downstream problems. Deception is the deliberate masking of issues of problems for the project and as noted above, it is common for key decision-makers and politicians to downplay problems or potential roadblocks, often knowing in advance that these problems will occur.

PROBLEMS 13.12 Using the following information, develop a simple S-curve representation of the expected cumulative budget expenditures for this project (figures are in thousands).

10

20

Duration (in days) 30 40 50

60

70

80

Activities

4

8

12

20

10

8

6

2

Cumulative

4

12

24

44

54

62

68

70

322 Copyright © 2019 Pearson Education, Inc.


Solution:

13.13 Suppose the expenditure figures in Problem 1 were modified as follows (figures are in thousands).:

10

20

Duration (in days) 30 40 50

Activities

4

8

10

14

20

24

28

8

Cumulative

4

12

22

36

56

80

108

116

60

70

80

Draw this S-curve. What does the new S-curve diagram represent? How would you explain the reason for the different, non-S-shape of the curve? Solution:

323 Copyright © 2019 Pearson Education, Inc.


The “non-S” shape of the curve reflects the fact that project expenditures occurred later in the project, suggesting that the project’s activities may not have followed the traditional life cycle model for resource usage.

13.14 Assume the following information (figures are in thousands): Budgeted Costs for Sample Project Duration (in weeks) 5 10 15 20 25 30 35 40 45 Total Design 4 4 2 Engineer 3 6 12 8 Install 4 12 24 6 Test 2 6 6 4 2 Total Monthly Cumul. a. Calculate the monthly budget and the monthly cumulative budgets for the project. b. Draw a project S-curve identifying the relationship between the project’s budget baseline and its schedule.

324 Copyright © 2019 Pearson Education, Inc.


Solution: a. Budgeted Costs for Sample Project Duration (in weeks) 5 10 15 20 25 30 Design 4 4 2 Engineer 3 6 12 8 Install 4 12 24 6 Test 2 6 Monthly 4 7 12 24 34 12 Total Monthly Cumul. 4 11 23 47 81 93

35

40

45

6 6 99

4 2 4 2 103 105

Total

b. S-Curve rendering—Using Excel spreadsheet option

325 Copyright © 2019 Pearson Education, Inc.


13.15 Use the following information to construct a tracking Gantt chart using MS Project. Activities A B C D E F

Duration 5 days 4 days 3 days 6 days 4 days 2 days

Preceding Activities none A A B, C B D, E

Highlight project status on day 14 using the tracking option and assuming that all tasks to date have been completed on time. Print the output file.

326 Copyright © 2019 Pearson Education, Inc.


Solution:

Microsoft product screen shot(s) reprinted with permission from Microsoft Corporation.

327 Copyright © 2019 Pearson Education, Inc.


13.16 Using the information in Problem 13.15, highlight the project’s status on day 14 but assume that activity D has not yet begun. What would the new tracking Gantt chart show? Print the output file. Solution:

Microsoft product screen shot(s) reprinted with permission from Microsoft Corporation.

328 Copyright © 2019 Pearson Education, Inc.


13.17 Use the following table to calculate project schedule variance based on the units listed (figures are in thousands).

Schedule Variance Work Units A B Planned 20 15 Value Earned 10 10 Value Schedule variance

C 10

D 25

E 20

F 20

Total 110

10

20

25

25

C 10

D 25

E 20

F 20

Total 110

10

20

25

25

100

0

-5

5

5

-10

Solution: Schedule Variance Work Units A B Planned 20 15 Value Earned 10 10 Value Schedule -10 -5 variance

329 Copyright © 2019 Pearson Education, Inc.


13.18 Using the data in the table below, complete the table by calculating the cumulative planned and cumulative actual monthly budgets through the end of June. Complete the earned value column on the right. Assume the project is planned for a 12-month duration and a $250,000 budget.

Activity

Jan

Feb

Staffing

8

7

Blueprinting

4

Prototype Development

Mar

Apr

May

6

2

Full Design

8 3

Construction

10

15

6

14

Plan

%C

15

100

10

100

10

70

8

10

21

67

2

30

32

25

10

10

0

Transfer

Monthly Plan Cumulative Monthly Actual Cumul. Actual

Jun

9

Value

40

330 Copyright © 2019 Pearson Education, Inc.


Solution:

Activity

Jan

Feb

Staffing

8

7

Blueprinting

4

Prototype Development

Mar

Apr

May

6

2

Full Design

8 3

Construction

8 8 10 10

11 19 15 25

8 27 6 31

11 38 14 45

Plan

%C

Value

15

100

15

10

100

10

10

70

7

8

10

21

67

14

2

30

32

25

8

10

10

0

0

Σ=

54

Transfer

Monthly Plan Cumulative Monthly Actual Cumul. Actual

Jun

10 48 9 54

50 98 40 94

331 Copyright © 2019 Pearson Education, Inc.


13.19 Using the data from Problem 13.18, calculate the following values: Schedule Variances Planned Value (PV) Earned Value (EV) Schedule Performance Index (SPI) Estimated Time to Completion Cost Variances Actual Cost of Work Performed (AC) Earned Value (EV) Cost Performance Index (CPI) Estimated Cost to Completion

Solution: Schedule Variances Planned Value (PV) Earned Value (EV) Schedule Performance Index (SPI) Estimated Time to Completion

98 54 EV/PV = 54/98 = .55 (1/.55) × 12 mos. = 21.75 mos.

Cost Variances Actual Cost of Work Performed (AC) Earned Value (EV) Cost Performance Index (CPI) Estimated Cost to Completion

94 54 EV/AC = 54/94 = .58 (1/.58) × $250,000 = $434,622

13.20 You are calculating the estimated time to completion for a project of 12 months’ duration and a budgeted cost of $500,000. Assuming the following information, calculate the Schedule Performance Index and the estimated time to completion (figures are in thousands).

Schedule Variances Planned Value of Work Scheduled (PV)

65

Earned Value (EV)

58

Schedule Performance Index Estimated Time to Completion 332 Copyright © 2019 Pearson Education, Inc.


Solution: Schedule Performance Index (SPI) = 58/65 = .89 Estimated Time to Completion = (1/.89) × 12 months = 13.48 months, or almost 1.5 months behind schedule.

13.21 Suppose, for the Problem 13.20, that your PV was 70 and your EV was 95. Recalculate the SPI and estimated time to completion for the project with this new data.. Solution: Schedule Performance Index (SPI) = 95/70 = 1.36 Estimated Time to Completion = (1/1.36) × 12 months = 8.84 months, or approximately 3 months ahead of schedule. 13.22 You have collected the following data based on three months of your project’s performance. Complete the table. Calculate cumulative CPI (CPIC). How is the project performing after these three months? Is the trend positive or negative?

January February March

EV $30,000 $95,000 $125,500

EVC

AC $35,000 $100,000 $138,000

ACC

CPI

CPIC

EV $30,000 $95,000 $125,500

EVC $30,000 $125,000 $250,500

AC $35,000 $100,000 $138,000

ACC $35,000 $135,000 $273,000

CPI 0.86 0.95 0.91

CPIC 0.86 0.93 0.92

Solution:

January February March

Overall, the project is performing below required levels (assuming CPI should equal 1.0 or higher). However, the original poor value is trending moderately better (moving from 0.86 to 0.92 over three months). Trend is positive but the CPI is still underperforming.

13.23 You have collected EV, AC, and PV data from your project for a five-month period. Complete the table. Calculate SPIC and CPIC. Compare the cost and schedule performance for the project on a month-by-month basis and cumulatively. How would you assess the performance of the project? (All values are in thousands 333 Copyright © 2019 Pearson Education, Inc.


$)

April May June July August

EV 8 17 25 15 7

EVC

AC 10 18 27 18 9

ACC

PV 7 16 23 15 8

PVC

SPI

SPIC

CPI

CPIC

EV 8 17 25 15 7

EVC 8 25 50 65 72

AC 10 18 27 18 9

ACC 10 28 55 73 82

PV 7 16 23 15 8

PVC 7 23 46 61 69

SPI 1.14 1.06 1.09 1.00 0.88

SPIC 1.14 1.09 1.09 1.07 1.04

CPI 0.80 0.94 0.93 0.83 0.78

CPIC 0.80 0.89 0.91 0.89 0.88

Solution:

April May June July August

In analyzing trends for this project, we see from the cumulative columns that the project’s SPI started very strong and has been gradually slipping over the course of the five months recorded, going from 1.14 to 1.04. On the other hand, cumulative CPI has actually been trending more positively in recent months, moving from the original 0.80 to 0.88. Although it is still below our threshold level of 1.0, CPI has shown marginal improvement.

13.24 Assume you have collected the following data for your project. Its budget is $75,000 and it is expected to last four months. After two months, you have calculated the following information about the project:

PV EV AC

= = =

$45,000 $38,500 $37,000

Calculate the SPI and CPI. Based on these values, estimate the time and budget necessary to complete the project. How would you evaluate these findings? Are they good news or bad news?

Solution: SPI = EV/PV = $38,500/45,000 = .86 CPI = EV/AC = $38,500/37,000 = 1.04 334 Copyright © 2019 Pearson Education, Inc.


Estimated Time to Completion = (1/.86) × 4 months = 4.68 months Estimated Cost to Completion = (1/1.04) × $75,000 = $72,078 The findings are a bit of good news and a bit of bad. The good news is that your estimated cost to completion is lower than the original budget; however, the bad news is that the project is behind schedule and is likely to take 4.65 months to complete, rather than the originally planned 4 months.

13.25 You had planned to construct a series of eight traffic roundabouts in your town, with each roundabout costing $150,000. Your goal was to complete the project during the four (4) months of good weather in the summer but due to unexpected delays, the project has not gone according to plan. After two months, earned value (EV) is calculated at $400,000. What is your budgeted cost at completion (BAC)? Solution: This is a bit of a trick question. If we know, in advance, that we are planning eight traffic roundabouts and they cost $150,000 each, the budgeted cost at completion is simply 8 × $150,000 = $1.2 million.

13.26 (Optional—Based on Earned Schedule discussion in Appendix 13.1) Suppose you have a project with a Budget at Completion (BAC) of $250,000 and a projected length of 10 months. After tracking the project for six months, you have collected the information in the table below. a. Complete the table. How do Earned Value SPI (based on $) and Earned Schedule SPI differ? b. Calculate the schedule variances for the project for both Earned Value and Earned Schedule. How do the values differ? Jan Feb Mar Apr May Jun PV ($) 25,000 40,000 70,000 110,000 150,000 180,000 EV ($) 20,000 32,000 60,000 95,000 123,000 151,000 SV ($) -5,000 -8,000 -10,000 -15,000 -27,000 -29,000 SPI ($) 0.80 0.80 0.86 0.86 0.82 0.84 ES (mo.) 0.80 1.47 2.67 3.63 4.33 5.03 SV (t) -0.20 -0.53 -0.33 -0.37 -0.67 -0.97 SPI (t) 0.80 0.74 0.89 0.91 0.87 0.84 Solution: a. The SPI values are relatively similar, the ES SPI figures are initially more positive than earned value figures. b. IEAC for Earned Value is (1/.0.84) x 10 mos. = 11.90 months. IEAC (t) for Earned Schedule is (10/0.84) = 11.90 months. 335 Copyright © 2019 Pearson Education, Inc.


The results are the same in this case; Earned Value Schedule variance is $-29,000 and Earned Schedule variance is −0.97.

MS Project EXERCISES Exercise 13.32 Using the following data, enter the various tasks and create a Gantt chart using MS Project. Assign the individuals responsible for each activity and once you have completed the network, update it with the percentage complete tool. What does the MS Project output file look like? Activity Duration A. Research product 6 B. Interview customers 4 C. Design Survey 5 D. Collect Data 4

Predecessors A A B, C

Resource % Complete Tom Allen 100 Liz Watts 75 Rich Watkins 50 Gary Sims 0

Solution:

Microsoft product screen shot(s) reprinted with permission from Microsoft Corporation.

Exercise 13.33 Now, suppose we assign costs to each of the resources in the following amounts: Resource Tom Allen Liz Watts Rich Watkins Gary Sims

Cost $50/hour $55/hour $18/hour $12.50/hour

Create the resource usage statement for the project as of the most recent update. What are project expenses per task to date?

336 Copyright © 2019 Pearson Education, Inc.


Solution:

Microsoft product screen shot(s) reprinted with permission from Microsoft Corporation.

Exercise 13.34 Use MS Project to create a Project Summary Report of the most recent project status.

Solution:

Microsoft product screen shot(s) reprinted with permission from Microsoft Corporation.

337 Copyright © 2019 Pearson Education, Inc.


Exercise 13.35 Using the data shown in the network precedence table below, enter the various tasks in MS Project. Then select a date approximately halfway through the overall project duration, and update all tasks in the network to show current status. You may assume that Activities A through I are now 100% completed. What does the tracking Gantt look like? Project—Remodeling an Appliance Activity A. Conduct competitive analysis B. Review field sales reports C. Conduct tech capabilities assessment D. Develop focus group data E. Conduct telephone surveys F. Identify relevant specification improvements G. Interface with Marketing staff H. Develop engineering specifications I. Check and debug designs J. Develop testing protocol K. Identify critical performance levels L. Assess and modify product components M. Conduct capabilities assessment N. Identify selection criteria O. Develop RFQ P. Develop production master schedule Q. Liaison with Sales staff R. Prepare product launch

Duration 3 2 5 2 3 3 1 5 4 3 2 6 12 3 4 5 1 3

Predecessors A, B, C D E F G H G J I, K L M M N, O P Q

Solution:

Microsoft product screen shot(s) reprinted with permission from Microsoft Corporation. 338 Copyright © 2019 Pearson Education, Inc.


Exercise 13.36 Use the following information to construct a Gantt chart in MS Project. What is the expected duration of the project (critical path)? Assume the project is halfway finished in terms of the schedule (day 16 completed) but activity completion percentages are as shown. Construct a tracking Gantt chart for the project (be sure to show the percentage complete for each activity). What would it look like? Activity

Duration (in Days)

Predecessors

A B C D E F G

6 2 4 7 10 6 5

None A A C D B, C E, F

% Completed (Day 16) 100% 100% 100% 14% 0% 33% 0%

Solution: The project’s duration is 32 days and the critical path is: A-C-D-E-G. The tracking Gantt view of the project, based on the 16th day in development and the % completed values, would be:

339 Copyright © 2019 Pearson Education, Inc.


Exercise 13.37 Using the data for Problem 13.36, add the resource assignments to each of the activities and input their hourly rates as shown. Construct an earned value chart for the project. Which activities have negative variances? What is the estimate at completion (EAC) for the project? (Hint: Remember to click “Set baseline” prior to creating the EVM table. This table is found by clicking on the “View” tab, then “Tables,” then “Other Tables.”) Resource Name Josh (Activity A) Mary (Activity B) Evan (Activity C) Adrian (Activity D) Susan (Activity E) Aaron (Activity F) Katie (Activity G)

Hourly Rate ($) 12.00 13.50 10.00 22.00 18.50 17.00 32.00

Solution: Partial table showing cost and schedule variances for activities D and F.

340 Copyright © 2019 Pearson Education, Inc.


CHAPTER TWELVE Resource Management Chapter Outline PROJECT PROFILE Environmental Concerns and Political Leadership Impact Fossil Fuel Project Cancellations INTRODUCTION 12.1 THE BASICS OF RESOURCE CONSTRAINTS Time and Resource Scarcity Example: Working with Project Constraints 12.2 RESOURCE LOADING 12.3 RESOURCE LEVELING Example: An In-Depth Look at Resource Leveling Step One: Develop the Resource-Loading Table Step Two: Determine Activity Late Finish Dates Step Three: Identify Resource Overallocation Step Four: Level the Resource-Loading Table 12.4 RESOURCE-LOADING CHARTS 12.5 MANAGING RESOURCES IN MULTIPROJECT ENVIRONMENTS Schedule Slippage Resource Utilization In-Process Inventory Resolving Resource Decisions in Multiproject Environments Summary Key Terms Solved Problem Discussion Questions Problems Case Study 12.1 The Problems of Multitasking Case Study 12.2 “First Come, First Served”: Resource Challenges for Sunrise Restoration Internet Exercises MS Project Exercises PMP Certification Sample Questions Integrated Project—Managing Your Project’s Resources Notes

261 Copyright 2019 Pearson Education, Inc.


TRANSPARENCIES Note for Instructors: To present transparencies in class, please download the PowerPoint Presentations (available on www.pearsonhighered.com) that accompanies this product. The PPT ISBN is 9780134730479.

12.1 CONSTRAINTS ON PROJECTS

1. TECHNICAL CONSTRAINTS 2. PHYSICAL CONSTRAINTS 3. RESOURCE CONSTRAINTS i. PEOPLE i. MATERIALS ii. MONEY iii. EQUIPMENT

262 Copyright 2019 Pearson Education, Inc.


12.2 RESOURCE LOADING TABLE WITH OVER ALLOCATION

Microsoft product screenshot(s) reprinted with permission from Microsoft Corporation.

263 Copyright 2019 Pearson Education, Inc.


12.3 RESOURCE LOADING TABLE

Microsoft product screenshot(s) reprinted with permission from Microsoft Corporation.

264 Copyright 2019 Pearson Education, Inc.


12.4 PRIORITIZING ACTIVITIES FOR RESOURCE ALLOCATION

1. Those with the smallest amount of slack 2. Those with the smallest duration 3. Those with the lowest activity identification number (e.g., those that start earliest in the WBS) 4. Those with the most successor tasks 5. Those requiring the most resources

12.5 EXAMPLE OF RESOURCE LEVELING Fully Developed Task Table for Sample Project Activity A B C D E F G H I J K

Duration 5 4 5 6 6 6 4 7 5 3 5

ES 0 5 5 5 9 10 11 16 15 15 23

EF 5 9 10 11 15 16 15 23 20 18 28

LS 0 6 5 8 10 10 14 16 18 20 23

LF Slack 5 — 10 1 10 — 14 3 16 1 16 — 18 3 23 — 23 3 23 5 28 —

265 Copyright 2019 Pearson Education, Inc.


12.5 (CON’D)

Microsoft product screenshot(s) reprinted with permission from Microsoft Corporation.

266 Copyright 2019 Pearson Education, Inc.


12.5 (CON’D)—SAMPLE PROJECT NETWORK

267 Copyright 2019 Pearson Education, Inc.


12.5 (CON’D)—Activity Float and Resource Needs for the Sample Network

Activity

Duration

Total Float

A B C D E F G H I J K

5 4 5 6 6 6 4 7 5 3 5

0 1 0 3 1 0 3 0 3 5 0

Resource Hours Needed per Week 6 2 4 3 3 2 4 3 4 2 5 Total

Total Resources Required 30 8 20 18 18 12 16 21 20 6 25 194

268 Copyright 2019 Pearson Education, Inc.


12.6 RESOURCE LOADING TABLE FOR SAMPLE PROBLEM

12

Resource Requirements

10 8 6 4 2 0 1

3

5

7

9

11

13

15

17

19

21

23

25

27

Project Days

269 Copyright 2019 Pearson Education, Inc.


12.7 Resource Loading Table for Sample Network When Activity Float is Included January Activity

1 2 3 4 5

A

6 6 6 6 6

8 9 10 11 12

15 16 17 18 19

22 23 24 25 26

February 29 30 31 1 2

B

2 2 2 2

C

4 4 4 4 4

D

3 3 3 3 3

3

E

3

3

3

3

3

3

2

2

2

2

2

2

4

4

4

4

F G

H

3

3

3

3

4

4

I

4

4

4

J

2

2

2

3

3 3  

K

Total

6 6 6 6 6

5 6 7

9 9 9 9 10

8

9

9

9

9

8 9

9

7

7

3

3 3

5 5

5 5 5

5 5

5 5 5

( = Late Finish)

270 Copyright 2019 Pearson Education, Inc.


12.8 – Resource Leveling the Network Table January Activity

1 2 3 4 5

A

6 6 6 6 6

8 9 10 11 12

15 16 17 18 19

February

22 23 24 25 26

29 30 31 1 2

B

2 2 2 2

C

4 4 4 4 4

D

3 3 3 3 3

3

E

3

3

3

3

3

3

3

2

2

2

2

2

2

4

4

4

4

F G

H

3

3

3

3 4

I

4

4

4

4

J

2

2

2

2

3

3 3  

K

Total

6 6 6 6 6

9 9

5 6 7

9 9 7

8

9

9

9

9

9

9

9

9

7

3

3 3

5 5

5 5 5

5 5

5 5 5

( = Late Finish)

271 Copyright 2019 Pearson Education, Inc.


12.9 RESOURCE LEVELING THROUGH SPLITTING ACTIVITIES

Microsoft product screenshot(s) reprinted with permission from Microsoft Corporation.

272 Copyright 2019 Pearson Education, Inc.


12.10 SAMPLE RESOURCE LOADING CHART

8 6 D

F

4

Resources

B

A

C

E

2 2

4

6

8

10

12

14

Project Days

273 Copyright 2019 Pearson Education, Inc.


DISCUSSION QUESTIONS 12.1 Consider a project to build a bridge over a river gorge. What are some of the resource constraints that would make this project challenging? Resource constraints would include money/budget constraints, specialized equipment, large material requirements, and personnel constraints revolving around contractors, and highly skilled construction crew. A bridge is subject to many potential risks related to environment, technology, and physical construction. All put the project in jeopardy of overextending its budget, which can make monetary budgets and necessary reserves a central resource constraint. Due to the specialized equipment and personnel required, scheduling these resources to arrive at appropriate times is important. Equipment rentals and labor can be expensive, so it is important to schedule these resources so that they arrive just in time for use, however this can be difficult to predict accurately. It is also unlikely that the same personnel will be required throughout the project. One construction team may be needed for pouring foundations, while another is hired to pave the driving surface. Again, identifying, sequencing and scheduling these resources is vital to the project’s on time completion.

12.2 For many projects, the key resources to be managed are the project team personnel. Explain in what sense and how project team personnel are often the project critical resource. Project teams are created to perform work on a specific project. Since projects often require a team member’s expertise in more than one area, it is not uncommon for a team member to be assigned to more than one task in the project. The problem comes when these two tasks need to occur simultaneously. If only one person on the team or within the organization can perform the function, then team personnel becomes the critical resource constraining the ability to complete the project as planned. Alternate scheduling has to be made in order to enable completion of both tasks and hence, progress on the project to continue.

12.3 What is the philosophy underlying resource loading? What does it do for our project? Why is it a critical element in effectively managing the project plan? Resource loading identifies the amount of resources needed over the project baseline. It is a process that assigns resources in the required amount to each project activity. Resource loading maps out resource use over the duration of the project. It is a useful tool for team members to determine conflicting uses and over allocation of resources prior to the start of the project. It is critical to effect project management in that it provides a better picture of resource availability, constraints, and misallocation. This allows managers to determine more accurately if extra resources are required and if the project baseline is reasonable. 274 Copyright 2019 Pearson Education, Inc.


12.4 It has been argued that a project schedule that has not been resource-leveled is useless. Do you agree or disagree with this statement? Why or why not? This question requires students to recognize that it is only after a schedule has been resource-leveled that it represents a true perspective of how long activities and the project as a whole are expected to take. It is important that instructors clearly identify the linkage between schedules and resource availability and commitment for viable project timeframes to completion. 12.5 Discuss the nature of “time/cost tradeoffs” on projects. What does this concept imply for our project management practices? When a resource is over allocated, meaning it is needed to perform multiple tasks at one time or is needed in a greater quantity than the current supply, time/cost tradeoff decisions have to be made. Due to the over allocation, project managers must decide to either increase the overall time of the project by extending the schedule to allow extra time to complete both tasks or increase the budget to allow for employment of additional resource use. Either way, the project is going to run over projected figures, it is simply a matter of whether the budget or timetable will be extended.

12.6 When resource-leveling a project, several heuristics can help us prioritize those activities that receive resources first. Explain how each of the following heuristics works and give an example: a. Activities with the smallest slack: Those activities that have the smallest amount of slack are given priority for resources. This is done to reduce the amount of slippage in the overall project, by attempting to meet the demands of tighter deadlines. For example, if activity A has 4 days of slack time and activity B has 3 days of slack time. Then, resources needed to complete B are allocated first. Activity A is scheduled around the resource use of B. b. Activities with the smallest duration: The activity requiring the shortest amount of time to complete may receive resources first. This heuristic focuses on completing a the greatest number of tasks by getting shorter tasks out of the way before engaging in long-term assignments. In this case, if activity A takes 10 days to complete while activity B will take 25 days. Then, A is scheduled first and resources for B are assigned based on remaining availability. c. Activities with the lowest identification number: Tasks starting earlier in the WBS sequence are assigned resources before those occurring later. If A begins on day 1 of the project and B begins on day 6, then resources will be assigned to A first.

275 Copyright 2019 Pearson Education, Inc.


d. Activities with the most successor tasks: Resources are assigned to the task which has the greatest number of subsequent tasks. This attempts to allow increase the number of downstream tasks that are able to get underway. Assuming activity A has 4 successor tasks following its completion and B has six successor tasks. B will be assigned resources first, leaving A to be scheduled with the remaining resource availability. e. Activities with requiring the most resources: Activities requiring the greatest use of resources is assigned priority. Then, those with lesser requirements are allocated. Here, if A requires 10 people and 5 specialized machines for its completion while B only needs 5 people and 1 machine, then A will be scheduled first.

12.7 Multitasking can have an important negative impact on your ability to resourcelevel a project. When team members are involved in multiple additional commitments, we must be careful not to assign their time too optimistically. In fact, it has been said: “Remember, 40 hours is not the same as one week’s work.” Comment on this idea. How does multitasking make it difficult to accurately resource-level a project? Multitasking decreases employee efficiency because employee time is divided among multiple projects. This division creates down time as employees shift from one project to the next. Simple things such as time spent getting to a new location or time switching from onset of thought processes to another eat up time that employees could be spending on task accomplishment. Therefore, scheduling may become difficult because employees’ time devoted to one project begins to affect their time availability for other projects. Managers in charge of scheduling need to be aware that an employee has multiple responsibilities and not assume that he or she can devote their full amount of working hours to one project.

12.8 Why is resource management significantly more difficult in a multiproject environment? What are some rules of thumb to help project managers better control resources across several simultaneous projects? A multiproject environment creates several problems. One is that using resources for one project may mean that those resources are not available for a co-existing project. Another stems from underutilization of resources. One project may hold on to resources even if it is not currently using them. This may occur because the team does not want to relinquish the asset for fear they may not be able to get it back when they need it. On the other hand, shared resources may become spread too thinly resulting in the delay of multiple projects. When shared resources exist in a multiproject environment, a delay (and hence, a longer use of resources) in one project may create delays in other projects. Finally, there may be a decrease in employees’ efficiency and quality as they attempt to juggle competing work assignments. 276 Copyright 2019 Pearson Education, Inc.


Some rules of thumb used by managers to resolve resource allocation problems include: first in line, greatest resource demand, greatest resource utilization, minimum late finish time, and mathematical programming. First in line simply allocates resources based on which projects are entered first. It is essentially a “first-come, first-served” approach. Using greatest resource demand as a guideline, managers allocate based on which projects demand the greater amount of available resources. The project with the highest demand is scheduled first, then the project with the second-highest demand, and so on. The greatest resource utilization approach is similar but allocates based on the degree of resource use. Here, it may not be the project that has the greatest overall use, but rather the greatest depth of use that is scheduled first. The minimum late finish time rule focuses on slack time by scheduling first the project with the earliest late finish time.

277 Copyright 2019 Pearson Education, Inc.


CASE STUDIES Case Study 12.1: The Problems of Multitasking The case, “The Problems of Multitasking,” highlights some of the critical issues that students must absorb in terms of the relationship between project activity scheduling and resource availability. Many firms require that their project team members engage in so many different projects at the same time (multitasking) that resource assignments are no longer viable because they do not recognize the over-committed nature of their resources. The old adage, “40 hours duration is not the same as one week’s work,” illustrates this point. If we can assume that a project resource can fully devote his or her time to one project, then the “40 hours duration” argument above changes – it is, in fact, one week. However, the more projects that are added to our plate, the more difficult it becomes to determine resources needs clearly. Questions 1. How does multitasking confuse the resource availability of project team personnel? As noted above, the more projects my personnel are committed to at the same time, the more difficult it becomes for me to accurately forecast the needs of any specific project. Activity duration estimates become “contaminated” by the interrelationship among the competing project commitments and my ability to forecast resource needs and activity durations gets extremely problematic. 2. “In modern organizations, it is impossible to eliminate multitasking for the average employee.” Do you agree or disagree with this statement? Why? While there is some truth to this statement, due to many firms’ desire to operate with a lean staff, this statement can be a very good source of discussion among students in a classroom setting. One method for minimizing the problems with multitasking is to do a better job of project portfolio management, adding new projects to the portfolio only after considering resource availability in addition to other strategic factors. 3. Because of the problems of multitasking, project managers must remember that there is a difference between an activity’s duration and the project calendar. In other words, 40 hours of work on a project task is not the same thing as one week on the baseline schedule. Please comment on this concept. Why does multitasking “decouple” activity duration estimates from the project schedule? See the general discussion of the case above. Multitasking confuses accurate resource requirements planning because it adds a level of complication to this process. In developing a resource-leveled project, it is critical that all the resource’s commitments be factored into the schedule, not just their commitments to the current project itself. 278 Copyright 2019 Pearson Education, Inc.


Case Study 12.2: “First Come, First Served”: Resource Challenges for Sunrise Restoration This case is intended to highlight the challenges in resource assignment, particularly in the common cases where project managers within the same firm are competing with each other for the use of scarce and valuable human resources to accomplish their tasks. Without clear guidance from top management and a valid priority system, the ability to acquire resources is often the result of chaotic bargaining and negotiation among equals. This case is based on a real situation and the outcomes were very much in line with the way they are described in the case. The business owner did not want to simply invest in more resources for fear that they would be underutilized. He much preferred the system of negotiating among his project managers, even if that led to inefficient utilization of the resources that were available. Students can be asked to take the side of the owner or Tyler to debate the options that Sunrise can use to manage its resources. Questions 1. Describe some of the resource constraints that Sunrise and its project managers are facing. Sunrise and its project managers are dealing with the need to make use of limited inhouse technician resources, as well as low-cost, high-quality, and speedy subcontractors. There is also limited equipment for cleaning and drying damaged construction sites. This equipment (and the human resources) is snatched up on a first come, first served basis. 2. Is Sunrise’s current model of prioritizing resource assignments viable? Why or why not? The question asks students to consider the difference between the “theoretical” priority system for reserving resources and the one-in-use, where valuable resources are often acquired by less-than-open methods. Ken, for example, ignores the priority system for reserving resources and, because of the way the company’s incentives work, has no problem with jumping the queue to grab the resources he needs when he needs them. 3. How could technology alleviate some of Sunrise’s resource management issues? Students can consider different methods for scheduling resources, including developing a master schedule at the company that lists all projects and prioritizes resource assignments. Also, using resource balancing or leveling software could ensure that the right resources are assigned to the most critical projects first. 4. Would Tyler’s suggestion to hire additional technicians and purchase more equipment solve the resource problems at Sunrise? Why or why not? The owner makes a valid point that he doesn’t want to simply pay for surplus resources if they will not be used. They are expensive and, to his mind, it does not make sense to have them in the warehouse just to calm down Tyler’s concerns. Extra resource would eliminate the bottlenecking problems but paying for them may not be cost effective. 279 Copyright 2019 Pearson Education, Inc.


Certainly, it would be appropriate to study the problem to find out projected usage rates before paying for the extra technicians and equipment. 5. Put yourself in Tyler’s position and create a one-page proposal for the Sunrise owner. Outline the current issues briefly with corresponding detailed recommendations for action. Your actionable improvements must be specific, address your (Tyler’s) concerns, align with owner’s priorities, and outline the benefits of your solutions in a persuasive manner. This last question requires student to carefully analyze the overall impact of the resource problem and develop a comprehensive resource management plan. There are no right or wrong answers to this question; it can be evaluated on the basis of thoroughness and clear understanding of the problem and possible solutions.

PROBLEMS 12.9 Consider a project Gantt chart with the following conditions (see Figure 12.22). Susan is your only programmer and she is responsible for Activities 3 and 4, which overlap. In resource-leveling the project so that Susan is only working a maximum of 8 hours each day, what would the new Gantt chart look like? What would be the new project completion date?

Figure 12.22

SOLUTION: Susan is overlapping two days of work (Tuesday and Wednesday of the week of June 29). Delaying the start of Activity 4 by two days until she completes Activity 3 would delay the overall project completion date by three days, as Activity 6 is a successor of the delayed Activity 4.

12.10 Referring to Figure 12.22, how would splitting Activity 3 on May 1 to complete Activity 4 and then finish Activity 3 affect the revised project completion date? Show your work. Do you recommend splitting Activity 3 or allow Susan to first complete it and then perform Activity 4? Which strategy would allow the project to finish sooner? Why? 280 Copyright 2019 Pearson Education, Inc.


SOLUTION: Splitting Activity 3 would not affect the completion date of the project because splitting Activity 3 would lead to a 2 day lag in its completion, thereby delaying the start of Activity 5 by 2 days. The overall project delay due to splitting Activity 3 would be to delay the project by 3 days. Splitting is not a better alternative (See Figure 12.22a) Figure 12.22a – Solution

12.11 Refer to the Gantt chart in Figure 12.23. Bob and George are carpenters who have been scheduled to work on the construction of a new office building. Just before the start of the project, George is injured in an accident and cannot work this job, leaving Bob to handle his own activities as well as George’s. Resource level this Gantt chart with Bob now responsible for Activities 3, 4, 6, and 7. What is the new projected completion date for the project? SOLUTION: The current planned duration for the project is 17 days. When Bob is required to complete his activities as well as George’s, the result will be to lengthen the project to 21 days, assuming resource leveling to maintain the standard 8-hour day. Bob would have to delay Activity 4 until he completed Activity 3. As this activity is 4 days in length and is the only resource conflict in the current schedule, the result would be to slide the entire schedule by 4 days.

12.12 Referring to Figure 12.23, because George is unavailable, suppose you have the opportunity to hire two new carpenters to perform George’s tasks (shortening them by 50%). What would be the new projected completion date for the project? Would it be worth it to you to hire two replacement carpenters instead of just one? Show your work.

281 Copyright 2019 Pearson Education, Inc.


Figure 12.23

SOLUTION: Reducing Activity 4 by 2 days due to adding the extra resources would conflict with Bob’s being needed to complete Activity 6 (see Figure 12.23a). To correct the problem, you would still end up with a completion date of 21 days (see Figure 12.23b) Figure 12.23a

282 Copyright 2019 Pearson Education, Inc.


Figure 12.23b

For Problems 12.13 to 12.17, consider a project with the following information:

Activity A B C D E F G H

Activity A B C D E F G H

Duration 3 5 7 3 5 4 2 5

Duration 3 5 7 3 5 4 2 5

Predecessors -A A B, C B D C E, F, G

ES 0 3 3 10 8 13 10 17

EF 3 8 10 13 13 17 12 22

LS 0 5 3 10 12 13 15 17

LF 3 10 10 13 17 17 17 22

Slack — 2 — — 4 — 5 —

283 Copyright 2019 Pearson Education, Inc.


Activity

Duration

Total Float

A B C D E F G H

3 weeks 5 weeks 7 weeks 3 weeks 5 weeks 4 weeks 2 weeks 5 weeks

— 2 — — 4 — 5 —

Resource Hours Total Resources Needed per Required Week 6 18 4 20 4 28 6 18 2 10 4 16 3 6 6 30 Total 146

12.13 Construct the project activity network using AON methodology. 12.14 Identify the critical path and other paths through the network. 12.15 Create a time-phased resource loading table for this project, identifying the activity early start and late finish points 12.16 Assume that there is a maximum of eight resource hours per week available for the project. Can you identify any weeks with resource over commitments? 12.17 Resource-level the loading table. Identify the activity that can be rescheduled and reconfigure the table to show this reallocation.

284 Copyright 2019 Pearson Education, Inc.


Solution to 12.13:

Microsoft product screenshot(s) reprinted with permission from Microsoft Corporation.

Solution to Problem 12.14: Alternative Paths through the Network: 1. A – B – E – H 2. A – B – D – F – H 3. A – C – G – H Critical Path: A – C – D – F – H

285 Copyright 2019 Pearson Education, Inc.


Solution to Problem 12.15: Weeks Activity

1

2

3

A

6

6

6]

4

5

6

7

8

B

4 4

4

4

4

C

4 4

4

4

4

9

10

12

13

6

6

6]

2

2

2

14

15

16

17

4

2

2

G

3

4

4

6

8

8

8

8

8

22

6

6

6

6

6

6

6

6

6

6

4]

3

]

H

6

21

] 4

6

19 20

4]

F

Total

18

]

D E

11

6

6

11

11

8

4

4

4

4

] = Late Finish

286 Copyright 2019 Pearson Education, Inc.



Solution to Problem 12.16: Resources are overcommitted in weeks 11 and 12 by three hours each (total of 11 hours are committed during those weeks).

288 Copyright 2019 Pearson Education, Inc.



Solution to Problem 12.17: Weeks Activity

1

2

3

A

6

6

6]

4

5

6

7

8

B

4 4

4

4

4

C

4 4

4

4

4

9

10

12

13

6

6

6]

2

2

2

14

15

16

17

4

2

2

G

3

4

3

4

4]

3

3]

H

6

6

8

8

8

8

8

21

22

6

6

6

6

6

6

6

6

6

6

] 4

6

19 20

4]

F

Total

18

]

D E

11

6

6

11 11 8 8

8

4

4

4 7

4 7

] = Late Finish

290 Copyright 2019 Pearson Education, Inc.



Problem 12.18: Consider the partial resource-loading chart shown below. Suppose that you can commit a maximum of eight resource hours per day.. a. What are the dates on which project resources are overallocated? b. How should the resource-loading table be reconfigured to correct for this overallocation? c. Now, suppose that the maximum resources hours per day you can commit is reduced to six. How would you reconfigure the resource-loading table to adjust for this number? What would be the new project completion date?

Project Calendar Dates June Activity

1 2 3 4 5

A

6 6 6 6 6]

8 9 10 11 12

B

2 2 2 2

C

4 4 4 4 4]

D

3 3 3 3 3

15 16 17 18 19

22 23 24 25 26

]

]

E

4

4

4

4

4]

F

2

2

2

2

2

2

G

Total

6 6 6 6 6

9 9 9 9 7

6

6

6

6

6

2

] 4

4

4

4

4

4

]

292 Copyright 2019 Pearson Education, Inc.


Solution to Problem 12.18 (a): Over allocation occurs on the week of the 8th through the 11th. Total hours allocated for each of these four days is nine.

Solution to Problem 12.18 (b): The simplest method would be to delay the start of Activity B to its late finish dates as shown in the figure below.

Project Calendar Dates June Activity

1 2 3 4 5

A

6 6 6 6 6]

8 9 10 11 12

B

15 16 17 18 19

2 2

C

4 4 4 4 4]

D

3 3 3 3 3

2

2]

22 23 24 25 26

]

E

4

4

4

4

4]

F

2

2

2

2

2

2

G

Total

6 6 6 6 6

7 7 7 7 7

8

8

8

8

6

2

] 4

4

4 ]

4

4

4

293 Copyright 2019 Pearson Education, Inc.


Solution to Problem 12.18 (c): Because activities C and D have little slack time, pushing them off to their Late Finish points is not a viable option. Activity D would have to be moved later, affecting the starting dates for Activities E, F and G. The new completion date for the project would be July 1, or three days late. Project Calendar Dates June

Activity

1 2 3 4 5

A

6 6 6 6 6

8 9 10 11 12

B

2 2

2

2

C

4 4

4

4 4

D

July

15 16 17 18 19

3

3

3

3

22 23 24 25 26

3

E

4

F

2

2

2

2

2

4

4

4

4

2

G

Total

6

6 6 6 6

6 6 6 6 4

5

5

5

5

29 30 1

5

6

4

4

4

4

4

4

4

4

4

4

294 Copyright 2019 Pearson Education, Inc.


12.19 Suppose you have the following information about Project Cross-Talk. Activity Duration Predecessors Resource Assigned A 5 --Beth B 5 A Sam C 6 A Jenny D 13 B, C Sam E 6 B Jenny F 4 D Frank G 9 C Bill H 2 E, F, G Kate Consider the project Gantt chart shown in Figure 12.24. Identify the resource constraints in the network. Who are the resources who have been incorrectly (over) assigned? What evidence do you have to support this view? a. Is Jenny really a resource bottleneck due to her assignment to Activities C and E? Why or why not (Hint: remember the slack in the project schedule)? b. Suppose Bill quit the company at the start of the project and left you shortstaffed, forcing you to have Jenny work on Activity G, in addition to her other assignments. How will adding Activity G to her responsibilities affect the project completion date? c. Suppose only Sam is capable to performing Activity G, in addition to his other current assignments. How will resource-leveling the project because of Sam and his resource conflicts affect the project completion date?

SOLUTION: a. Jenny is not really a bottle-neck because there is slack time in her second Activity (E). Although the current schedule shows overlap between activities C and E for which Jenny is responsible, Activity E can be delayed for the overlapped 1 day without delaying the project. b. Jenny would be responsible for both Activities E and G, which are occurring in parallel for a significant part of their durations. Students could use MS Project to resource level this network in order to discover the impact of Jenny’s dual assignments. They would show a significant delay in the project as the final schedule is pushed out because of the need to resource-level Jenny’s work across both Activities E and G. c. The answer to part c is similar to the answer to part b above. In either case, Sam (or Jenny), adding the extra task assignment to their responsibilities will 295 Copyright 2019 Pearson Education, Inc.


make the project much more delayed due to the resource over-allocation. MS Project can again be used to resource level the project to discover how Sam’s overallocation affects the final project delivery date.

MS PROJECT EXERCISES Exercise 12.23 Refer to the activity network table below. Enter this information using MS Project to produce a Gantt chart. Assume that each resource has been assigned to the project activity on a full-time (8 hours/day or 40 hours/week) basis.

Activity Duration A. User survey 4 B. Coding 12 C. Debug 5 D. Design interface 6 E. Develop training 5

Predecessors None A B A, C D

Resource Assigned Gail Wilkins Tom Hodges Wilson Pitts Sue Ryan Reed Taylor

Solution:

Microsoft product screenshot(s) reprinted with permission from Microsoft Corporation.

Exercise 12.24 Using the above information from Exercise 12.23, produce a Resource Usage sheet, identifying the total number of hours and daily commitments of each project team member.

296 Copyright 2019 Pearson Education, Inc.


Solution: The following is a partial Resource Usage sheet, identifying the team members and their total work hour commitments.

Microsoft product screenshot(s) reprinted with permission from Microsoft Corporation.

297 Copyright 2019 Pearson Education, Inc.


Exercise 12.25 Refer to the activity network table shown in Exercise 12.23. Suppose that we modified the original table slightly to show the following predecessor relationships between tasks and resources assigned to perform these activities. Enter this information using MS Project to produce a Gantt chart. Assume that each resource has been assigned on a fulltime (8 hours/day or 40 hours/week) basis. Activity Duration A. User survey 4 B. Coding 12 C. Debug 5 D. Design interface 6 E. Develop training 5

Predecessors None A A B, C D

Resource Assigned Gail Wilkins Tom Hodges Tom Hodges Sue Ryan Reed Taylor

A. Using the Resource Usage view, can you determine any warning signs that some member of the project team has been overassigned? Solution: The output highlights (in red) the warning that Tom Hodges has been overassigned to multiple simultaneous tasks.

Microsoft product screenshot(s) reprinted with permission from Microsoft Corporation.

B. Click on the Resource Graph view to determine the specific days when there is a conflict in the resource assignment schedule. Solution: The Resource Graph screen indicates the exact dates that show a resource assignment conflict, providing the user with the basis for leveling the activity network.

298 Copyright 2019 Pearson Education, Inc.


Microsoft product screenshot(s) reprinted with permission from Microsoft Corporation.

Exercise 12.26 Using the information provided in Exercise 12.25, how might you resource-level this network to remove the conflicts? Show how you would resource-level the network. From a schedule perspective, what is the new duration of the project? Solution: The challenge here lies in reassigning Tom Hodges in such a manner that the schedule no longer shows conflict among his project responsibilities. Clicking on the Tools taskbar shows a Resource Leveling function, under which MS Project will automatically smooth out any resource conflicts (see below).

299 Copyright 2019 Pearson Education, Inc.


The new Gantt chart is shown below.

Microsoft product screenshot(s) reprinted with permission from Microsoft Corporation.

300 Copyright 2019 Pearson Education, Inc.


CHAPTER ELEVEN Advanced Topics in Planning and Scheduling: Agile and Critical Chain Chapter Outline PROJECT PROFILE General Electric—Using Agile Methods to Speed New Project Delivery INTRODUCTION 11.1 AGILE PROJECT MANAGEMENT What Is Unique About Agile PM? Tasks Versus Stories Key Terms in Agile PM Steps in Agile Sprint Planning Daily Scrums The Development Work Sprint Reviews Sprint Retrospective Keys to Success with Agile Problems with Agile PROJECT MANAGEMENT RESEARCH IN BRIEF Does Agile Work? 11.2 EXTREME PROGRAMMING (XP) 11.3 THEORY OF CONSTRAINTS AND CRITICAL CHAIN PROJECT SCHEDULING Theory of Constraints 11.4 THE CRITICAL CHAIN SOLUTION TO PROJECT SCHEDULING Developing the Critical Chain Activity Network Critical Chain Solutions Versus Critical Path Solutions PROJECT PROFILE Eli Lilly Pharmaceuticals and Its Commitment to Critical Chain Project Management 11.5 CRITICAL CHAIN SOLUTIONS TO RESOURCE CONFLICTS 11.6 CRITICAL CHAIN PROJECT PORTFOLIO MANAGEMENT PROJECT MANAGEMENT RESEARCH IN BRIEF Advantages of Critical Chain Scheduling 11.7 CRITIQUES OF CCPM

229 Copyright 2019 Pearson Education, Inc.


Summary Key Terms Solved Problem Discussion Questions Problems Case Study 11.1 It’s an Agile World Case Study 11.2 Ramstein Products, Inc. Internet Exercises Notes

230 Copyright 2019 Pearson Education, Inc.


TRANSPARENCIES Note for Instructors: To present transparencies in class, please download the PowerPoint Presentations (available on www.pearsonhighered.com) that accompanies this product. The PPT ISBN is 9780134730479.

Figure 11.1 WATERFALL MODEL FOR PRODUCT DEVELOPMENT

231 Copyright 2019 Pearson Education, Inc.


Figure 11.2 SCRUM PROCESS FOR PRODUCT DEVELOPMENT

232 Copyright 2019 Pearson Education, Inc.


Figure 11.3 MEMBERS OF THE SCRUM TEAM

233 Copyright 2019 Pearson Education, Inc.


Figure 11.4 STAGES IN AN AGILE SPRINT

234 Copyright 2019 Pearson Education, Inc.


Figure 11.5 SAMPLE BURNDOWN CHART FOR DAY 9 OF A SPRINT

235 Copyright 2019 Pearson Education, Inc.


Figure 11.6 FIVE KEY STEPS IN THEORY OF CONSTRAINTS METHODOLOGY

1. Identify the system constraint

5. Reevaluate the system

4. Elevate the system constraint

2. Exploit the system constraint

3. Subordinate everything else to the system constraint

236 Copyright 2019 Pearson Education, Inc.


Figure 11.7 REDUCTION IN PROJECT DURATION AFTER AGGREGATION

237 Copyright 2019 Pearson Education, Inc.


Figure 11.8 CCPM NETWORK EMPLOYING FEEDER BUFFERS

Noncritical Activity X

Noncritical Activity Y

Feeder Buffer

Critical Activity A

Critical Activity B

Critical Activity C

Critical Activity D

Project Buffer

238 Copyright 2019 Pearson Education, Inc.


Figure 11.9 EXAMPLE—ORIGINAL PROJECT SCHEDULE USING EARLY START A (10)

B (50)

E (30) C (20)

Slack

D (10)

90 Days

Figure 11.9 EXAMPLE (CON’D)—REDUCED SCHEDULE USING LATE START

A (5)

B (25)

E (15)

C (10)

45 Days Copyright 2019 Pearson Education, Inc.

D (5)

239


Figure 11.9 EXAMPLE (CON’D)—CRITICAL CHAIN SCHEDULE WITH BUFFERS ADDED

A (5)

B (25)

E (15)

C (10)

D (5)

Project Buffer (22.5)

Feeder Buffer (7.5)

67.5 Days

240 Copyright 2019 Pearson Education, Inc.


Figure 11.10 CRITICAL PATH NETWORK WITH RESOURCE CONFLICTS

Feeder Buffer

Bob

Bob

Critical Path

Bob

Feeder Buffer

241 Copyright 2019 Pearson Education, Inc.


Figure 11.11 THE CRITICAL CHAIN SOLUTION

Feeder Buffer

Bob

Feeder Buffer

Bob

Feeder Buffer

Project Buffer

Bob

The Critical Chain is shown as a dotted line

242 Copyright 2019 Pearson Education, Inc.


Figure 11.12 CRITICAL CHAIN PORTFOLIO MANAGEMENT—THREE PROJECTS STACKED TO USE A DRUM RESOURCE

C

C

Resource Supply

B

B

A

Priority: 1. Project A 2. Project B 3. Project C

B

A

A

Time

243 Copyright 2019 Pearson Education, Inc.


Figure 11.13 APPLYING CCBs TO DRUM SCHEDULES

Resource Supply

CCB B

B

A

C

A

B

A

C

Time A & B start immediately

Project C start date

244 Copyright 2019 Pearson Education, Inc.


DISCUSSION QUESTIONS 11.1

What are the practical implications internally (in terms of team motivation) and externally (for the customer) of making overly optimistic project delivery promises?

Optimistic project delivery predictions can actually demotivate project team members if they are viewed as unattainable. There is a big difference between “push goals” that are intended to motivate, but still are viewed as “doable”, and too-optimistic promises that are immediately perceived as impossible. In the short term, the temptation to use them is understandable because while they may frustrate your project team, they are welcomed by the client. However, as the project continues to drag along and the delivery dates slip, the client will become increasingly angry with you, while your team resents the pressure you must now increasingly bring to bear on them to try and salvage an impossible delivery schedule that your over-optimism caused in the first place.

11.2

In considering how to make a big change in organizational operations (as in the case of switching to CCPM), why might it be necessary to focus on changing the organization’s current culture? That is, why does a shift in project scheduling require so many other linked changes to occur?

A number of problems confront us when we consider any significant organizational change; particularly a change that can affect the culture and/or basic operations of an organization. A systematic approach that applies a change methodology in a series of programmatic steps offers the best probability of creating change that is lasting and leads to significant improvement in operations.

11.3

Why are traditional project planning methods insufficient when project deliverables are subject to changing requirements or continuous input from the project client?

The changing requirements that come from continuous input from clients cannot work with a traditional waterfall project development process. Waterfall assumes a linear process for project development, from requirements gathering through design, and so on. This process is flawed for several classes of project, such as IT, that are based on rapid design, changing needs, technological shifts, and other pressures throughout the project’s development. As the text suggests, the philosophy of simply gathering initial requirements, disappearing for an extended period, and then presenting the client with the finished product is not a recipe for success.

245 Copyright 2019 Pearson Education, Inc.


11.4

What are the advantages and disadvantages of the waterfall planning model for project development?

Waterfall planning is useful under conditions where: (a) requirements are very well understood and fixed at the outset of the project; (b) product definition is stable and not subject to changes; (c) technology is understood; (d) ample resources with required expertise are freely available; and, (e) the project is of short duration.

11.5

What are the advantages and disadvantages of Agile PM?

Advantages—designed for unpredictable environments, including changing customer requirements and technology shifts; useful for complex projects requiring multiple iterations; useful for projects of long-time frame; work well when initial requirements are vague or likely to change over the development cycle. Agile recognizes the importance of user stories as the best means to develop specifications; that is, understanding what the user actually does, not just what they say they think they need; finally, Agile focuses on product “features” rather than a lengthy WBS that fixes our scope. Disadvantages—Agile is not useful for short-term projects or those with a fixed and unchangeable set of requirements; Agile can get out of control if customers and the scrum team allow constant change with no spec freeze ever (i.e., there is a fine line between modifying the project and constantly shifting it to the point where it is never “right”); potential for scope creep due to evolving specs; hard to predict at the beginning of a project what the end product will look like; because requirements are kept to a minimum, there can be confusion about the final outcomes; integrated testing through the development adds to costs; testing and sign-offs are nearly continuous throughout the development.

11.6

How are the duties of the Scrum Master like a project manager? How do they differ?

The Scrum Master is responsible for moving the project forward between iterations, removing impediments, or resolving differences of opinion between major stakeholders. In many ways, the Scrum Master can function as a project manager but their role is typically related to only planning activities that are associated with the Agile sprints and Scrum meetings. They do not play a role in “people management” and instead focus only on maintaining the development process and enforcing the rules of the Agile methodology.

11.7

Why is a focus on project features and user stories important when developing requirements?

Focusing on features and user stories removes the need for the project developers to either force the client to fully identify all requirements initially or completely understand 246 Copyright 2019 Pearson Education, Inc.


the technology and how it will be used. With an emphasis on stories and features, the project developers learn to understand what the client really does and what they need the project to do for them. This takes the burden of technical knowledge off the user and forces the technical people on the project team to think in terms of utility, not simply adding “gold plating” to a project that may not be useful. In this way, project success is now defined in terms of usefulness of the new product.

11.8

What would be the difficulties in using Extreme Programming (XP) to develop projects? What types of projects would be best suited to employing XP?

XP requires paired programmers to collaborate throughout code development. Refactoring can add to costs because all testing and improvements occur in real time rather than at the end of the initial coding cycle. Complex IT projects or software projects that involve extensive coding are among the best uses for XP.

11.9

How does aggregation of project safety allow the project team to reduce overall safety to a value that is less than the sum of individual task safeties? How does the insurance industry employ this same phenomenon?

Aggregation of safety is based on reducing the overall amount of slack by applying a buffer at the project, not individual task, level. To do this, aggregation first calculates the total amount of slack added to duration estimates. This is done by adding the durations for all the activities and arriving at the project overall duration estimate. This number is then cut in half. There are now two project duration estimates, the original and the reduced estimate. The midpoint of these two numbers is the desirable duration estimate (under the aggregation technique). As a result, the project safety is 50% less than the sum of individual task safeties. The insurance industry uses the Central Limits Theorem to arrive at a realistic estimate of potential risk.

11.10 Distinguish between project buffers and feeder buffers. What is each buffer type used to accomplish? Project buffers are used to create slack in the critical chain, thereby lengthening the entire duration estimate of the project. On the other hand, feeder buffers are used where noncritical and critical paths intersect. The feeder buffer is the amount of time between when noncritical activities will be completed and when they need to join with activities in the critical path. These buffers are used to eliminate delays in the critical path.

11.11 It has been said that a key difference between CCPM safety and ordinary PERT chart activity slack is that activity slack is determined after the network has been created, whereas critical chain path safety is determined in advance. 247 Copyright 2019 Pearson Education, Inc.


Explain this: How does the project team “find” slack in a PERT chart versus they use the activity buffer in Critical Chain Project Management? In PERT, team members establish buffer time for each individual activity. PERT relies on task dependencies meaning that slack in one task can be affected by events upstream that reduce or inflate safety. Because resource leveling is done prior to scheduling, CCPM does not rely on task dependencies for establishing slack. CCPM adds buffer at the project level using aggregation, which reduces the overall amount of slack in the project.

11.12 What are the steps that CCPM employs to resolve conflicts on a project? How does the concept of activity late starts aid this approach? CCPM begins by establishing the availability of critical resources. The critical chain is then created based on these availabilities so that delays are not created by lack of a resource. Feeder buffers (instead of activity buffers in the critical chain) are used to adjust for resource availability. However, conflicts may still arise that require a resource to be available for two activities at one time. To resolve this, CCPM uses late starts. The method applies “start-as-late-as-possible” times to preceding tasks to the later-starting activity in the conflict. Working backwards from this activity, each predecessor is given a late-start time until the activities in conflict no longer overlap.

11.13 What key steps are necessary to employ CCPM as a method for controlling a firm’s portfolio of projects? First, all current portfolio projects are compiled. Next, the chief resource constraint must be identified. Then, the constraint is exploited. After that, sequencing of portfolio projects is determined around the constraint. Buffering between projects, called capacity constraint and drum buffers, may be used to create safety between projects to ensure proper flow and use of the constrained resource. The next step is to evaluate the constraint and increase the drum capacity if possible. Lastly, the steps are repeated to improve flow and resource levels.

11.14 What is a drum resource? Why is the concept important to understand in order to better control resource requirements for project portfolios? A drum resource is a systemwide resource constraint. The drum resource limits the production capacity of the entire firm. Therefore, the concept must be understood because it affects how all projects in the portfolio need to be scheduled. It also affects the number of projects the firm can support at one time.

248 Copyright 2019 Pearson Education, Inc.


CASE STUDIES Case Study 11.1: It’s an Agile World This case illustrates a common problem in software and IT development, where programmers and IT staff are anxious to lock in specifications as early as possible so they can “get to work” without having to worry about invasive or disruptive input from the end users. Unfortunately, what typically happens is that the finished product is not what the users needed or thought they needed and a long list of fixes and modifications are needed to make it work correctly. This case is based on a true story in a hospital IT department that routinely struggled with these sorts of user conflicts until they sifted to an Agile methodology. Questions 1. Why does the classic waterfall project planning model fail in this situation? What is it about the IT department’s processes that leads to their finished systems being rejected constantly? As the case suggests, constantly changing user requirements, governmental regulations, and other technological changes make it necessary for updating the initial user specifications. Programmers like to lock in features as early as possible but with extended development cycles, this almost guarantees that the final product will not be what the end user actually needs and can immediately use. 2. How would an Agile methodology correct some of these problems? What new development cycle would you propose? Agile would allow programmers to work closely with system users throughout the development cycle, while creating an iterative development process based on first understanding user needs and allowing for any regulatory changes or other features that will make the user happier with the finished product. Students can suggest alternative development cycles here that seem to make sense, based on an understanding of the sprints involved in such a project, the average development and delivery cycle, and so forth. 3. Why are “user stories” and system “features” critical components of an effective IT software development process? As this case illustrates, it is critical to hear what the client asks for but more important is recognizing what they do on their job, the features of the project that are most useful to them, and how external issues, such as governmental regulatory changes or changing technology can be integrated into the product before it is implemented. 4. Using the terms “Scrum,” “Sprint,” and “User stories,” create an alternative development cycle for a hypothetical software development process at Northwest Regional Hospital. This question asks the students to create an alternative development cycle. They may identify the people who should be part of the Scrum team, the various elements in the development cycle, and some sample burndown charts for different sprints as part of this question. There is no right or wrong answer—it requires students to understand how these 249 Copyright 2019 Pearson Education, Inc.


terms apply to a real-project situation and modify the traditional waterfall model appropriately.

Case Study 11.2: Ramstein Products, Inc. The Ramstein case is an example of the problems that often occur once we have padded our activity estimates, through wasting project safety in a variety of ways. The case mentions that Jack is eager to fix project overruns, which are becoming more prevalent, but does not have the option of simply adding resources to his department. Any solutions must come from dealing with existing issues and fixing them. Questions 1. Applying Goldratt’s idea of critical resources, what is the system constraint within the Special Projects Division that is causing bottlenecks and delaying the projects? The primary system constraint is the seven system integration engineers who must support a large portfolio of projects. All project activities should be scheduled around the availability of this critical resource, but there is no indication that the organization is clearly recognizing this critical resource constraint, much less that they are attempting to reorder projects to most efficiently use the resources. 2. How is multitasking contributing to systemic delays in project development at Ramstein? The chapter discusses the impact that multitasking has on resources’ ability to effectively manage time across multiple projects. As the chapter demonstrates, multitasking serves to waste project safety by not permitting resources to apply themselves completely to one task before moving on to another. 3. How could the drum buffer concepts from Critical Chain Portfolio Management be applied to this problem? When we recognize that the seven system integration engineers are the “drum” in this example, we can set up a sample chart to schedule projects for access to the drum resource. Instructors could ask students to create a simplified chart that identifies the manner in which the drum resource constrains the system and the manner in which Jack must schedule all projects in order not to overload the critical resource (using the capacity constraint buffer process).

250 Copyright 2019 Pearson Education, Inc.


PROBLEMS 11.15 Assume the network diagram shown in Figure 11.19. Megan is responsible for activities A and C. Use the Critical Chain methodology to resource level the network. What are two options for redrawing the network? Which is the most efficient in terms of time to completion for the project? Show your work.

Figure 11.19

Microsoft product screenshot(s) reprinted with permission from Microsoft Corporation.

Solution: One solution is to order Megan’s work so that she performs Activity A first and then Activity C. The MS Project output is shown below and the expected duration for these five activities is now 45 days, delayed five days from the original schedule.

251 Copyright 2019 Pearson Education, Inc.


Microsoft product screenshot(s) reprinted with permission from Microsoft Corporation.

An alternative is to reconfigure the order to that Megan first performs Activity C and then completes Activity A (see alternative solution below). Because of precedence in the activities, this solution will lead to a longer total duration for the five activities of 48 days; thus, the first solution is better as it saves 3 days.

Microsoft product screen shot(s) reprinted with permission from Microsoft Corporation.

252 Copyright 2019 Pearson Education, Inc.


11.16 Consider the following activities and their durations. The original project schedule, using early activity starts, is shown in Figure 11.20. Reconfigure the network using critical chain project scheduling. Activity A B C D E

Duration 5 days 30 days 10 days 10 days 15 days

A (5)

B (30)

E (15)

C (10)

Slack

D (10)

50 Days

Figure 11.20

a. What is the critical path? b. How much slack is currently available in the non-critical path? c. Reconfigure the network in Figure 11.20 as a critical chain network. What is the new duration of the project? How long are the project and feeder buffers?

253 Copyright 2019 Pearson Education, Inc.


Solution: In the current network configuration, the feeder path (Activities C and D) has 15 days of slack. Total project length is 50 days. When we reconfigure the network as shown below, the new project length is 37.5 days, the feeder buffer is 5 days and the project buffer is 12.5 days (half of the total project time savings reapplied as project buffer).

A (2.5)

B (15)

E (7.5) C (5)

D (5)

PB (12.5)

FB (5) 37.5 Days

254 Copyright 2019 Pearson Education, Inc.


11.17 Reconfigure the network in Figure 11.21 using the critical chain approach. Remember to reconfigure the activities to late start where appropriate. What is the original critical path? What is the original project duration? How much feeder buffer should be applied to the noncritical paths? What is the length of the project buffer? Assume the 50% likelihood is exactly half the duration of current project activities.

Figure 11.21 A (12)

B (10)

D (8)

C (15)

E (10)

F (18)

H (15)

G (15)

255 Copyright 2019 Pearson Education, Inc.


Solution: The noncritical paths are Activities D and E, with 19 days of slack and Activities F and G, with 4 days of slack. The original project duration is 52 days. Reconfiguring this network as a critical chain approach would yield the following:

A (6)

B (5)

D (4)

F (9)

E (5)

G (7.5)

C (7.5)

FB (4.5)

H (7.5)

PB (13)

FB (2)

39 Days

256 Copyright 2019 Pearson Education, Inc.


11.18 Assume the network in Figure 11.22 with resource conflicts. How would you redraw the network using a critical chain to eliminate the resource conflicts? Where should feeder buffers be applied? Why?

Figure 11.22 Feeder Buffer

Joe

Joe CRITICAL PATH

Joe

Feeder Buffer

257 Copyright 2019 Pearson Education, Inc.


Solution: Applying critical chain methodology in order to account for resource conflicts, we would create a different path through the network to get most efficient use of the “system constraint,” in this case, Joe. The reconfigured network, showing a replacing of feeder buffers and reordering of Joe’s activities, would be the following:

Feeder Buffer

Joe

Feeder Buffer

Joe

Feeder Buffer

Project Buffer

Joe

The Critical Chain is shown as a dotted line.

258 Copyright 2019 Pearson Education, Inc.


11.19 Consider the project portfolio problem shown in Figure 11.23. You are required to manage resources to accommodate the company’s current project portfolio. One resource area, comprising Carol, Kathy, and Tom, is responsible for all program debugging as new projects are completed. Four projects have activities that need to be completed. How would you schedule Carol, Kathy, and Tom’s time most efficiently? Using buffer drum scheduling, reconfigure the schedule below to allow for optimal use of the resource time. Priority: Project X Project Y Project Z Project Q Where would you place capacity constraint buffers? Why? Figure 11.23

Q

Q

Resource Supply

Z

Y

Z

Y

X

Y

X

X

Solution: Time

259 Copyright 2019 Pearson Education, Inc.


Resource Supply

Z

Y

Y

Z

CCB

X

Q

X

Y

X

Q Time

X & Y start immediately

Project Q start date

260 Copyright 2019 Pearson Education, Inc.


CHAPTER TEN Project Scheduling Lagging, Crashing, and Activity Networks Chapter Outline PROJECT PROFILE Kiruna, Sweden—A Town on the Move! INTRODUCTION 10.1 LAGS IN PRECEDENCE RELATIONSHIPS Finish to Start Finish to Finish Start to Start Start to Finish 10.2 GANTT CHARTS Adding Resources to Gantt Charts Incorporating Lags in Gantt Charts PROJECT MANAGERS IN PRACTICE Chris Fultz, Rolls-Royce Plc. 10.3 CRASHING PROJECTS Options for Accelerating Projects Crashing the Project: Budget Effects 10.4 ACTIVITY-ON-ARROW NETWORKS How Are They Different? Dummy Activities Forward and Backward Passes with AOA Networks AOA VERSUS AON 10.5 CONTROVERSIES IN THE USE OF NETWORKS Conclusions Summary Key Terms Solved Problems Discussion Questions Problems Case Study 10.1 Project Scheduling at Blanque Cheque Construction (A) Case Study 10.2 Project Scheduling at Blanque Cheque Construction (B) MS Project Exercises PMP Certification Sample Questions Integrated Project—Developing the Project Schedule Notes

193 Copyright 2019 Pearson Education, Inc.


TRANSPARENCIES 10.1 NETWORK INCOPORATING FINISH TO START LAG OF 4 DAYS

0

A 6 Spec Design

6

6

B 11 Design Check

Lag 4

15

5

C 22 Blueprinting 7

10.2 FINISH TO FINISH NETWORK RELATIONSHIP

30

R

36 Wiring

6 31

S 33 Plumbing 2

33

T

3

36 HVAC

36

U 42 Interior Const. 6

194 Copyright 2019 Pearson Education, Inc.


10.3 START TO START NETWORK RELATIONSHIP

30

R

36 Wiring

6 33

T

3 days 3 31

36 HVAC

36 U 42 Interior Construction 6

S 33 Plumbing 2

195 Copyright 2019 Pearson Education, Inc.


10.4 START TO FINISH NETWORK RELATIONSHIP

20

W

26

6 3 days

20

Y

3 18

X

23

23

Z

29

6

20

2

196 Copyright 2019 Pearson Education, Inc.


10.5

BENEFITS OF GANTT CHARTS

1) They are very easy to read and comprehend. 2) They identify the project network coupled with its schedule baseline. 3) They allow for updating and project control. 4) They are useful for identifying resource needs and assigning resources to tasks. 5) They are easy to create.

197 Copyright 2019 Pearson Education, Inc.


10.6 SAMPLE GANTT CHART

Microsoft product screenshot(s) reprinted with permission from Microsoft Corporation.

198 Copyright 2019 Pearson Education, Inc.


10.7 SAMPLE TRACKING GANTT CHART WITH CRITICAL PATH A-C-D-F-H

Microsoft product screenshot(s) reprinted with permission from Microsoft Corporation.

199 Copyright 2019 Pearson Education, Inc.


10.8 GANTT CHART WITH RESOURCES IDENTIFIED FOR EACH TASK

Microsoft product screenshot(s) reprinted with permission from Microsoft Corporation.

200 Copyright 2019 Pearson Education, Inc.


10.9 CRASHING PROJECTS—UNDER WHAT CONDITIONS? 1. The initial schedule may be too aggressive. 2. Market needs change and the project is in demand earlier than anticipated. 3. The project has slipped considerably behind schedule. 4. The contractual situation provides even more incentive to avoid schedule slippage.

10.10 Options for Accelerating Projects 1. Improving the productivity of existing project resources a) Changing the working method employed for the activity b) Compromise Quality and/or Reduce Project Scope c) Fast-Tracking by: i. Shorten the longest critical activities ii. Partially overlap activities iii. Employ start-to-start lag relationships

d) Overtime

e) Adding Resources to the Project Team

201 Copyright 2019 Pearson Education, Inc.


10.11 TIME-COST TRADEOFFS FOR CRASHING ACTIVITIES

Crash Point Crashed

Cost Normal Point Normal

Crashed

Normal

Activity Duration

202 Copyright 2019 Pearson Education, Inc.


10.12 CRASHING ACTIVITIES—AN EXAMPLE

Formula

Slope =

crash cost − normal cost normal time − crash time

Example —Calculating the Cost of Crashing SUPPOSE: NORMAL ACTIVITY DURATION = 8 WEEKS NORMAL COST = $14,000 CRASHED ACTIVITY DURATION = 5 WEEKS CRASHED COST = $23,000 THE ACTIVITY COST SLOPE = 23,000 – 14,000 8–5

or,

$9,000

= $3,000 per week 3

203 Copyright 2019 Pearson Education, Inc.


10.13 SAMPLE AOA NETWORK DIAGRAM

3 B

1

A

2 C

4

204 Copyright 2019 Pearson Education, Inc.


10.14 PARTIAL AOA NETWORK ILLUSTRATING DUMMY ACTIVITIES

3 B A 1

2 C

4

205 Copyright 2019 Pearson Education, Inc.


DISCUSSION QUESTIONS

10.1

Give examples of circumstances in which a project would employ lag relationships between activities using: a. Finish to Start b. Finish to Finish c. Start to Start d. Start to Finish

a. Finish to Start: In home construction, plaster is applied to walls (activity A) before they are painted (activity B). The plaster has to dry before the walls can be painted. This creates a lag of one day between the finish of activity A and the start of activity B. b. Finish to Finish: A contractor may want the gas line and plumbing to be completed at the same time so that appliances can be installed in a kitchen. In this case, he/she may delay installation of a gas line (activity A) so that it will be completed at the same time as the plumbing (activity B). This creates a lag prior to the start of activity A so that A and B will finish on the same day. c. Start to Start: A contractor may elicit RFQs from subcontractors (activity B) while blueprints are still being fine tuned (activity A). This creates a lag between the start of activity A and the start of activity B. d. Start to Finish: Workers can begin putting up shower molding (activity A), but the work cannot be finished until installation of the shower head and faucet begins (activity B).

10.2

The advantage of Gantt charts lies in their linkage to the project schedule baseline. Explain this concept.

Because they are tied to the project schedule, Gantt charts allow project teams to track a project’s actual progress with the project’s planned progression. They create an easy reference for project teams that alert members to variance from the schedule baseline and the impact of the variance on the project network. Delays in one activity may create delays in subsequent activities. Here, Gantt charts can be used to update the baseline. Their depiction of variances from the baseline makes Gantt charts a useful project control tool by allowing the PM to identify problems that may jeopardize the team’s ability to meet project milestones.

10.3

What are the advantages in the use of Gantt charts over PERT diagrams? In what ways might PERT diagrams be advantageous? 206

Copyright 2019 Pearson Education, Inc.


Gantt charts display the project baseline. Each activity can then easily be referenced at any point during the project to see if progress is on target. Gantt charts are also more useful because the use of resources can be added to the charts. Finally, Gantt Charts are visually appealing and easily understandable. PERT diagrams offer a means to convey complex network relationships more completely and make the identification of predecessor and successor relationships easy to recognize.

10.4

How do concepts such as Brook’s Law and the effects of sustained overtime cause us to rethink the best ways to accelerate a project? Is it particularly ironic that these “acceleration” efforts can actually lead to serious delays?

Adding resources to a project in order to speed up activity completion is a natural response to the desire for project acceleration. However, Brook’s Law suggests that this response is not always the best one because it does not take into consideration learning curve effects, as new project team resources struggle to get caught up with the current team members. These effects can be somewhat dampened, provided enough time is allowed for new resources to learn their duties, or if they are assigned to tasks further downstream in the schedule. Likewise, a common approach to project acceleration is to require overtime work from project staff. Research, however, suggests that it is important not to rely too heavily on overtime, as the more hours of sustained overtime required per week of resources, the lower their overall productivity, due to fatigue and fatigue-induced errors requiring rework. The irony is that some of the most widely-used methods to accelerate a project are actually common causes of project delays.

10.5

Under what circumstances might you wish to crash a project?

Crashing may be desirable if initial time estimates are inaccurate, the project falls behind schedule or the project completion date is moved up. In these cases, the project cannot be completed on time given the original schedule. Crashing would expedite the project/activities to meet new completion dates. Another scenario occurs when late fees or penalties may be incurred. It may be more cost effective to crash activities than to pay the additional fees.

10.6

In crashing a project, we routinely focus on those activities that lie on the critical path, not activities with slack time. Explain why this is the case.

The critical path is the longest path through the network. Crashing activities on the critical path reduces the overall project duration time. Crashing activities outside of the CP (those with slack time) will increase overall costs, but only reduce the time to complete that single activity, not the overall time of the project. Therefore, it is more efficient, when trying to expedite the entire project, to crash activities that lie on the CP. 207 Copyright 2019 Pearson Education, Inc.


10.7

What are some of the advantages in the use of AOA notation as opposed to AON? Under what circumstances does it seem better to apply AON methodology in network development?

Advantages of the AOA notation include its prevalence in some specialized business fields, its clearer depiction of complex projects and event nodes/milestones in AOA are easier to identify. The AON methodology is more suitable when there are several merge points within the network. Merges make AOA notation convoluted (requiring the use of dummy activities) due to tasks being listed on arrows connecting activities in the diagram. AON is also more applicable when computer scheduling is desired because AON notation is used in most PM computer software packages.

10.8

Explain the concept of a “dummy variable.” Why is this concept employed in AOA notation? Why is there no need to use dummy variables in an AON network?

Dummy variables are arrows used in AOA notation to establish precedent relationships between activities. There are no durations or work description assigned to the arrows; they are simply used to indicate relationships. AOA requires the use of dummy variables because arrows cannot be used to connect two predecessors to one successor activity. Meaning if A and B are predecessors to C, there is no way, without dummy variables, to connect the dual predecessors to C. AON does not use dummy variables because arrows are used to indicate relationships between activities. If multiple predecessors exist, then multiple lines will be used to connect A and B (or as many activities as needed) to C.

10.9

Identify and discuss some of the problems or dangers in using project networks. Under what circumstances can they be beneficial, and when can they be dangerous?

One problem is that complex project may make networks meaningless. Project may be stretched out over several years making a traditional network diagram to complex to use. There is also the danger that information in the network may be incorrect or oversimplified leading to errors in its use downstream. Networks may also be misapplied. Companies may try to employ networks where they are unsuitable (i.e. scheduling problems outside of project management). Caution must be taken when using networks in a situation where multiple subcontractors are employed. Subcontractors need to be informed of other scheduling (i.e., other subcontractors) taking place in the project. Otherwise, independent networks for each subcontractor will exist and these networks may conflict with one another. Lastly, optimism in time estimates may create faulty network construction. Though there are several potential dangers inherent in project networks, they can be extremely beneficial to project teams. Networks visually depict what needs to be done, when it needs to be started and completed and how one activity affects other activities within the project. This visual map can be very helpful in making sense of complex project plans. 208 Copyright 2019 Pearson Education, Inc.


CASE STUDIES Case Study 10.1—Project Scheduling at Blanque Cheque Construction (A) The general nature of the project, developing a strip mall with four stores of roughly equal size, is intended to challenge students to identify a variety of activities necessary to accomplish the project. The first step is to create a simple WBS of the various project activities and deliverables that will be required. Instructors can decide in advance how much detail they want students to go into for this assignment; for example, they may wish to leave all elements at the Deliverable level and not get down to specific Work Packages. Questions 1. Develop a project network consisting of at least 20 steps that should be done to complete the project. As the case suggests, keep the level of detail for these activities general, rather than specific. Be sure to indicate some degree of precedence relationship among the activities. As with the above explanation, the key here is to have students first identify some of the activities (or Deliverables) for this construction project. The instructor may wish to work with the students, offering some prompts in the cases where people are unsure what actually goes into a mall development. After 20 steps are identified, challenge the students to create a simple activity network with precedence ordering of the steps. This step in the activity works well as a group exercise where students can bounce their ideas off each other and challenge basic assumptions about precedence ordering. 2. Suppose you now wanted to calculate duration estimates for these activities. How would you make use of the following approaches? Are some more useful than others? i. Expert opinion ii. Past history iii. Mathematical derivation In answering this question, students can first be assigned to create a duration estimation table using Beta distributions. Brainstorming will allow them to identify reasonable worst, most likely, and best-case scenarios for each activity duration. Then, they can answer the question regarding the use of expert opinion, past history, or mathematical derivation by showing how each could be applied to duration estimation. Instructors can challenge each of these points, arguing that expert opinion is just that, the experience of experts, not us. Likewise, they can point out that historical estimates may work or intervening issues may have come up that make historical parameters unduly pessimistic or optimistic (e.g., the loss of regional suppliers makes it harder and lengthens the time to identify and contract for project logistics).

209 Copyright 2019 Pearson Education, Inc.


3. Joe is trying to decide which scheduling format to employ for his planning: AON or AOA. What are some of the issues that Joe should first consider prior to choosing between these methods? The chapter identifies a number of issues that serve to help determine whether to use AON or AOA notation. Specifically: The benefits of AON are centered primarily in the fact that it has becomes the most popular format for computer software packages, such as MS Project. Hence, as more and more companies use software-based project scheduling software, they are increasingly using the AON method for network diagrams. The other benefits of AON are to place the activity within a node and use arrows merely as connection devices, thereby simplifying the network labeling. This convention makes AON networks very easy to read and comprehend, even for novice project managers. The primary drawback with AON networks occurs when the project is very complex, with numerous paths through the model. The sheer number of arrows and node connections when multiple project activities are merging or bursting can make AON networks difficult to read. On the other hand, AOA modeling’s greatest benefit lies in its accepted use in certain business fields, such as construction, where AON networks have not yet made significant inroads. Also, in the cases of large, complex projects it is often easier to employ the path process used in AOA. Finally, because the activity and node system is used, for projects that have many significant milestones, such as supplier deliveries, AOA event nodes are very easy to identify and flag. On the other hand, there is no question that some conventions in AOA diagramming are awkward; most particularly, the use of dummy activities. Dummy activities are not a simple concept to master and require more training on the part of novice project managers to be able to use them easily. Finally, AOA networks can be “information intensive” in that both arrows and nodes contain some important project information. Rather than centralizing all data into a node, as in the AON convention, AOA networks use both arrow and nodes to label the network.

Case Study 10.2—Project Scheduling at Blanque Cheque Construction (B) This case identifies a situation in which the project manager is facing a late project with serious consequences as a result. He is trying to decide whether or not to crash activities from this point to the end of the project and the focus is intended to make students understand the benefits and drawbacks of crashing. Because it should not be done without serious consideration of the escalating costs, crashing a project typically comes down to a benefits and drawbacks analysis, such as is the case here. Questions 1. What are some of the issues that weigh in favor of and against crashing the project?

210 Copyright 2019 Pearson Education, Inc.


Crashing must be considered in light of a number of factors, principally the penalties that the contractor will be forced to absorb should the project be late. In this case, the decision is based on the fact that top management has informed the project manager that he has some discretionary money to spend but the project cannot be late, suggesting that crashing makes good sense.

2. Suppose you were the site supervisor for this project. How would you advise Joe to proceed? Before deciding whether or not to crash the project, what questions should you consider and how should you evaluate your options? Among the important questions to ask is the critical one: will crashing the activities get us back on track? There is no point in crashing noncritical activities, nor should it be done if the net effect is only to spend money but not to complete the project within the necessary time period. Thus, the “dollar-day” choices have to be clear enough to warrant the decision to crash remaining activities. In this project case, most students will recommend crashing the activities as it is a relatively cost-free exercise for the project manager (i.e., he has discretionary money and has calculated that crashing remaining activities will bring them close to the project deadline for completion). Instructors can use this scenario with real data to demonstrate how to prioritize activities for crashing.

PROBLEMS 10.10 Develop a Gantt chart with the following information. What is the expected duration of the project? What is the critical path? Activity A B C D E F

Expected Duration 12 8 5 10 10 5

Predecessors None None None A and B C and D A and B

SOLUTION:

211 Copyright 2019 Pearson Education, Inc.


The expected duration of the project is determined by the linking of the longest (critical) path through the network (A – D – E) for an expected duration of 28 days. The critical path can also be found by highlighting “Tracking Gantt” in the MS Project package.

10.11

Given the following information, answer the questions about this project: Activity A B C D E F G H I

Expected Duration 4 days 9 days 11 days 5 days 3 days 7 days 3 days 2 days 1 day

Predecessors — A A B B C D, F E, G H

a. Draw the network as a Gantt chart. b. What is the critical path? Which activities have slack time? c. What would happen if activities B and D each took 5 extra days to complete instead of the expected duration? How would the critical path change? SOLUTION: a. Draw the network as a Gantt chart.

b. What is the critical path? Which activities have slack time? The critical path is A – C – F – G – H – I for a total expected duration of 28 days. Activity B has 4 days of slack, Activity D has 4 days of slack, and Activity E has 9 days of slack. 212 Copyright 2019 Pearson Education, Inc.


c. What would happen if activities B and D each took 5 extra days to complete instead of the expected duration? How would the critical path change? The new critical path would be A – B – D – G – H – I for a total expected duration of 34 days.

10.12 Develop the network activity chart and identify the critical path for a project based on the following information. Redraw the activity network as a Gantt chart. What is the expected duration of the project?

Activity A B C D E F G H I J

Expected Duration 5 days 6 days 2 days 4 days 6 days 6 days 12 days 4 days 6 days 7 days

Predecessors — A A A B, C D, E F G F H, I

SOLUTION:

213 Copyright 2019 Pearson Education, Inc.


Microsoft product screenshot(s) reprinted with permission from Microsoft Corporation.

Expected duration of the project can be found by adding the lengths of the longest linked paths. In this case, it is 46 days.

214 Copyright 2019 Pearson Education, Inc.


10.13 Develop a Gantt chart for the following activities. Identify all paths through the network. What is the critical path? Optional: Solve this problem with Microsoft Project. How does clicking on “Tracking Gantt” view demonstrate the critical path?

Activity A B C D E F G

Expected Duration 2 days 3 days 4 days 4 days 5 days 6 days 4 days

Predecessors — A A B, C B D C, E, F

SOLUTION:

The paths through the network include: (1) A – B – E – F – G – H – J; (2) A – C – E – F – G – H – J; (3) A – B – E – F – I – J; (4) A – C – E – F – I – J; (5) A – D – F – G – H – J; (6) A – D – F – I – J Critical Path: A – B – E – F – G – H – J Using “Tracking Gantt” option, we can recreate the network to show the critical path, highlighted in red. 10.14 Reconfigure the Gantt chart in Problem 10.13 to include some different predecessor relationships. Optional: Solve this problem with Microsoft Project. a. Assume that activities B and C are linked with a “Finish-to-Finish” relationship. Does that change the expected completion date for the project? b. For activity F, add a lag of 3 days to its predecessor relationship with activity D. By adding the 3-day lag to F, what is the new expected duration for the project? 215 Copyright 2019 Pearson Education, Inc.


c. Suppose you now added a start-to-start relationship between activities F and G to the new Gantt chart. How does this additional relationship change the expected completion date for the project?

SOLUTION: a. No, linking B and C in a Finish-to-Finish relationship will not affect the critical path because activity C is not on the critical path, having 4 days of slack time. b. No change, because D had slack time in its schedule (i.e., was not on the critical path so the 3-day lag did not affect overall completion of the project). c. It would save one week on the overall project completion date by altering the task relationship to start-to-start. It is important to determine whether such a relationship is possible, given the nature of the tasks to be performed.

10.15 Consider a project with the following information. Construct the project activity network using AOA methodology and label each node and arrow appropriately. Identify all dummy activities required to complete the network.

Activity A B C D E F G H

Duration 3 5 7 3 5 4 2 5

Predecessors -A A B, C B D C E, F, G

216 Copyright 2019 Pearson Education, Inc.


Activity A B C D E F G H

Duration 3 5 7 3 5 4 2 5

ES 0 3 3 10 8 13 10 17

EF 3 8 10 13 13 17 12 22

LS 0 5 3 10 12 13 15 17

LF 3 10 10 13 17 17 17 22

Slack — 2 — — 4 — 5 —

217 Copyright 2019 Pearson Education, Inc.


SOLUTION:

218 Copyright 2019 Pearson Education, Inc.


10.16 You are considering the decision of whether or not to crash your project. After asking your operations manager to conduct an analysis, you have determined the “precrash” and “postcrash” activity durations and costs, shown in the table below (assume all activities are on the critical path):

Activity

Normal Duration Cost

Crashed Duration Cost

A

4 days

$1,000

3 days

$2,000

B

5 days

2,500

3 days

5,000

C

3 days

750

2 days

1,200

D

7 days

3,500

5 days

5,000

E

2 days

500

1 day

2,000

F

5 days

2,000

4 days

3,000

G

9 days

4,500

7 days

6,300

a. Calculate the per day costs for crashing each activity b. Which are the most attractive candidates for crashing? Why?

219 Copyright 2019 Pearson Education, Inc.


SOLUTION: a. Applying the formula for crashing, the per day costs for crashing each activity is: Activity Per Day Cost A $1,000 B $1,250 C $450 D $750 E $1,500 F $1,000 G $900 b. The most attractive candidates for crashing (assuming we know nothing of critical activities) are those with the lowest per day cost to crash. In this case, we would crash the activities in order: C, D, G, A and F, B, E. 10.17 Suppose you are considering crashing a project. The project’s network is as follows, along with a table identifying its critical activities and the crash costs for all tasks. a. What is the cost of the project? b. Which activities are the best candidates for crashing? c. What is the expected duration of the project once it has been fully crashed? d. What will the cost of the fully crashed project be?

220 Copyright 2019 Pearson Education, Inc.


Project Activities and Costs (Normal vs. Crashed) Normal Activity Duration Cost (in U.S. dollars) A 4 days 1,000 B 6 days 1,800 C 4 days 2,500 D 9 days 2,700 E 8 days 2,400 F 5 days 3,500 G 2 days 2,400 Total costs $16,300

Activity A B C D E F G

Crashed Duration Cost (in U.S. dollars) 3 days 2,000 3 days 3,000 3 days 4,000 6 days 6,500 6 days 4,500 3 days 7,000 1 day 3,000 $30,000

Crashing Costs Crashing Costs per Day ($) 1,000 400 1,500 1,267 1,050 1,750 600

On Critical Path? Yes Yes No Yes No Yes Yes

SOLUTION: a. What is the cost of the project? We know from the cost table that the normal cost (noncrashed) cost for the project is $16,300. b. Which activities are the best candidates for crashing? When identifying best candidates for crashing, we first have to identify the critical path because crashing activities on the critical path will lead to shorter project duration. Based on the table, we can identify activities A, B, D, F, and G as critical. In order of attractiveness (lowest crashing cost per day), we would prioritize (in order): Activity B, Activity G, Activity A, Activity D, and Activity F. c. What is the expected duration of the project once it has been fully crashed? The fully crashed project will have an expected duration of 16 days. d. What will the cost of the fully crashed project be? Remember that we do not need to crash non-critical activities, so it would be a mistake to simply select $30,000 as the cost value. Crashing (in order) Activities B, G, A, D, and F will add: Crash Activity Days Crashed Cost per day Total Cost Activity B 3 days $400/day $1,200 Activity G 1 day $600/day 600 221 Copyright 2019 Pearson Education, Inc.


Activity A 1 day Activity D 3 days Activity F 2 days Total costs of crashing =

$1,000/day $1,267/day $1,750/day

1,000 3,800 3,500 $10,100

Therefore, the total cost of the project = $16,300 + $10,100 = $27,400

10.18 Suppose you were trying to decide whether or not it made sense to crash your project. You know that normal project duration and direct costs are 60 days and $125,000. You are worried because you have a very tight delivery schedule and the customer has placed a severe penalty into the contract in the form of $5,000 in liquidated damages for every day the project is late after 50 days. After working with the cost accountant, you have generated the following table of project costs at different completion durations: Project Duration (in days) 60 57 54 51

Direct Costs

$125,000 140,000 175,000 210,000

Overhead Costs $15,500 13,000 10,500 8,000

Penalty Charges $50,000 35,000 20,000 5,000

Total Costs

190,500 188,000 205,500 223,000

d. Complete the table. How many days would you advise the project should be crashed? Why? e. Suppose direct costs of crashing the project only increased $5,000 per day crashed at a steady rate (starting with $125,000 on day 60). How many days would you advise that the project be crashed? Show your work.

SOLUTION: a. Crashing the project 3 days to a total duration of 57 days is the most cost effective option among the choices because it minimizes total costs ($188,000).

b. If direct costs due to crashing only increase at a steady rate of $5,000 per day, the best choice would be to crash the full 51 days for a total cost of $183,000. This would minimize overhead and penalty charges and minimize total costs overall. 10.19 When deciding on whether or not to crash project activities, a project manager was faced with the following information. Activities of the critical path are highlighted with an asterisk:

222 Copyright 2019 Pearson Education, Inc.


Activity A B* C D* E* F G* H

a.

b. c.

d.

e.

Normal Cost Duration $5,000 4 weeks 10,000 5 weeks 3,500 2 weeks 4,500 6 weeks 1,500 3 weeks 7,500 8 weeks 3,000 7 weeks 2,500 6 weeks

Crashed Extra Cost Duration $4,000 3 weeks 3,000 4 weeks 3,500 1 week 4,000 4 weeks 2,500 2 weeks 5,000 7 weeks 2,500 6 weeks 3,000 5 weeks

Identify the sequencing of the activities to be crashed in the first four steps. Which of the critical activities should be crashed first? Why? What is the project’s critical path? After four iterations involving crashing project activities, what has the critical path shrunk to? Suppose project overhead costs accrued at a fixed rate of $500 per week. Chart the decline in direct costs over the project life relative to the increase in overhead expenses. Assume that a project penalty clause kicks in after 19 weeks. The penalty charged is $5,000 per week after 19 weeks. When the penalty charges are added, what does the total project cost curve look like? Develop a table listing the costs accruing on a per-week basis. If there were no penalty payments accruing to the project, would it make sense to crash any project activities? Show your work.

SOLUTION: a) The correct sequence for crashing activities is listed as: 1. Activity E or G (they both cost $2,500 more) 2. Activity E or G 3. Activity B 4. Activity D Following this order assumes that activities are crashed in order of their cost. b) The project’s critical path is B – D – E – G, or 21 weeks. After four iterations of crashing project activities, the critical path baseline has shrunk to 16 weeks. c) The chart showing the decline in costs relative to the increase in overhead expenses would resemble the following:

223 Copyright 2019 Pearson Education, Inc.


Duration 21 weeks 20 weeks 19 weeks 18 weeks 16 weeks

Direct Costs 37,500 40,000 42,500 45,500 49,500

Penalties 10,000 5,000 -0-0-0-

Overhead 10,500 10,000 9,500 9,000 8,000

Total 58,000 55,000 52,000 54,000 56,500

60

50

Total costs

40

Direct costs

Cost (thousands) 30

20 Overhead Penalty

10 10

12 14 16 18 Schedule Baseline (Weeks)

20

22

d) Also shown on the figure above. e) If no penalties kicked in, there would not be a cost basis for crashing the project. It may still be appropriate given other project information; however, direct costs decline proportionally to increase in overhead expenses, suggesting no reason to crash the project from a cost perspective.

224 Copyright 2019 Pearson Education, Inc.


MS PROJECT EXERCISES Exercise 10.19 Suppose we have a complete activity predecessor table (shown below) and we wish to create a Network diagram highlighting the activity sequence for this project. Using MS Project, enter the activities A through E, their durations, and their predecessors. Note that all duration times are in days. Project:Remodeling an Appliance Activity A. Conduct competitive analysis B. Review field sales reports C. Conduct tech capabilities assessment D. Develop focus group data E. Conduct telephone surveys F. Identify relevant specification improvements G. Interface with Marketing staff H. Develop engineering specifications I. Check and debug designs J. Develop testing protocol K. Identify critical performance levels L. Assess and modify product components M. Conduct capabilities assessment N. Identify selection criteria O. Develop RFQ P. Develop production master schedule Q. Liaison with Sales staff R. Prepare product launch

Duration 3 2 5 2 3 3 1 5 4 3 2 6 12 3 4 5 1 3

Predecessors — — — A, B, C D E F G H G J I, K L M M N, O P Q

Solution: Entering the five Activities and their predecessors, the partial Gantt chart should look like the following:

225 Copyright 2019 Pearson Education, Inc.


Microsoft product screenshot(s) reprinted with permission from Microsoft Corporation.

Exercise 10.20

Now, continue developing your Gantt chart with the rest of the information contained in the table above and create a complete activity Network diagram for this project. Solution:

Microsoft product screenshot(s) reprinted with permission from Microsoft Corporation.

226 Copyright 2019 Pearson Education, Inc.


Exercise 10.21 Identify the critical path for this project. How can you tell? (Hint: Click on the “Tracking Gantt” option.) Solution: The tracking Gantt chart automatically highlights (in red) the critical activities within the network. Thus, following the MS Project output below, we can determine that the critical path for this project is the path C – D – E – F – G – H – I – L – M – O – P – Q – R

Microsoft product screenshot(s) reprinted with permission from Microsoft Corporation.

Exercise 10.22 Suppose that we wished to incorporate lag relationships into our activity network. Consider the table shown below and the lag relationships noted. Develop an MS Project Gantt chart that demonstrates these lags. Activity A. Wiring B. Plumbing C. HVAC D. Interior construction

Duration 6 2 3 6

Predecessor Relationship None None Wiring (Finish-to-Start), Plumbing (Finish-to-Finish) HVAC (Start-to-Start)

Solution:

227 Copyright 2019 Pearson Education, Inc.


Microsoft product screenshot(s) reprinted with permission from Microsoft Corporation.

228 Copyright 2019 Pearson Education, Inc.


CHAPTER EIGHT

Cost Estimation and Budgeting Chapter Outline PROJECT PROFILE New York City’s Second Avenue Subway – Two Miles Completed for Only $5 Billion Spent 8.1 COST MANAGEMENT Direct versus Indirect Costs Recurring versus Nonrecurring Costs Fixed versus Variable Costs Normal versus Expedited Costs 8.2 COST ESTIMATION Learning Curves in Cost Estimation Software Project Estimation—Function Points PROJECT MANAGEMENT RESEARCH IN BRIEF Software Cost Estimation Problems with Cost Estimation PROJECT MANAGEMENT RESEARCH IN BRIEF “Delusion and Deception” Taking Place in Large Infrastructure Projects 8.3 CREATING A PROJECT BUDGET Top-Down Budgeting Bottom-Up Budgeting Activity-Based Costing 8.4 DEVELOPING BUDGET CONTINGENCIES Summary Key Terms Solved Problems Discussion Questions Problems Case Study 8.1 The Hidden Costs of Infrastructure Projects—The Case of Building Dams Case Study 8.2 Sochi Olympics—What’s the Cost of National Prestige? Internet Exercises PMP Certification Sample Questions Answers Integrated Project—Developing the Cost Estimates and Budget Notes

128 Copyright © 2019 Pearson Education, Inc.


TRANSPARENCIES Note for Instructors: To present transparencies in class, please download the PowerPoint Presentations (available on www.pearsonhighered.com) that accompanies this product. The PPT ISBN is 9780134730479.

8.1 SOURCES OF PROJECT COSTS 1. Direct versus Indirect Costs 2. Recurring versus Non-recurring Costs 3. Fixed versus Variable Costs 4. Normal versus Expedited Costs

8.2 PROJECT PRICE BREAKDOWN

129 Copyright © 2019 Pearson Education, Inc.


8.3 COST ESTIMATION

1. Ballpark Estimates (Order of Magnitude) a. ± 30% 2. Comparative Estimates a. Parametric Estimates b. Analogous Estimates c. ± 15% 3. Feasibility Estimates a. ± 10% 4. Definitive Estimates a. ± 5%

8.4 COST ESTIMATOR FOR CONSTRUCTION (RS MEANS)

130 Copyright © 2019 Pearson Education, Inc.


8.5 LEARNING CURVE MODEL

131 Copyright © 2019 Pearson Education, Inc.


8.6 FUNCTION POINT ESTIMATION Function points—standard units of measure that represent the functional size of a software application Based on: 1. Number of inputs 2. Number of outputs 3. Number of interfaces 4. Number of queries 5. Number of files

8.7 PROBLEMS WITH COST ESTIMATION 1. Low Initial Estimates 2. Unexpected Technical Difficulties 3. Lack of Definition 4. Specification Changes 5. External Factors

132 Copyright © 2019 Pearson Education, Inc.


8.8 EXAMPLE OF A TIME-PHASED BUDGET

Months

Activity

January

Survey

4,000

February

March

April

May

Total by Activity 4,000

Design

5,000

Clear Site

4,000

Foundation

3,000

8,000 4,000

7,500

7,500

Framing

8,000

2,000

10,000

Plumb and Wire

1,000

4,000

5,000

Monthly Planned

4,000

9,000

10,500

9,000

6,000

Cumulative

4,000

13,000

23,500

32,500

38,500

38,500

133 Copyright © 2019 Pearson Education, Inc.


8.9 CALCULATING DIRECT LABOR COSTS

Name

Hours Needed

Overhead Charge

Personal Time Rate

Hourly Rate

Total Direct Labor Cost

John

40

1.80

1.12

$21/hr.

$1,693.44

Bill

40

1.80

1.12

$40/hr.

3,225.60

J.P.

60

1.35

1.05

$10/hr.

850.50

Sonny

25

1.80

1.12

$32/hr.

1,612.80

Total Direct Labor Cost = $7,382.34

134 Copyright © 2019 Pearson Education, Inc.


DISCUSSION QUESTIONS 8.1 Describe an environment in which it would be common to bid for contracts with low profit margins. What does this environment suggest about the competition levels? Low barriers to entry, a lack of economies of scale, easier access to technology and, oftentimes, powerful buyers commonly characterize an environment with very low profit margins. The relative ease of competing in such industries increases competition levels both domestically and internationally. 8.2 How has the global economy affected the importance of cost estimation and cost control for many project organizations? Globalization of the economy has resulted in lower barriers to trade, market-driven economies, deregulation, and privatization. These effects have created greater competition and larger markets. With the ease of competing internationally, companies need to be sure to provide accurate, competitive bids. Bids need to be low in order to be competitive in the heightened competition of the global economy. Secondly, accuracy takes on new complications when considering bids abroad. Exchange rates, transportation and trade costs all need to be accounted for in the bid. These added costs make cost control (in other controllable areas of the project) vital. Without tight cost control, the company will not be able to compete with bids of domestic firms. 8.3 Why is cost estimation such an important component of project planning? Discuss how it links together with the Work Breakdown Structure and the project schedule. Cost estimation, if done correctly, enables a firm to determine if the project will be profitable, if the company can afford the project and in general if the project is worth pursuing. It also provides the company with a cost range for bidding (in the case of a customer-oriented project). With respect to Work Breakdown Structure and the project schedule, cost estimation is important because it leads to budgeting of monetary and other resources (both material and human). These allocations must coordinate with the Work Breakdown Structure and the project schedules prepared by management to figure out if the required resources will be available as needed. 8.4 Imagine you are developing a software package for your company’s intranet. Give examples of the various types of costs (labor, materials, equipment and facilities, subcontractors, etc.) and how they would apply to your project. Potential costs of creating software package include costs of labor, materials, subcontractors, equipment, and facilities and travel. Software engineers, developers, computer technicians, trainers (for end users) and technical writers would incur labor costs. Material costs may come from printing and creating installation CDs/disks, additional mainframe hardware, memory or accessories, and any printing and paper 135 Copyright © 2019 Pearson Education, Inc.


requirements for user manuals. Developers and contractors may require extra space or equipment. Subcontractors may be used to consult on design and implementation. Subcontractors may require costs associated with travel if the firm is not local. 8.5 Give reasons both in favor of and against the use of personal time charge as a cost estimate for a project activity. Using a personal time charge can create a more accurate assessment of time by including a reasonable amount of downtime in estimates of work time. By using the personal time charge, a company can be better compensate for its labor resources, as all time (productive or not) spent on a particular job is a use of human/intellectual resources. However, from a customer’s perspective this charge may appear unwarranted. The personal time charge allows time for unproductive breaks. Customers will most likely be reluctant to pay for unproductive time—resulting in payment disputes. 8.6 Think of an example of parametric estimating in your personal experience, such as the use of a cost multiplier based on a similar, past cost. Did parametric estimating work or not? Discuss the reasons why. This is a personal example question and should only be applied to students with some project experience. 8.7 Suppose your organization used function point analysis to estimate costs for software projects. How would the expertise level of a recently hired programmer affect your calculation of their function points on a monthly basis when compared to an older, more experienced programmer? Function point analysis assumes that there are differential challenges with software functions depending upon the resource. A newly hired programmer might rate more of the functions to which he is assigned as “Higher Complexity,” which would add extra hours to the time (and therefore, cost) estimates. Complexity estimates determine the costs of variance program features, so the more seasoned and expert the programmer, the lower the overall cost of the sequence. 8.8 Put yourself in the position of a project customer. Would you insist on the cost adjustments associated with learning curve effects or not? Under what circumstances would learning curve costs be appropriately budgeted into a project? As a customer, I would not accept fees when the repetitive work (that accounts for the learning curve) is a routine job for the supplier. The reason for this is that I would be paying for learning effects that others would reap the benefit from. Also, learning effects associated with new employees would be unreasonable to include in project billing. On the other hand, if the repetitive work/learning curve effects were project or customer specific, then budgeting the costs into the project would be appropriate. 136 Copyright © 2019 Pearson Education, Inc.


8.9 Consider the common problems with project cost estimation and recall a project with which you have been involved. Which of these common problems did you encounter most often? Why? This is a personal example question and should only be applied to students with some project experience. 8.10 Would you prefer the use of bottom-up or top-down budgeting for project cost control? What are the advantages and disadvantages associated with each approach? Top-down budgeting would be preferred because it, more often, results in better cost control. It also draws on experience (historical costs, etc.) of top management for reasonable project estimates. However, these advantages come at a cost. Top-down budgeting can result in friction between top, middle, and lower level managers as the approach results in a zero-sum game (what one manager receives another loses). Bottomup budgeting has the benefit of being detailed from the very beginning. Costs can be directly tied to WBS and individual tasks. Unfortunately, this process can be time consuming and removes control from top management, which may lead to variances from strategic goals of the project. 8.11 Why do project teams create time-phased budgets? What are their principal strengths? Time-phased budgets all teams to match the project schedule with the project’s budget. They can also identify milestones for completion of work and budget expenditures. This enables them to see when and where money was spent. This creates a better idea of where variances occurred and creates better control mechanisms. Moreover, time-phased budgets result in better documentation for future project planning. 8.12 Project contingency can be applied to projects for a variety of reasons. List three of the key reasons why a project organization should consider the application of budget contingency. First, changes in scope and specifications can occur creating increased costs. Secondly, rework and interaction fees can be difficult to predict in the initial planning stages. For example, project may require communication between departments or office branches. This may incur travel and coordination (shipping, etc) costs that are unseen prior to implementation. Lastly, uncertainty (especially related to the environment or politics) is a known complication of project work. Changes can occur that restrict the use or availability of resources. This may cause delays in production or create the need for additional accommodations.

137 Copyright © 2019 Pearson Education, Inc.


CASE STUDIES 8.1 The Hidden Costs of Infrastructure Projects—The Case of Building Dams A fascinating study that was recently completed by a research group at Oxford University (headed by Professor Bent Flyvbjerg) found that many developing countries engage in large infrastructure projects under the assumption that these are the key to future economic success. A particular case in point is building large-scale dams. Demand for electricity has gone up dramatically, particularly in third-world countries that are assumed could greatly benefit from hydropower through dams. Flood control, crop irrigation, transportation, and other reasons also support the idea that large dams are a universal benefit. In fact, research is now suggesting that the enormous costs of dam projects are such that countries simply cannot afford them. Close analysis of benefits and drawbacks shows that rarely do countries and contractors take a hard look at the “real” costs of large infrastructure projects, like dams, and if they did, they would be much more wary of engaging in these projects. The case concludes with some alternatives, such as the policy pursued in Norway, of smaller, more eco-friendly dam projects that can produce needed energy but are affordable and environmentally sound. Questions 1. Given the history of large cost overruns associated with megadam construction, why do you believe they are so popular, especially in the developing world? One obvious answer is the prestige they offer, especially in regard to presenting evidence of development and improvement to the national economy. Some developing countries pursue them as a sign of international legitimacy. There is also the darker reason that large projects offer the opportunity for large-scale graft and corruption, as has been rumored with the Sochi Olympics construction projects. 2. Develop an argument in support of megadam construction. Develop an argument against these development projects. This question works well for setting up an in-class debate between groups. The best approach is to first assign them to teams and have them research this question, so they come prepared with actual evidence, rather than simple opinion. There are multiple arguments on both sides of the debate and a thorough response should consider both the benefits and drawbacks of such massive construction projects, though the current longerterm evidence suggests that they are more a curse than a benefit. 8.2 Sochi Olympics—What’s the Cost of National Prestige? The Sochi Olympics was an example of project costs running out of control; so much so that the final price tag (estimated to be more than $50 billion), dwarfed the costs of every other Olympic Games to this point in time. In addition to picking a questionable, sub138 Copyright © 2019 Pearson Education, Inc.


tropical location for the Games, from the very beginning, the process of developing the site was subject to meddling from politicians, including President Vladimir Putin. Another major cost to the final price tag was related to charges of wide-spread corruption, as sub-projects for the Games (such as infrastructure, buildings, and roads) ended up with highly inflated price tags. This is a great case for general class discussion as we consider the purpose for Olympic Games, the challenges of cost control with critical deadlines that must be adhered to, and the fact that costs are rising for Games to the point where in 2017, Rome, Budapest, and several other cities withdrew their bids to host the 2024 Olympics because they could not trust the final costs and could not justify the value they would gain versus the costs of the project. Questions 1. Consider the following statement: “Government-funded projects intended to serve as ‘prestige projects,’ such as the Sochi Olympics, should not be judged on the basis of cost.” Do you agree or disagree with this statement? Why? This question asks us whether or not the standard rules of oversight apply to special projects, sometime termed “prestige projects,” such as Olympic Games (or the Channel Tunnel or Sydney Opera House). The issue is whether we willingly should ignore standard estimation and oversight procedures due to the special nature of these projects. As there is no obvious right or wrong answer, the question leads to good discussion of the benefits and drawbacks of these types of project.

2. Project success is defined as adherence to budget, schedule, functionality (performance), and client satisfaction. Under these criteria, cite evidence that suggests the Sochi Olympics project was a success and/or failure. Clearly, there is very little to recommend the Big Dig from the standard success criteria relating to cost. On the other hand, it could be argued that with a looming deadline and unmovable completion date (the starting date for the Olympics is established years in advance), one “card” that the Games developers have to play is their willingness to spend as much money as possible to get the venues ready on time. Because the Games were ready and went off without significant problems, one could argue that they were a success, in spite of a dreadful budget performance. 3. When a project has a “hard gate,” like being ready on time, how does that affect normal success criteria? Is it fair to judge a project with a critical completion date by normal project success standards? Why or why not? This question links to the one previously discussed. Clearly, hitting their schedule deadline was of paramount importance and all other success criteria had to be subordinated to that issue. A more interesting question is whether having a critical deadline and a willingness to crash activities (pouring extra money into them) leads to an 139 Copyright © 2019 Pearson Education, Inc.


atmosphere where it becomes easier for graft and corruption to flourish. In other words, when the project has to be completed on time, can anyone reasonably be expected to keep close track of how all the money was spent? 4. Consider the problems with the Rio Olympics sites that quickly occurred following completion of the 2016 Summer Games. Access the internet to find evidence of the current state of the Sochi Olympic site. How is it being used and what are the current problems and opportunities for Sochi? The Rio Olympics and their aftermath represent one of the worst problems that face countries hosting the Games; i.e., the rapid deterioration of the site and facilities. The argument that the Games can raise living standards for those living around them has been shown to be generally false, as witnessed by the current state of the Rio site. Students can access and compare and contrast the Sochi site with the obvious deterioration of the Rio facilities. PROBLEMS 8.13 Calculate the fully loaded cost of labor for a project team member using the following data: Hourly rate: $17.50/hr Hours needed: 200 Overhead rate: 35% Solution: The formula for calculating fully loaded cost of labor is: Hourly rate Hours needed ($17.50) × (200) ×

Overhead charge (1.35)

=

Total direct labor cost $4,725.00

8.14 Calculate the fully loaded cost of labor for a project team member using the following data: Hourly rate: $52/hr Hours needed: 120 Overhead rate: 60% Solution: The formula for calculating fully loaded cost of labor is: Hourly rate Hours needed ($52) × (120) ×

Overhead charge (1.60)

=

Total direct labor cost $9,984.00

8.15 Calculate the fully loaded cost of labor for your Project Engineer using the following data: Hourly rate: $40/hr Estimated hours of work: 120 140 Copyright © 2019 Pearson Education, Inc.


Overhead rate: Personal time:

65% 15%

Solution: The formula for calculating fully loaded labor costs is: Hourly rate Hours needed ($40) × (120)

Overhead charge × (1.65) ×

Personal time (1.15) =

Fully loaded labor cost $9,108.00

8.16 Calculate the fully loaded cost of labor for the project team using the following data. What are the costs for the individual project team members? What is the fully loaded cost of labor?

Solution: Name

Hours Needed

Overhead Charge

Sandy 60 1.35 Chuck 80 1.75 Bob 80 1.35 Penny 40 1.75 Fully Loaded Labor Cost = $9,818

Personal Time Rate

Hourly Rate

1.12 1.12 -01.12

$18/hr. $31/hr. $9/hr. $30/hr.

Fully Loaded Labor Cost $1,633 $4,861 $972 $2,352

8.17 Assume that overhead is charged on a flat rate basis. Each member of the project is assigned an overhead charge of $150/week. What would the fully loaded cost of labor be for an employee, given that she is assigned to the project for 200 hours at $10.50/hour? Solution: The hours that the employee works (200 hours) would equate to 5 weeks, thus the calculation would show: Hourly rate Hours needed Overhead charge Total direct labor cost ($10.50) × (200) + $750 = $2,850

141 Copyright © 2019 Pearson Education, Inc.


8.18 Calculate the fully loaded labor costs for members of your project team using the following data. Who is the most expensive member of your team? What proportion of the overall fully loaded cost of labor is taken up by this individual?

Name Todd Stan Mary Alice

Hours Overhead Personal Needed Charge Time Rate 200 1.55 1.15 200 1.75 -0150 1.55 -080 1.75 1.15 Total Fully Loaded Labor Cost 

Hourly Rate $32/hr $15/hr $24.5/hr $30/hr

Fully Loaded Labor Cost 11,408.00 5,250.00 5,696.25 4,830.00 $27,184.25

Solution: The fully loaded costs are embedded in the table in the right hand column. The most expensive resource for this project is Todd. His labor costs are approximately 42% of the total fully loaded labor cost for the project.

8.19 Using the following information about work package budgets, complete the overall time-phased budget for your project. (All cost figures are in $000s). Which are the weeks with the greatest budget expense? Task Budget Week 1 Week 2 Week 3 Week 4 Week 5 Week 6 Week 7 A 5 3 2 B 8 1 4 3 1 C 12 2 7 3 D 7 3 3 1 E 14 5 5 2 2 F 6 1 2 Plan 52 4 8 13 12 6 3 4 Cumulat 4 12 25 37 43 46 50 ive Solution: The plan and cumulative time-phased budget totals are shown in the bottom two rows of the table. The weeks with the biggest planned expense are Week 3 ($13,000) followed by Week 4 ($12,000). The total cumulative budget for the project is $53,000.

8.20 Given the following information, complete a time-phased budget for your project. (All cost figures are in $000s). What are weekly planned and cumulative costs for the project?

142 Copyright © 2019 Pearson Education, Inc.

Week 8

3 3 53


Work Package Cost per Week Work Package Staffing Blueprinting Prototyping Full Design Plan Cumulative

Budget

Week 1

Week 2

9 16 17 30

7 4

2 10 2

11 11

14 25

Week 3

Week 4

Week 5

2 10 4 16 41

4 12 16 57

1 14 15 72

Solution: The plan and cumulative time-phased budget totals are shown in the bottom two rows of the table. The total cumulative budget for the project is $72,000. For the Problems 8.21 through 8.23, refer to the following chart of learning curve coefficients (unit and total time multipliers). Learning Curve Coefficients (Unit Time and Total Time Multipliers) 70% 75% 80% Unit Unit Total Unit Total Unit Total Rate Time Time Time Time Time Time 5 .437 3.195 .513 3.459 .596 3.738 10 .306 4.932 .385 5.589 .477 6.315 15 .248 6.274 .325 7.319 .418 8.511 20 .214 7.407 .288 8.828 .381 10.485 25 .191 8.404 .263 10.191 .355 12.309 30 .174 9.305 .244 11.446 .335 14.020 35 .160 10.133 .229 12.618 .318 15.643 40 .150 10.902 .216 13.723 .305 17.193 Based on a = 1.

85% Unit Time .686 .583 530 .495 .470 .450 .434 .421

Total Time 4.031 7.116 9.861 12.402 14.801 17.091 19.294 21.425

8.21 It took MegaTech, Inc., 100,000 labor-hours to produce the first of several oil-drilling rigs for Antarctic exploration. Your company, Natural Resources, Inc., has agreed to purchase the fifth (steady-state) oil-drilling rig from their manufacturing yard. Assume that MegaTech experiences a learning rate of 80%. At a labor rate of $35 per hour, what should you, as the purchasing agent expect to pay for the fifth unit? Remember, the simplified formula for calculating learning rate time is given as: TN = T1C Where TN T1 C

= Time needed to produce the nth unit = Time needed to produce the first unit = Learning curve coefficient 143

Copyright © 2019 Pearson Education, Inc.


Solution: The table lists the fourth unit learning curve coefficient as .596, assuming a learning rate of 80%. Using the formula, to produce the fifth unit takes: TN = T1C Where TN T1 C T5

= Time needed to produce the nth unit = Time needed to produce the first unit = Learning curve coefficient

= (100,000) (.596) = 59,600 hours

To find the cost, multiply the hours by the hourly rate: = (59,600) ($35 per hour) = $2,086,000

8.22 Problem 8.21 identified how long it should take to complete the fifth oil drilling platform that Natural Resources plans to purchase. How long should all five oil-drilling rigs take to complete? Solution: We can look at the total time column in the table associated with a learning rate of 80%. The multiplier is listed as 3.738. Using this value, the total time necessary to complete the five rigs is calculated as: T5

= (100,000) (3.738) = 373,800 hours to complete all five oil-drilling rigs

8.23 Suppose that you are assigning costs to a major project to be undertaken this year by your firm, DynoSoft Applications. One coding process involves many labor-hours, but highly redundant work. You anticipate a total of 200,000 laborhours to complete the first iteration of the coding and a learning curve rate of 70%. You are attempting to estimate the cost of the twentieth (steady state) iteration of this coding sequence. Based on this information and a $60 per hour labor rate, what would you expect to budget as the cost of the twentieth iteration? The fortieth iteration? Solution: T20

= = =

T1C (200,000) (.214) 42,800 hours 144

Copyright © 2019 Pearson Education, Inc.


T40

= = =

T1C (200,000) (.150) 30,000 hours

The costs for the twentieth and fortieth iterations are found as: Twentieth iteration:

(42,800 hours) ($60 per hour) =

$2,568,000

Fortieth iteration: (30,000 hours) ($60 per hour) = $1,800,000

8.24 Assume you are a project cost engineer calculating the cost of a repetitive activity for your project. There are a total of 20 iterations of this activity required for the project. The project activity takes 2.5 hours at its steady state rate and the learning rate is 75%. Calculate the initial output time for the first unit produced, using the learning curve formula: Yx = aXb Where: Yx = the time required for the steady state, x, unit of output a = the time required for the initial unit of output X = the number of units to be produced to reach the steady state b = the slope of the learning curve, represented as: log decimal learning rate/log 2 Solution: Use the formula to first determine the slope of the learning curve (log decimal learning rate/ log 2): b

= = =

log 0.75/log 2 −.2877/0.693 −.415

2.5 hrs =

a (20) −0.415

a

8.667 hours

=

8.25 As the manager of the IT group at your insurance firm, you have been asked to develop a cost estimate for upgrades to the computerized accidentreporting and claims adjustment system you have in place. Your system is basic, without many features, but it needs some general modifications, based on complaints from customers and claims adjusters at your firm. You know that your programmer is capable of handling 3 function points in a person-month and your 145 Copyright © 2019 Pearson Education, Inc.


programmer makes $60,000, so her cost is $5,000 per month. The costs for the project are based on the following requirements:

Function Input Output Interfaces Queries Files

Number of Screens 9 4 7 7 12

Complexity Medium Low Medium High Low

The complexity weighting for these functions follows a standard formula:

Function Low Number of Inputs 1  _____  Number of Outputs 2  _____  Number of Interfaces 10  _____  Number of Queries 3  _____  Number of Files 1  _____ 

Medium 2  _____  6  _____  15  _____  6  _____  3  _____ 

High Total 3  _____  10  _____  20  _____  9  _____  5  _____ 

a. Calculate the total estimated number of function points for this project. b. What is the total expected cost of the project? Solution: a. Because we know the estimated complexity for each of the software functions that must be coded and the number of screens for each function, the calculation is relatively straightforward: Multiplying, we find that the total function points for this project are estimated to be 206.

Function Low Number of Inputs 1  _____  Number of Outputs 2  __4__  Number of Interfaces 10  _____  Number of Queries 3  _____  Number of Files 1  _12__ 

Medium 2  __9__  6  _____  15  __7__  6  _____  3  _____ 

High 3  _____  10  _____  20  _____  9  __7__  5  _____ 

Total 18 8 105 63 12

b. Your programmer can complete 3 function points per month, so the total number of person-months needed to complete the project is: 206/3 = 68.67. We multiply this value by the cost per programmer per month of $5,000 to estimate the total cost for this project as $343,333.

146 Copyright © 2019 Pearson Education, Inc.


8.26 You work at a regional health care center and have been asked to calculate the expected cost for a software project in your organization. You know that historically, your programmers can handle 5 function points each person-month and that the cost per programmer in your company is $4,000 per month. The project whose costs you are estimating is based on the following requirements: Function Input Output Interfaces Queries Files

Number of Screens 8 6 15 5 25

Complexity Low Low High High Medium

Further, you know that the complexity weighting for these functions follows a standard internal formula, shown as:

Function Number of Inputs Number of Outputs Number of Interfaces Number of Queries Number of Files

Complexity Weighting Low Medium High 2 × _____ = 4 × _____ = 6 × _____ = 3 × _____ = 6 × _____ = 12 × _____ = 6 × _____ = 12 × _____ = 18 × _____ = 4 × _____ = 2 × _____ =

6 × _____ = 4 × _____ =

Total

8 × _____ = 8 × _____ =

a. Calculate the total estimated number of function points for this project. b. Calculate the total expected cost of the project. Solution: Because we know the estimated complexity for each of the software functions that must be coded and the number of screens for each function, the calculation is relatively straightforward: Multiplying, we find that the total function points for this project are estimated to be 414.

Function Number of Inputs Number of Outputs Number of Interfaces Number of Queries Number of Files

Low 2 × __8 _ = 3 × __6__ =

Complexity Weighting Medium

High

18 × _15__ = 8 × __5__ = 4 × _25__ =

Total 16 18 270 40 100

Each resource can complete 5 function points per month, so the total number of personmonths needed to complete the project is 414/5 = 82.80. We multiply this value by the 147 Copyright © 2019 Pearson Education, Inc.


cost per programmer per month of $4,000 to estimate the total cost for this project as $331,200.

148 Copyright © 2019 Pearson Education, Inc.


CHAPTER NINE Project Scheduling Networks, Duration Estimation, and Critical Path Chapter Outline PROJECT PROFILE Preparing for a Major Golf Tournament – It’s a Long Road to the First Tee INTRODUCTION 9.1 PROJECT SCHEDULING 9.2 KEY SCHEDULING TERMINOLOGY 9.3 DEVELOPING A NETWORK Labeling Nodes Serial Activities Concurrent Activities Merge Activities Burst Activities 9.4 DURATION ESTIMATION 9.5 CONSTRUCTING THE CRITICAL PATH Calculating the Network The Forward Pass The Backward Pass Probability of Project Completion Laddering Activities Hammock Activities Options for Reducing the Critical Path PROJECT MANAGEMENT RESEARCH IN BRIEF Software Development Delays and Solutions Summary Key Terms Solved Problems Discussion Questions Problems Moving the Historic Capen House Internet Exercises MS Project Exercises PMP Certification Sample Questions Answers Notes

149 Copyright © 2019 Pearson Education, Inc.


TRANSPARENCIES Note for Instructors: To present transparencies in class, please download the PowerPoint Presentations (available on www.pearsonhighered.com) that accompanies this product. The PPT ISBN is 9780134730479.

9.1 RULES FOR DEVELOPING ACTIVITY NETWORKS 1. Some determination of activity precedence ordering must be done prior to creating the network. 2. Network diagrams usually flow from left to right. 3. An activity cannot begin until all preceding connected activities have been completed. 4. Arrows on networks indicate precedence and logical flow. Arrows can cross over each other, although it is helpful for clarity’s sake to limit this effect when possible. 5. Each activity should have a unique identifier associated with it (number, letter, code, etc.). 6. Looping, or recycling through activities, is not permitted. 7. Although not required, it is common to start a project from a single beginning node. A single node point also is typically used as a project end indicator.

150 Copyright © 2019 Pearson Education, Inc.


9.2 LABELS FOR ACTIVITY NODE Early start

Identifier number

Activity float

Activity descriptor

Late start

Activity duration

Early finish

Late finish

9.3 PROJECT ACTIVITIES LINKED IN SERIES

9.4 ACTIVITIES LINKED IN PARALLEL (CONCURRENT)

151 Copyright © 2019 Pearson Education, Inc.


9.5 MERGE ACTIVITIES

Activity A

Activity B

Activity D

Activity C

152 Copyright © 2019 Pearson Education, Inc.


9.6 BURST ACTIVITIES

Activity B

Activity A

Activity C

Activity D

153 Copyright © 2019 Pearson Education, Inc.


9.7 EXAMPLE OF CREATING A PROJECT ACTIVITY NETWORK

Information for Network Construction Name: Project Delta Activity

Description

Predecessors

A

Contract signing

None

B

Questionnaire design

A

C

Target market ID

A

D

Survey sample

B, C

E

Develop presentation

B

F

Analyze results

D

G

Demographic analysis

C

H

Presentation to client

E, F, G

154 Copyright © 2019 Pearson Education, Inc.


9.8 ACTIVITY NETWORK FOR EXAMPLE

E Dev. Present.

B Design

A Contract

D Survey

C Market ID

F Analysis

H Present

G Demog.

155 Copyright © 2019 Pearson Education, Inc.


9.9 ACTIVITY DURATION ESTIMATION – BETA DISTRIBUTION

ESTIMATED TIME FORMULA TE = A + 4(B) + C 6

where A = Optimistic Time B = Most Likely Time C = Pessimistic Time

156 Copyright © 2019 Pearson Education, Inc.


9.10 CONSTRUCTING THE CRITICAL PATH

INFORMATION FOR PROJECT DELTA Activity A

Description Contract signing

Predecessors Estimated Duration None 5

B

Questionnaire design

A

5

C

Target market ID

A

6

D

Survey sample

B, C

13

E

Develop presentation

B

6

F

Analyze results

D

4

G

Demographic analysis

C

9

H

Presentation to client

E, F, G

2

157 Copyright © 2019 Pearson Education, Inc.


9.10 (Con’d)

Partial Project Activity Network with Task Durations

B Design 5

A Contract 5

E Dev. Present

6

D Survey 13

C Market ID 6

F Analysis 4

H Present 2

G Demog. 9

158 Copyright © 2019 Pearson Education, Inc.


RULES WHEN USING THE FORWARD PASS

1. Add all activity times along each path as we move through the network (ES + Dur = EF),

2. Carry the EF time to the activity nodes immediately succeeding the recently completed node. That EF becomes the ES of the next node, unless the succeeding node is a merge point.

3. At a merge point, the largest preceding EF becomes the ES for that node.

159 Copyright © 2019 Pearson Education, Inc.


9.12 ACTIVITY NETWORK WITH FORWARD PASS

5

0

B 10 Design 5

A 5 Contract 5

10

C 11 Market ID 6

16

6

11

5

E

Dev. Present

D 24 Survey 13

24 F 28 Analysis 4

11

28

H 30 Present 2

G 20 Demog. 9

160 Copyright © 2019 Pearson Education, Inc.


9.13 RULES FOR USING THE BACKWARD PASS 1. Subtract activity times along each path as you move through the network (LF − Dur = LS).

2. Carry back the LS time to the activity nodes immediately preceding the successor node. That LS becomes the LF of the next node, unless the preceding node is a burst point.

3. In the case of a burst point, the smallest succeeding LS becomes the LF for that node.

161 Copyright © 2019 Pearson Education, Inc.


9.14

ACTIVITY NETWORK WITH BACKWARD PASS

5 6

0 0

B 10 Design 5 11

A 5 Contract 5 5

10 22

11 11

5

C 11 Market ID 5 6 11

E

16

Dev. Present

6

D 24 Survey 13 24

28

24

F

28

28

Analysis

24

11

G

4

28

H

30

Presentation

28

2

30

20

Demograph.

19

9

28

162 Copyright © 2019 Pearson Education, Inc.


9.15

COMPLETED ACTIVITY NETWORK WITH CRITICAL PATH AND ACTIVITY SLACK TIMES IDENTIFIED

Critical Path is indicated in bold 5 1 6

0 0 0

B 10 Design 5 11

A 5 Contract 5 5

10 22

11 0 11

5 C 11 0 Market ID 5 6 11

E

16

12 Dev. Present

6

D 24 Survey 13 24

11

G

28

24

F

0

Analysis

28

28

0 Presentation

24

4

28

28

H 2

30 30

20

8 Demograph.

19

9

28 ES

ID

Slack

Task Name

LS

Duration

EF

LF

163 Copyright © 2019 Pearson Education, Inc.


9.16

Probability of Project Completion The formula for activity variance is: s2 = [(b − a)/6]2, where b is the most pessimistic time and a is the most optimistic Example: Activity A: [(11 − 3)/6]2 = (8/6)2 = 64/36, or 1.78 weeks variance The formula for project variance is:  p2  Project variance   (variances of activities on critical path)

164 Copyright © 2019 Pearson Education, Inc.


Example: Suppose our critical path had four activities with the following variances: Activity A: 1.78 Activity B: 4.00 Activity C: 0.69 Activity D: 1.00 Project variance: 1.78 + 4.00 + 0.69 + 1.00 = 7.47 days Project standard deviation = √7.47 = 2.73 days

165 Copyright © 2019 Pearson Education, Inc.


Suppose our project has a critical path of 16 days. What is the probability that the project will finish by day 18? Solution: Using the standard normal equation: Z = (Due date − Expected date of completion)/σp Z = (18 − 16)/2.73, or 0.73 Normal tables show that a Z value of 0.73 equates to a probability of 0.7673 There is a 76.7% chance of finishing the project by day 18. How many extra days do we need to generate a 99% chance of finishing on time?

Solution: Using the normal table, we find that a 99% likelihood equates to a Z score of 2.33, Therefore: Due Date = Expected date of completion + (Z × σp) = 16 days + (2.33 × 2.73) = 16 + 6.36 = 22.36 days

166 Copyright © 2019 Pearson Education, Inc.


ACTIVITY NETWORK DEMONSTRATING LADDERING TECHNIQUE

A1 Design

A2 Design

A3 Design

A1 Coding

A2 Coding

A3 Coding

A1 Debugging

A2 Debugging

A3 Debugging

167 Copyright © 2019 Pearson Education, Inc.


9.17

NETWORK DEMONSTRATING HAMMOCK ACTIVITY

5 13 18 0 0 0

A

5

5

5

B

9

4

22

5 9 14

12 10 22

C

12

7

21

5 D 11 0 User needs 5 6 11

21

9

31

12 9 21

H

22

31

I

10

31

31

4

11

E

25

25

0

Coding

0

Debugging

25

6

11

5

G

14

25

F

35 0 35

31 31

A 31 Hammock 26

168 Copyright © 2019 Pearson Education, Inc.


9.18

STEPS TO REDUCE THE CRITICAL PATH

1. ELIMINATE TASKS ON THE CRITICAL PATH. 2. RE-PLAN SERIAL PATHS TO BE IN PARALLEL. 3. OVERLAP SEQUENTIAL TASKS. 4. SHORTEN THE DURATION OF CRITICAL PATH TASKS. 5. SHORTEN EARLY TASKS. 6. SHORTEN LONGEST TASKS. 7. SHORTEN EASIEST TASKS. 8. SHORTEN TASKS THAT COST THE LEAST TO SPEED UP.

DISCUSSION QUESTIONS 9.1 Define the following terms: a. Path: group of activities sequenced by relationship through project network logic b. Activity: any piece of work that will be performed during the project, which has an expected time and cost for completion c. Early start: the earliest possible date upon which an uncompleted activity or project can start, based on sequencing and scheduling constraints d. Early finish: the earliest possible date upon which an uncompleted activity or project can be completed e. Late start: the latest date an activity may start without delaying other project milestones or the project’s expected completion date f. Late finish: the latest date an activity may end without delaying other project milestones or the project’s expected completion date g. Forward pass: a process that works forward though the project network to determine the earliest start and earliest finish time for an activity

169 Copyright © 2019 Pearson Education, Inc.


h. Backward pass: a process that works backward through the project network to calculate the latest finish times for an uncompleted activity i. Node: a convergence point of dependent paths in a network j. AON: Activity on Node; a method of logic that determines activity networks in which a node depicts an activity and arrows that indicate sequencing between nodes k. Float or Slack: a calculation that determines the amount of time an activity can be delayed from its earliest start date without delaying the project’s completion date l. Critical path: the path through the project network having the least amount of float time and the longest time duration m. PERT: Program Evaluation and Review Technique; a network analysis system based on events and probability used when activities and their duration are difficult to define

9.2 Distinguish between serial activities and concurrent activities. Why do we seek to use concurrent activities as a way to shorten a project’s length? Serial activities begin with the first step and proceed to subsequent steps one at a time sequentially until the project is completed. Serial activities must be completed in an order and one at a time. Therefore, step 2 cannot begin until step1 has been completed, and so on. Concurrent activities allow more than one activity to be performed during the same time period. This means step 1 may still be in progress when step 2 is started. Project teams seek out concurrent activities because they allow multiple phases of the project to be progressing simultaneously. Time savings occur from several activities being completed at the same time and delays in one step do not created delays in other concurrent activities. This method allows activities to work more independently, which means the project can progress at a faster pace.

9.3 List three methods for deriving duration estimates for project activities. What are the strengths and weaknesses associated with each method? One method for deriving time estimates is past experience. This method is beneficial in that it is easy and uses past examples of similar activities to predict future time estimates. However, it is limited in that estimates can be distorted by extenuating circumstances, changes in time and conditions, and information obsolescence. Another method uses expert opinion. Again, the approach is simple to use and draws on experience and knowledge of experts. The shortcomings here involve potential inadequacy of staff (at least relative to the expert giving the opinion) and project-specific complications. A third method employs mathematical derivations. This approach is more objective and allows 170 Copyright © 2019 Pearson Education, Inc.


multiple estimates (based on best, most likely and worst case analysis). The weaknesses of this method are that it is slightly more difficult to use and it disregards past failures (a.k.a. lessons learned).

9.4 In your opinion, what are the chief benefits and drawbacks of using beta distribution calculations (based on PERT techniques) to derive activity duration estimates? Beta distribution allows for the likelihood that optimistic and pessimistic times will not be symmetrical. By including realistic estimates of pessimistic and optimistic durations, beta distribution creates a more accurate distribution of alternative duration times. One drawback to this method is that it relies on estimates of pessimistic and optimistic time estimates, which are not be reliable. There has also been some debate related to how the time estimates in this method should be calculated and/or interpreted. 9.5 “The shortest total length of a project is determined by the longest path through the network.” Explain the concept behind this statement. Why does the longest path determine the shortest project length? This is based on the concept of critical path. The critical path combines the project activity network (the order to be followed for start/completion of activities) and the estimated time duration of activities in the sequence (how long each activity will take to complete) to determine the length of time required to complete the project. The longest path of sequential events is used to establish the project’s duration because the events in the path must be performed one after another. Adding the duration times of activities in the critical path will result in the shortest project length (i.e., how long it will take to perform required serial activities).

9.6 The float associated with each project task can only be derived following the completion of the forward and backward passes. Explain why this is true. The forward pass establishes the earliest time that activities in the network can begin and end. The backward pass determines the latest time that activities in the network can begin and end. Float time is the difference between the task’s latest and earliest end time (or the task’s latest and earliest start time). Hence, float cannot be calculated until the forward and backward pass have been completed.

171 Copyright © 2019 Pearson Education, Inc.


PROBLEMS 9.7 Consider a project such as moving to a new neighborhood, completing a longterm school assignment, or even cleaning your bedroom. Develop a set of activities necessary to accomplish that project, and then order them in a precedence manner to create sequential logic. Explain and defend the number of steps you identified and the order in which you placed those steps for best completion of the project. SOLUTION: This problem is intended to get students thinking sequentially; that is, developing first a set of activities or tasks and then applying some informal sequential logic to the order so that they can become familiar with concepts such as predecessor and successor activities. The key is to challenge their sequencing to determine if they have correctly identified both the necessary activities and the order in which they should be considered.

9.8 What is the time estimate of an activity in which the optimistic estimate is 4 days, pessimistic is 12 days, and most likely is 5 days? Show your work. SOLUTION: Using the beta distribution for probabilistic estimation, the formula is given as: TE = (a + 4m + b)/6 where TE = estimated time for activity a = optimistic time to complete the activity m = most likely time to complete the activity (the mode of the distribution) b = pessimistic time to complete the activity The solution to this problem is: TE

= (4 + 4(5) + 12)/6, or = 6 days

9.9 What is the time estimate of an activity in which the optimistic time is 5 days, the likely time is 8 days, and the pessimistic time is 14 days? Show your work.

172 Copyright © 2019 Pearson Education, Inc.


SOLUTION: Using the beta distribution for probabilistic estimation, the formula is given as:

TE = (a + 4m + b)/6 where: TE = estimated time for activity a = optimistic time to complete the activity m = most likely time to complete the activity (the mode of the distribution) b = pessimistic time to complete the activity The solution to this problem is: TE

= (5 + 4(8) + 14)/6, or = 8.5 days

9.10 Using the following information, develop an activity network for Project Alpha. Activity A B C D E F G H

Preceding Activities — A A B, C B D C E, F, G

173 Copyright © 2019 Pearson Education, Inc.


SOLUTION:

9.11 Activity A B C D E F G H I J

Construct a network activity diagram based on the following information: Preceding Activities — A A A B C, D E, F F G, H I

SOLUTION:

174 Copyright © 2019 Pearson Education, Inc.


9.12 You have a partial network for your project, and you are about to conduct a forward pass through it. Explain why the early start (ES) for activity G is 23 days, not 19 days. Remember, activity G is a merge activity.

SOLUTION: We know from our decision rules for merge activities that the early start for a successor (merge) activity will be the predecessor with the largest early finish time. This is because the merge activity cannot begin until all predecessors have been completed, which means that the largest early finish determines when all of the predecessor activities have been completed.

9.13 Referring to the figure in Problem 9.12, suppose that Activity G was the last activity in the network and you were about the start a backward pass. a. What is the project’s duration? (Hint: complete the forward pass) b. What are the late finish (LF) and late start (LS) values for Activities E, F, and G? c. Which activity has slack time? How many days?

175 Copyright © 2019 Pearson Education, Inc.


SOLUTION: A. Adding 4 days duration to the early start of 23 days (found from problem 9.12) will give an early finish of 27 days. This is the project duration. B. Late finish and late start are found by starting with the final activity node (G) and working backward through the network. We know that the late finish for Activity G is 27 days from the decision rule and then subtract 4 days for the late start time of 23 days. Our decision rule suggests moving that 23 number to the late finish for both Activities E and F and then subtracting their durations for the late start days. Activity E has a late start of 16 days and a late finish of 23 days. Activity F has a late finish of 23 days and a late start of 18 days. a. Activity F has 4 days of slack time 9.14 Referring to the figure shown below, suppose that Activity O was the last activity in the network and you were about to start a backward pass. a. What is the project’s duration? (Hint: complete the forward pass) b. What are the late finish (LF) and late start (LS) values for activities L, M, N, and O? c. Which activities have slack time? How many days?

176 Copyright © 2019 Pearson Education, Inc.


SOLUTION: a. Adding 4 days duration to the early start of 18 days (largest of the predecessor early finish times) will give an early start to Activity O of 18 days. Adding its 4 day duration gives an early finish of 22 days. This is the project duration. b. Late finish and late start are found by starting with the final activity node (G) and working backward through the network. We know that the late finish for Activity O is 22 days from the decision rule and then subtract 4 days for the late start time of 18 days. Our decision rule suggests moving that 18-day number to the late finish for activities L, M, and N and then subtracting their durations for the late start days. Activity L has a late start of 11 days and a late finish of 18 days. Activity M has a late finish of 18 days and a late start of 13 days. Activity N has a late finish of 18 days and a late start of 16 days. c. Activity L has 1 day of slack and Activity N has 2 days of slack.

177 Copyright © 2019 Pearson Education, Inc.


9.15 Your university is holding a fund-raiser and will be hiring a band to entertain spectators. You have been selected to serve as the event project manager and have created a Work Breakdown Structure and duration estimates for the activities involved in site preparation for the event. Construct a network activity diagram based on the following information: Activity A B C D E F G H I J

Description Site selection Buy concessions Rent facilities Build stands Generator & wiring installation Security Lighting installation Sound system installation Stage construction Tear down

Predecessors None A A A C

Duration (Days) 4 4 2 5 2

B E E, F D G, H, I

4 2 2 3 4

a. Conduct both a forward and backward pass using AON notation. What is the estimated total duration for the project? b. Identify all paths through the network. Which is the critical path? c. Which activities have slack time? d. Identify all burst activities and merge activities.

SOLUTION:

178 Copyright © 2019 Pearson Education, Inc.


a. The estimated total duration for the project is 18 days. b. Identify all paths through the network. The paths through the network include: (1) A – B – F – H – J; (2) A – C – E – H – J; (3) A – C – E – G – J; and (4) A – D – I – J. The critical path is A – B – F – H – J, containing no slack activities. c. The slack activities are shown with the slack time embedded in the AON notation (e.g., Activity C has 4 days slack). d. Burst Activities are A and E. Merge activities are H and J.

9.16 Consider the following project tasks and their identified best, likely, and worstcase estimates of task duration. Assume the organization you work for computes TE based on the standard formula. Calculate the TE for each of the following tasks (round to the nearest integer): Activity

Best

Likely

Worst

A

5

5

20

B

3

5

9

C

7

21

26

D

4

4

4

E

10

20

44

F

3

15

15

G

6

9

11

H

32

44

75

I

12

17

31

J

2

8

10

TE

179 Copyright © 2019 Pearson Education, Inc.


SOLUTION: Using the beta distribution for probabilistic estimation, the formula is given as:

TE = (a + 4m + b)/6 where TE = estimated time for activity a = optimistic time to complete the activity m = most likely time to complete the activity (the mode of the distribution) b = pessimistic time to complete the activity

Activity

Best

Likely

Worst

TE

A

5

5

20

8

B

3

5

9

5

C

7

21

26

20

D

4

4

4

4

E

10

20

44

22

F

3

15

15

13

G

6

9

11

9

H

32

44

75

47

I

12

17

31

19

J

2

8

10

7

9.17 Consider the following project tasks and their identified best, likely, and worstcase estimates of task duration. Assume the organization you work for computes TE based on the standard beta distribution formula. Calculate the TE for each of the following tasks (round to the nearest integer):

180 Copyright © 2019 Pearson Education, Inc.


SOLUTION: Activity

Best

Likely

Worst

TE

A

4

5

10

6

B

4

6

9

6

C

2

5

8

5

D

5

8

10

8

E

12

16

20

16

F

6

10

12

10

G

5

9

14

9

H

14

16

22

17

I

10

14

20

14

J

1

2

5

2

181 Copyright © 2019 Pearson Education, Inc.


9.18 Using the following information, create an AON network activity diagram. a. Calculate each activity TE (rounding to the nearest integer), the total duration of the project, its early start, early finish, late start, and late finish times, and the slack for each activity. Finally, show the project’s critical path. b. Now, assume that activity E has taken 10 days past its anticipated duration to complete. What happens to the project’s schedule? Has the duration changed? Is there a new critical path? Show your conclusions. SOLUTION: a. Activity Preceding activities

Best

Likely

Worst

TE

A

12

15

25

16

B

A

4

6

11

7

C

12

12

30

15

D

B, C

8

15

20

15

E

A

7

12

15

12

F

E

9

9

42

15

G

D, E

13

17

19

17

H

F

5

10

15

10

I

G

11

13

20

14

J

G, H

2

3

6

3

K

J, I

8

12

22

13

182 Copyright © 2019 Pearson Education, Inc.


Total duration of the project is 82 days. Critical Path: A – B – D – G – I – K c. Because Activity E has 10 days of slack associated with it, delaying the completion of E by 10 days uses up all the slack; therefore, Activity E is now part of the project’s critical path. The overall project’s schedule remains unchanged except there is a new critical path: A – E – G – I – K.

9.19 An advertising project manager has developed a program for a new advertising campaign. In addition, the manager has gathered the time information for each activity as shown in the following table.

Activity A B C D E F G

Optimistic 1 2 3 6 4 6 2

Time Estimates (week) Most Likely Pessimistic 4 7 6 10 3 9 13 14 6 14 8 16 5 8

Immediate Predecessor(s) — — B A A, C B D, E, F 183

Copyright © 2019 Pearson Education, Inc.


a. Calculate the expected activity times (round to nearest integer). b. Calculate the activity slacks. What is the total project length? Make sure you fully label all nodes in the network. c. Identify the critical path. What are the alternative paths and how much slack time is associated with each noncritical path? d. Identify the burst activities and the merge activities. e. Given the activity variances, what is the likelihood of the project finishing on week 24? f. Suppose you wanted to have a 99% confidence in the project finishing on time. How many additional weeks would your project team need to negotiate for in order to gain this 99% likelihood? Solution: a. Calculated activity times are: Activity Expected Duration A 4 B 6 C 4 D 12 E 7 F 9 G 5

b. 0 1 1

0 0 0

A

4

4

5

B

6

6

6

4 1 5

D

6 0 6

16

12 17

C

10

4

10

10 0 10

E

17

7

17

6 2 8

F

15

9

17

17 G 0 17 5

22 22

The total project length is 22 weeks. 184 Copyright © 2019 Pearson Education, Inc.


c. The critical path is: B – C – E – G

Other paths are:

A–D–G A–E–G B–F–G

Path Slack 2 days 1 day 2 days

d. Burst Activities: A and B Merge Activities: E and G

e. Duration Estimates Activity Pessimistic Likely Optimistic TE (Beta) A 7 4 1 4 B 10 6 2 6 C 9 3 3 4 D 14 13 6 12 E 14 6 4 7 F 16 8 6 9 G 8 5 2 5 2 Project variance ( p ) = 1.78 + 1.00 + 2.78 + 1.00 = 6.56

Variance [(7 − 1)/6]2 = 36/36 = 1.00 [(10 − 2)/6]2 = 64/36 = 1.78 [(9 − 3)/6]2 = 36/36 = 1.00 [(14 − 6)/6]2 = 64/36 = 1.78 [(14 − 4)/6]2 = 100/36 = 2.78 [(16 − 6)/6]2 = 100/36 = 2.78 [(8 − 2)/6]2 = 36/36 = 1.00

The project standard deviation (σp) is found as: project variance = 2.56 weeks The likelihood of the project finishing on week 24 is found by the formula: Z = (Due date − Expected date of completion)/σp = (24 − 22)/2.56, or 0.78 Consulting the normal table (Appendix A), we find that a Z value of 0.78 equates to a probability of 78.23% f. In order to have a 99% confidence in the project finishing on time, we would rewrite the above formula as: Due Date

= Expected date of completion + (Z × σp) = 22 weeks + (2.33 × 2.56) = 27.96, or approximately 28 weeks would give us a 99% probability.

185 Copyright © 2019 Pearson Education, Inc.


9.20

Consider a project with the following information:

Activity A B C D E F G H

Activity A B C D E F G H

Duration 3 5 7 3 5 4 2 5

Duration 3 5 7 3 5 4 2 5

ES 0 3 3 10 8 13 10 17

Predecessors — A A B, C B D C E, F, G

EF 3 8 10 13 13 17 12 22

LS 0 5 3 10 12 13 15 17

LF 3 10 10 13 17 17 17 22

Slack — 2 — — 4 — 5 —

a. Construct the project activity network using AON methodology and label each node. b. Identify the critical path and other paths through the network.

186 Copyright © 2019 Pearson Education, Inc.


Solution: a.

a. The critical path is: A – C – D – F – H Alternative paths are:

A–B–E–H A–B–D–F–H A–C–G–H

9.21 Use the following information to determine the probability of this project finishing within 34 weeks of its scheduled completion date. Assume activities A – B – D – F – G are the project’s critical path. a. Calculate the expected durations for each activity. b. Calculate individual task variances and overall project variance. c. The company must file a permit request with the local government within a narrow time frame after the project is expected to be completed. What is the likelihood that the project will be finished by week 34? d. If we wanted to be 99% confident of on-time delivery of the project, how much additional time would we need to add to the project’s expected delivery time?

187 Copyright © 2019 Pearson Education, Inc.


a. Activity

Optimistic Likely Pessimistic

A B C D E F G

1 3 4 3 5 3 4

4 5 6 7 10 6 7

8 9 10 15 16 15 12

Expected Time 4.17 5.33 6.33 7.67 10.17 7.0 7.33

Variance s2 = [1/6(b – a)]2 [(8 – 1)/6]2 = 1.36 [(9 – 3)/6]2 = 1.00 [(10 – 4)/6]2 = 1.00 [(15 – 3)/6]2 = 4.00 [(16 – 5)/6]2 = 3.36 [(15 – 3)/6]2 = 4.00 [(12 – 4)/6]2 = 1.78

b. Recall that the critical activities for this project were: A – B – D – F – G. For the overall project variance, the calculation is: Project variance ( p2 ) = 1.36 + 1.00 + 4.00 + 4.00 + 1.78 = 12.14 The project standard deviation (σp) is found as:

project variance = 3.48 weeks

c. The expected time to complete the project is the sum of the expected durations of all activities on the critical path. In this case, the calculation is: TE = 4.17 + 5.33 + 7.67 + 7.0 + 7.33 = 31.50 weeks Applying the standard normal equation to this problem, we find: Z = (Due date − Expected date of completion)/σp = (34 − 31.5)/3.48, or 0.72 We then go to the normal table (Appendix A) to determine that a value of 0.72 equates to a probability of .7642, or a 76.42% chance of on-time completion.

d. Using the normal table (Appendix A) we find that a 99% chance of completion equates to a Z value of 2.33. We can calculate the required time by using the formula: Due Date

= Expected date of completion + (Z × σp) = 31.5 weeks + (2.33 × 3.48) = 39.61 weeks, or about 8 additional weeks past the original completion time.

Case Study 9.1 Moving the Historic Capen House

188 Copyright © 2019 Pearson Education, Inc.


Based on a real event, this case is intended to task students with the challenge of translating into a project schedule, the challenge of identifying and ordering various project activities to create a coherent and logical network. Students have to identify relevant tasks, organize them into a predecessor network, and make it as efficient as possible by identifying paths that can occur in parallel as well as those tasks that must occur in series. There is no strictly right or wrong answer to this challenge, but instructors can reward the individual or team that develops the most efficient and logical network. Further, if instructors wish, they can add durations to the activities and challenge students to create the best (fastest) workable network that moves the house in the fastest time.

MS Project Exercises Exercise 9.27 Consider the following information that you have compiled regarding the steps needed to complete a project. You have identified all relevant steps and have made some determination regarding predecessor/successor relationships. Using MS Project, develop a simple network diagram for this project, showing the links among the project activities. Activity A – Survey site B – Install sewer and storm drainage C – Install gas and electric power lines D – Excavate site for spec house E – Pour foundation

Predecessors — A A B, C D

Solution: Entering the activities and assigning the predecessor relationships using MS Project, the network diagram should resemble the following:

Microsoft product screen shot(s) reprinted with permission from Microsoft Corporation.

189 Copyright © 2019 Pearson Education, Inc.


Exercise 9.28 Suppose we have a complete activity predecessor table (shown below) and we wish to create a network diagram highlighting the activity sequence for this project. Using MS Project, enter the activities and their predecessors and create a complete activity network diagram for this project. Project—Remodeling an Appliance Activity A. Conduct competitive analysis B. Review field sales reports C. Conduct tech capabilities assessment D. Develop focus group data E. Conduct telephone surveys F. Identify relevant specification improvements G. Interface with marketing staff H. Develop engineering specifications I. Check and debug designs J. Develop testing protocol K. Identify critical performance levels L. Assess and modify product components M. Conduct capabilities assessment N. Identify selection criteria O. Develop RFQ P. Develop production master schedule Q. Liaison with sales staff R. Prepare product launch

Predecessors — — — A, B, C D E F G H G J I, K L M M N, O P Q

190 Copyright © 2019 Pearson Education, Inc.


Solution: Entering the above values and predecessor relationships, we would derive the following network diagram. Note that as in the case of Exercise 9.27, without duration estimates, all activities are assumed to be of similar duration, making every activity in the network part of the critical path.

Microsoft product screen shot(s) reprinted with permission from Microsoft Corporation.

191 Copyright © 2019 Pearson Education, Inc.


Exercise 9.29 Suppose that we add some duration estimates to each of the activities from Exercise 9.28. A portion of the revised table is shown here. Recreate the Network diagram for this project and note how MS Project uses nodes to identify activity durations, start and finish dates, and predecessors. What is the critical path for this network diagram? How do we know? Activity

Duration

A – Survey site B – Install sewer and storm drainage C – Install gas and electric power lines D – Excavate site for spec house E – Pour foundation

5 days 9 days 4 days 2 days 2 days

Predecessors — A A B, C D

Solution: Entering the above values and durations, the network diagram would resemble the following. If this network was shown on a computer screen, you would note that the critical path is highlighted in red, identifying the activities A – B – D – E as lying on the critical path.

Microsoft product screen shot(s) reprinted with permission from Microsoft Corporation.

192 Copyright © 2019 Pearson Education, Inc.


CHAPTER SEVEN Risk Management CHAPTER OUTLINE PROJECT PROFILE Samsung’s Galaxy Note 7—Failure to Manage a New Product for Risk 7.1 INTRODUCTION: WHAT IS PROJECT RISK? PROJECT MANAGERS IN PRACTICE Mathew Paul, General Electric Company 7.2 RISK MANAGEMENT: A FOUR-STAGE PROCESS Risk Identification PROJECT PROFILE Japan Decommissions a $9 billion Nuclear Reactor that was Hardly Used Risk Breakdown Structures Analysis of Probability and Consequences Risk Mitigation Strategies Use of Contingency Reserves Other Mitigation Strategies Control and Documentation PROJECT PROFILE Collapse of Shanghai Apartment Building 7.3 PROJECT RISK MANAGEMENT: AN INTEGRATED APPROACH Summary Key Terms Solved Problems Discussion Questions Problems Case Study 7.1 Classic Case: de Havilland’s Falling Comet Case Study 7.2 The Building that Melted Cars Case Study 7.3 Classic Case: Tacoma Narrows Suspension Bridge Internet Exercises PMP Certification Sample Questions Answers Integrated Project—Project Risk Assessment Notes

108 Copyright 2019 © Pearson Education, Inc.


TRANSPARENCIES Note for Instructors: To present transparencies in class, please download the PowerPoint Presentations (available on www.pearsonhighered.com) that accompanies this product. The PPT ISBN is 9780134730479.

7.1

RISK VERSUS AMOUNT AT STAKE

109 Copyright 2019 © Pearson Education, Inc.


7.2

TYPICAL RISK FACTORS 1. Financial Risks

2. Technical Risks

3. Commercial Risks

4. Execution Risks

5. Contractual or Legal Risks

110 Copyright 2019 © Pearson Education, Inc.


7.3 RISK BREAKDOWN STRUCTURE (RBS)

111 Copyright 2019 © Pearson Education, Inc.


7.4 QUALITATIVE RISK ASSESSMENT MATRIX

112 Copyright 2019 © Pearson Education, Inc.


7.5 QUANTITATIVE RISK ASSESSMENT CALCULATIONS

113 Copyright 2019 © Pearson Education, Inc.


7.6 QUANTITATIVE RISK ASSESSMENT CALCULATIONS (CON’D)

114 Copyright 2019 © Pearson Education, Inc.


7.7 QUANTITATIVE RISK ASSESSMENT CALCULATIONS (CON’D)

115 Copyright 2019 © Pearson Education, Inc.


7.8 CONTINGENCY DOCUMENT FOR ADJUSTMENTS TO PROJECT PLAN Probable Event

ADJUSTMENT TO PLANS

Absenteesim

Resignation

Pull-aways

Unavailable Staff/skills

Spec Change

Added work

Need more training

Vendors late

116 Copyright 2019 © Pearson Education, Inc.


DISCUSSION QUESTIONS 7.1 Do you agree with the following statement: “With proper planning, it is possible to eliminate most/all risks from a project”? Why or why not? It is not possible to eliminate risk from a project regardless of planning. The role of risk management is to identify and analyze potential risks associated with a project. Once risks have been identified, preventative action or contingency plans may be established to reduce the impact of the risk on the success of the project. While this presents a way to help control the effects of risk, it does not eliminate risk from the project management equation.

7.2 In evaluating projects across industries, it is sometimes possible to detect patterns in terms of the more common types of risks they routinely face. Consider the development of a new software product and compare it to coordinating an event, such as a school dance. What likely forms of risk would your project team face in either of these circumstances? A new software product would include a higher amount of risk than a school dance. The software project would be subject to risk related to market volume/price, technical risk, financing, scheduling, resignation, organizational, and operating risk. Coordinating a school dance would run risks associated with availability of staff and an appropriate facility as well as adequate funding. Both projects would involve risk variables such as cost estimate risk, integration risk, “acts of God,” inadequate skills availability, and absenteeism.

7.3 Analyze Figure 7.2 (degree of risk over the project life cycle). What is the practical significance of this model? What implications does it suggest for managing risk? The diagram demonstrates the relationship between level of opportunity and risk throughout the project life cycle. This is helpful in determining points at which uncertainty is at its highest and when the greatest amount of risk may be realized. Project teams can focus their risk analysis in these areas or use the diagram to give weight or perspective to potential risk variables. The diagram also depicts the risk–reward (opportunity) trade-off for various stages of a project. Managers can use this information to determine if risk at a certain point is worth the potential payoffs.

7.4 What are the benefits and drawbacks of using the various forms of risk identification mentioned in the chapter (e.g., brainstorming meetings, expert opinion, etc.)? 117 Copyright 2019 © Pearson Education, Inc.


Brainstorming, expert opinion, and multiple assessment approaches share similar benefits and drawbacks. The benefits include a variety of experience and multiple angles of analysis due to different points of view and areas of expertise. Synergy among group members may also create a wider range of risk identification. Although there are shortcomings to these approaches. It may be difficult to come to an agreement or consensus in such processes due to conflicting opinions and personal issues. Brainstorming, expert opinion (when performed in a forum setting rather than through the Delphi approach) and multiple assessments may also have difficulty reaching consensus due to egos/authority issues and functional biases (when members of different departments are present). When the Delphi technique is employed, many of these issues are resolved because members do not meet face to face. The process becomes anonymous reducing interference of egos and personal problems. However, the Delphi approach can take considerable time to complete successfully. Past history is a unique approach to the other three. It uses historical facts to reach conclusions, which removes many obstacles that the other techniques encounter. The benefits of risk identification through past history is that, if past projects were well documented, information related to similar projects can easily be found and project teams can avoid pitfalls by learning from the mistakes of others. Unfortunately, the drawback is that past performance does not always predict future performance. Project-specific data (i.e., competitive environment, economy, etc.) is unaccounted for. Thus, past history may do little to forecast present risks.

7.5 What are the benefits and drawbacks of using a qualitative risk impact matrix for classifying the types of project risk? Qualitative risk assessment matrix is beneficial in providing a visual depiction of potential risk factors. The matrix enables the project team to prioritize risk based on severity of consequences and likelihood of occurrence. For instance, those that rest in the “high” portion of both consequences and likelihood would be top priorities during project planning. Drawbacks of the matrix may revolve around differences in opinion as to where risk variables should be placed on the matrix. It may also create tunnel vision where the team fails to acknowledge the significance of tasks that fall outside of the highpriority areas.

7.6 What are the benefits and drawbacks of using a quantitative risk assessment tool such as the one shown in the chapter? One benefit of such tools is in the ability to set thresholds based on calculations of probability and consequence. A numerical point system creates an easy way to compare different risk variables. This point system also provides more detail than a matrix in depicting the level of risk. Additionally, once the point system is designed, it can be used over and over to compare risk factors of future projects. Problems with these tools may arise in disagreement over assigning points and creating thresholds. Also, the point 118 Copyright 2019 © Pearson Education, Inc.


system is not an exact science. It relies on rules of thumb and may be subject to interpretation.

7.7 Give some examples of projects using each of the risk mitigation strategies (accept, minimize, share, or transfer). How successful were these strategies? In hindsight, would another approach have been better? This question requires students to do some online research, investigating projects and analyzing them in terms of various risk mitigation approaches. One suggestion is to assign a specific project, such as the new Airbus A380 and have the students research it, address relevant risks factors, and the mitigation strategies the company employed.

7.8 Explain the difference between managerial contingency and task contingency. Contingency reserves are provisions set aside in case of unforeseen problems. The primary differences between task and managerial contingency are that managerial contingency is applied at the project level while task contingency is applied at the individual task or work package level. Managerial contingencies are budgeting buffers that teams can fall back on in case of natural disasters, severe divergence from original process or technical plans or other “acts of God.” Task contingencies are established because estimates (made during initial planning) for individual tasks may be unreliable. Reserves for these contingencies may be adjusted as the project advances and estimates become more accurate.

7.9 What are the advantages of developing and using a systematic risk management approach such as the PRAM methodology? Do you perceive any disadvantages of the approach? PRAMs and other similar systems are advantageous because they provide a detailed stepby-step approach to risk management. This helps project teams work through each part of risk management effectively before moving on to subsequent tasks. This way the team does not get stalled at one stage or leave out a vital step altogether. PRAMs adds an additional feature—the feedback loop—that acts as a built-in safeguard to overlooking risk associated with project changes. The loop also keeps varying levels of managers and team members up-to-date on project and risk adjustments. The disadvantage to the PRAMs model is that it may be too involved for low-budget or otherwise small projects. The amount of time required to learn and institute the steps may be unreasonable in a short-term, low-budget initiative. Also, it may not be feasible for smaller companies who do not have the resources or time to devote to such a system. 7.10. Consider the following observation: “The problem with risk analysis is that it is possible to imagine virtually anything going wrong on a project. Where do you draw 119 Copyright 2019 © Pearson Education, Inc.


the line? In other words, how far do you take risk analysis before it becomes overkill?” How would you respond? The tools in this chapter address this very problem. Looking for possible risks could be an endless process; however, it is important (as outlined in the text) to approach risk in a systematic way. The early stages of risk management may best answer this question. The prioritization of risks makes the process manageable, and when done properly, will prevent a situation of overkill. After potential risks have been identified (using a supervised, controlled method such as brainstorming, expert opinion, etc.), managers can use qualitative and quantitative tools to sift through scores of risks to identify what is probable enough to worry about. Once vital risk variables have been identified, then mitigation and contingencies can be established. This process does not ensure that all risks will be identified or that the right contingencies will be created. All the same, the benefits of taking part in risk management far outweigh the danger of not preparing for potential problems.

CASE STUDIES Case Study 7.1 Classic Case: de Havilland’s Falling Comet The de Havilland story is a fascinating example of a well-respected organization that sought to be first to market with a radical new technology and cut some important safety corners, with disastrous results. The story highlights the problems when innovations in design are pushed too far, too quickly. With its unique design and all the additional features that made it radical, the Comet should have been slowly integrated into production and use, instead of being rushed to market. De Havilland knew that Boeing was at work on its own design, the 707, and felt the need to be first to market. In this rush, they cut a number of safety corners with disastrous results. Questions 1. How could risk management have aided in the development of the Comet? De Havilland was producing an aircraft that was so revolutionary in so many ways that they may have become overawed by the push in technology for its own sake. Certainly, the original Comet included several radical design elements (embedded engines in the wing root, square windows, pressurized cabin, and so forth) that any one of them could have been a significant advance on its own. Putting them all together into the same new aircraft design without adequate testing was a disaster. The question of how much testing is enough is difficult to answer but certainly, with so many innovations in one design, it is clear that they did not engage in sufficient risk assessment and design testing. 2. Discuss the various types of risk (technical, financial, commercial, etc.) in relation to the Comet. Develop a qualitative risk matrix for these risk factors and assess them in terms of probability and consequences. 120 Copyright 2019 © Pearson Education, Inc.


This question asks students to identify the wide variety of risks that were present in this aircraft. Commercially, de Havilland had a huge investment in its success and perceived that first-mover advantage would allow them to pick up a big piece of the commercial jet aircraft market if they were first off the mark. Technically, the aircraft had so many new and radical features that several of them could have been perceived as risky in their own right. All of them together were simply too much risk, too fast. Although in might be difficult for students to reasonably comment on probability of failure, there is no question that consequences would be catastrophic (as they were). 3. Given that a modified version of the Comet (the Comet IV) was used until recently by the British government as an antisubmarine warfare aircraft, it is clear that the design flaws could have been corrected, given enough time. What, then, do you see as de Havilland’s critical error in the development of the Comet? Over time, the fundamental aircraft design (minus the original square windows) has been proven to be a success, though the company never again attempted to launch it in the commercial jet market. De Havilland tried to do too much too fast and created an unsafe design due to inadequate testing. Students are quick to recognize this basic error on de Havilland’s part. It is often instructive to note that this foray into the commercial jet aircraft market was de Havilland’s last and they never returned to the level of technical or commercial success they had enjoyed during and just after World War II. 4. Comment on this statement: “Failure is the price we pay for technological advancement.” This question can generate several opinions from students as evidence demonstrates again and again that organizations continue to push the edges of the technological envelope with new designs for buildings, aircraft, automobiles, and so on. The issue that we have to address is just how far one should be allowed to push this envelope and at what point does some system of controls come into play. Clearly, the greater the consequences of failure, the greater the oversight needed to ensure that sufficient risk analysis and management has taken place prior to opening Pandora’s Box.

Case Study 7.2 The Building that Melted Cars This modern example of poor project management and ineffective technical risk assessment has a number of important lessons. First, innovative techniques and designs in construction may not take into consideration potential side effects that can pose dangers to those in proximity to the project. The case also reveals that many times, project risks are impossible to fully identify in advance; sometimes it is only in the development of the project that its downsides become apparent. This point is important for students to understand because they must be aware that not every risk is foreseeable. The case is also “fun” in that it offers a unique twist on a project type that may not obviously lend itself to risks of this sort. 121 Copyright 2019 © Pearson Education, Inc.


Questions: 1 Search “London Walkie-Talkie Building” on the internet, then click through pictures of the structure and read some of the articles posted. For example, note that the building won the “Carbuncle Cup” by the Building Design Magazine for “Worst Building of the Year” in 2015. What are some of the reasons the building has been so ridiculed? Google searches of the Walkie-Talkie building are interesting because the odd shape of the structure is immediately obvious. The aesthetics of the design are arresting but not necessarily pleasing. Coupled with the rather odd design are the unintended consequences of having the solar radiation from the concave windows concentrating sunlight to dangerous temperatures. All in all, the design has caused a lot of controversy and the problems continue to become apparent as it is in use.

2

What are some of the challenges with assessing risk when constructing a building? In other words, what risks can be assessed up front, and what risks are examples of “unknown-unknowns”?

When construction projects follow well-understood guidelines or use designs and building techniques that are simply modifications of existing and understood technologies, the risks tend to be much less intrusive and manageable. On the other hand, the more a building’s design or building methods push out the boundaries of the innovative envelope, it is necessary to start asking “what if?” questions and developing a database of similar projects to determine likely problems. For example, in this case, the building’s designer had already had a similar problem occur with a structure he designed in Las Vegas, suggesting that there were ways to determine likely outcomes of adopting similar design features.

3

Consider the statement: “With construction, risk is simply the residue of creative designs. You cannot account for it in your plans.” Pick either the “pro” or the “con” side of this debate and develop an argument supporting your position. Is risk a natural result of creativity?

This question is intended to get a discussion going in class, based on the experiences with the construction of the building at 20 Fenchurch Street in London. Clearly, arguments can be made in either direction. It is true that unique structures (like the Sydney Opera House) result in unique and, often, unforeseeable technical difficulties. On the other hand, a counter argument can be made that perhaps a large building in the middle of London’s financial district is not the best place to experiment with modern design, especially when the solar results of curvilinear glass has been documented from other structures.

122 Copyright 2019 © Pearson Education, Inc.


Case Study 7.3 Classic Case: Tacoma Narrows Suspension Bridge The famous story of the bridge that shook itself to pieces is familiar to many students. The background of the story is not as well-known and illustrates what happens when we attempt to be too innovative without recognizing the full implications of our choices. The Tacoma Narrows Bridge (TNB) was built at a sight with numerous design challenges and difficulties, including physical location, size and length of the bridge, use of nonoptimal materials, and so forth. From a risk analysis perspective, the TNB represents another example of journeying too far into the unknown and only belatedly recognizing the implications of major design decisions. Questions 1. In what ways were the project’s planning and scope management appropriate? When did the planners begin taking unknowing or unnecessary risks? Discuss the issue of project constraints and other unique aspects of the bridge in the risk management process. Were these issues taken into consideration? Why or why not? It is easy, in hindsight, for students to criticize several elements in the TNB development and construction; however, it is important that instructors not allow them to take the easy way out and focus on the result of the construction. Rather, it is useful to examine how the development of this bridge actually led to the understanding of an entirely new field of engineering—aerodynamics. It was thought, up to that point, that bridge-building was simply a static engineering problem, concerned with downward loads. It took this failure for engineers and scientists to understand the implications of entirely new forces. In developing the answers to this question, it is useful to consider all the constraints (known and discovered) that they were dealing with at the sight and with the design they had. In those circumstances, ask student what they did correctly and where they ignored ample warning signs of problems to come. 2. Conduct either a qualitative or quantitative risk assessment on this project. Identify the risk factors that you consider most important for the suspension bridge construction. How would you assess the riskiness of this project? Why?

This exercise allows faculty to tease out the various risk factors that the TNB project encountered. After listing them on the board, develop a qualitative risk matrix and ask students to help classify the various risk elements. Ask the question: Why was so little risk analysis done at the time? The answer is that they did not recognize the risk in the design, sighting of the bridge, or construction until after it had been constructed.

123 Copyright 2019 © Pearson Education, Inc.


3. What forms of risk mitigation would you consider appropriate for this project? Instructors can discuss various risk mitigation strategies in light of the TNB example (share it, transfer it, etc.). It is useful to ask how each mitigation strategy might be used, who it would be used with (which stakeholder parties), and how effective each strategy might be.

PROBLEMS 7.11. Assessing Risk Factors. Consider the planned construction of a new office building in downtown Houston at a time when office space is in surplus demand (more office space than users). Construct a risk analysis that examines the various forms of risk (technical, commercial, financial, etc.) related to the creation of this office building. How would your analysis change if office space were in high demand? Solution: This question can be answered by students in a number of ways. The key point is to get them thinking in terms of potential risks that are bound to exist prior to initiating a new project. Commercial risk is paramount here because office space is in low demand, making any new office building project questionable from a financial perspective.

7.12. Qualitative Risk Assessment. Imagine that you are a member of a project team that has been charged with developing a new product for the residential building industry. Using a qualitative risk analysis matrix, develop a risk assessment for a project based on the following information: : Identified risk factors 1. 2. 3. 4. 5.

Key team members pulled off project Chance of economic downturn Project funding cut Project scope changes Poor spec. performance

Likelihood 1. 2. 3. 4. 5.

High Low Medium High Low

Based on this information, how would you rate the consequences of each of the identified risk factors? Why? Construct the risk matrix and classify each of the risk factors in the matrix. Solution: Student can draw a simple 3×3 risk matrix with Probability and Consequences as the two axes. Depending upon how they view the consequences of each of the above risks, it is 124 Copyright 2019 © Pearson Education, Inc.


possible to classify them into one of the quadrants of the qualitative risk matrix. The key is that students justify their classification by giving a logical reason for the consequences they perceive for each risk factor, should the problem actually occur.

7.13. Developing Risk Mitigation Strategies. Develop a preliminary risk mitigation strategy for each of the risk factors identified in Problem 2. If you were to prioritize your efforts, which risk factors would you address first? Why? Solution: Students can construct a risk mitigation approach (accept it, transfer it, etc.) for each of the risk factors identified. They must justify their mitigation strategy on the basis of how severe the risk effect could be, the alternatives, and the reasons why they selected the mitigation strategy they chose.

7.14. Quantitative Risk Assessment. Assume the following information: Probability of Failure

Consequences of Failure

Maturity = .3 Complexity = .3 Dependency = .5

Cost = .1 Schedule = .7 Performance = .5

Calculate the Overall risk factor for this project. Would you assess this level of risk as low, moderate, or high? Why? Solution: Using the formula from the chapter, the solution to this problem is: Pf = (.3 + .3 + .5)/3 = .37 Cf = (.1 + .7 + .5)/3 = .43 Risk factor = .37 + .43 − (.37 × .43) = .6409, or .64 According to the severity levels, this would be classified as medium risk.

7.15 Quantitative Risk Assessment. Assume the following information for an IT project. Probability of Failure Maturity = .7 Complexity = .7 Dependency = .5

Consequences of Failure Cost = .9 Schedule = .7 Performance = .3 125

Copyright 2019 © Pearson Education, Inc.


Client Concerns = .5 Programmer Skill = .3

Future Business = .5

Calculate the overall risk factor for this project. Would you assess this level of risk as low, moderate, or high? Why? Solution: Using the formula from the chapter, the solution to this problem is: Pf = (.7 + .7 + .5 + .5 + .3)/5 = .54 Cf = (.9 + .7 + .3 + .5)/4 = .6 Risk factor = .54 + .6 – (.54 × .6) = .82. According to the severity levels, this would be classified as a high-risk project.

7.16. Developing Risk Mitigation Strategies. Assume that you are a project team member for a highly complex project based on a new technology that has never been directly proven in the marketplace. Further, you require the services of several subcontractors to complete the design and development of this project. Because you are facing severe penalties in the event the project is late to market, your boss has asked you and your project team to develop risk mitigation strategies to minimize your company’s exposure. Discuss the types of risk that you are likely to encounter. How should your company deal with them (accept them, share them, transfer them, or minimize them)? Justify your answers. Solution: This question is not intended to elicit a specific answer, but to make students consider all potential issues that could arise and begin to formulate strategies for mitigating those risks. The more interesting discussion typically emerges around the question of which mitigation strategies are best for different risks. Some creative thinking on the part of students can often identify alternatives to standard approaches for risk mitigation, although it is also useful to have them recognize that for certain low-impact risks, simply accepting them is often the best (least expensive and fastest) solution.

7.17. Assessing Risk and Benefits. Suppose you are a member of a project team that is evaluating the bids of potential contractors for developing some subassemblies for your project. Your boss makes it clear that any successful bid must demonstrate a balance between risk and price. Explain how this is so; specifically, why are price and risk seen as equally important but opposite issues in determining the winner of the contract? Is a lowprice/high-risk bid acceptable? Is a high-price/low-risk bid acceptable? Why or why not?

126 Copyright 2019 © Pearson Education, Inc.


Solution: The issue of price and risk is critical for understanding the thought process that often goes on with clients when deciding which contractor to award the project. At times, a higher price bid can win a contract, provided that the client recognizes a lower risk level with a certain contractor. For example, in cases where an older, well-established firm bids a contract, they may offer a higher bid but it is attractive because of their obvious project management expertise and lower concomitant risk. On the other hand, an untested or problematic contractor’s low bid may be refused because they bring unacceptable levels of risk to the project. Price and risk thus serve as the weights on two opposite scale pans. Too much risk requires exceptionally lower prices, whereas a higher bid can be offset with significant reductions in risk.

127 Copyright 2019 © Pearson Education, Inc.


CHAPTER FIVE

Scope Management Chapter Outline PROJECT PROFILE Berlin’s Brandenburg Willy Brandt International Airport INTRODUCTION: THE IMPORTANCE OF SCOPE MANAGEMENT 5.1 CONCEPTUAL DEVELOPMENT The Statement of Work The Project Charter PROJECT PROFILE Statements of Work: Then and Now 5.2 THE SCOPE STATEMENT The Work Breakdown Structure Purposes of the Work Breakdown Structure The Organization Breakdown Structure (OBS) The Responsibility Assignment Matrix PROJECT PROFILE Defining a Project Work Package 5.4 WORK AUTHORIZATION Scope Reporting PROJECT MANAGEMENT RESEARCH IN BRIEF Information Technology (IT) Project “Death Marches”: What Is Happening Here? CONTROL SYSTEMS Configuration Management PROJECT CLOSEOUT Summary Key Terms Discussion Questions Problems Case Study 5.1 Boeing’s Virtual Fence Case Study 5.2 California’s High-Speed Rail Project Case Study 5.3 Project Management at Dotcom.com Case Study 5.4 The Expeditionary Fighting Vehicle Internet Exercises PMP Certification Sample Questions MS Project Exercises Appendix 5.1 Sample Project Charter Integrated Project––Developing the Work Breakdown Structure Notes

79 Copyright 2019 Pearson Education, Inc.


TRANSPARENCIES Note for Instructors: To present transparencies in class, please download the PowerPoint Presentations (available on www.pearsonhighered.com) that accompanies this product. The PPT ISBN is 9780134730479.

5.1 ELEMENTS IN PROJECT SCOPE MANAGEMENT 1. Conceptual Development  Problem statement  Requirements gathering  Information gathering  Constraints  Alternative analysis  Project objectives  Business case  Statement of Work  Project charter 2. Scope Statement  Goal criteria  Management plan  Work Breakdown Structure  Scope baseline  Responsibility Assignment Matrix 3. Work Authorization  Contractual requirements  Valid consideration  Contracted terms 4. Scope Reporting  Cost, schedule, technical performance status  S-curves  Earned value  Variance or exception reports 5. Control Systems  Configuration control  Design control  Trend monitoring  Document control  Acquisition control  Specification control 80 Copyright 2019 Pearson Education, Inc.


6. Project Closeout  Historical records  Postproject analysis  Financial closeout

5.2 PURPOSES OF THE WORK BREAKDOWN STRUCTURE  IT ECHOES PROJECT OBJECTIVES.  IT IS THE ORGANIZATION CHART FOR THE PROJECT.  IT CREATES THE LOGIC FOR TRACKING COSTS, SCHEDULE, AND PERFORMANCE SPECIFICATIONS FOR EACH ELEMENT IN THE PROJECT.  IT MAY BE USED TO COMMUNICATE PROJECT STATUS.  IT MAY BE USED TO IMPROVE OVERALL PROJECT COMMUNICATION.  IT DEMONSTRATES HOW THE PROJECT WILL BE CONTROLLED.

5.3 PARTIAL WORK BREAKDOWN STRUCTURE

1.0

1.2

1.3

1.4

1.2.1

1.3.1

1.4.1

1.2.2

1.3.2

1.4.2

1.2.3

1.5

1.4.3

81 Copyright 2019 Pearson Education, Inc.


5.4 THE INTERSECTION OF THE WBS AND OBS 1.0

IT Installation 1.3

Prepare proposal

Project

1.4

1.5

Seek & hire IT consultant

Seek support for IT

1.4.2

1.4.3

1.4.1

Search committee

Develop criteria

Select consultant

Information Systems

Cost Account

Cost Account

Cost Account

Human Resources

Cost Account

Cost Account

Deliverables

Work Packages

Departments

Procurement

Cost Account

82 Copyright 2019 Pearson Education, Inc.


5.5 COST ACCOUNT ROLLUP USING OBS 1.0

IT Installation 1.3

Prepare proposal

1.4

1.5

Seek & hire IT consultant

Seek support for IT

1.4.2

1.4.3

1.4.1

Search committee

Project

Develop criteria

Select consultant

Information Systems

$500

$1,000

Human Resources

$500

Procurement

$500

Deliverables

Work Packages

Departments

Totals

-0-

$1,500

$1,000

83 Copyright 2019 Pearson Education, Inc.


5.6 RESPONSIBILITY ASSIGNMENT MATRIX

DISCUSSION QUESTIONS 5.1 What are the principal benefits of developing a comprehensive project scope analysis? A comprehensive project scope analysis serves several benefits that permit a company to guide the dream of a project to a successful completion. One benefit is transforming the idea of the project into a working concept. Once the concept is developed, project details can be mapped out. Scope analysis outlines each step of the project in a detailed manner. The primary benefit to this is it aligns the project so that the final product will be consisted with the original goals/objectives. By outlining each step, scope analysis increases the likelihood that the project will stay within budget and time constraints as well as meet predetermined specifications and quality parameters.

84 Copyright 2019 Pearson Education, Inc.


5.2 What are the key characteristics of a work package? A work package is a specific step of the project. Work packages are smaller pieces of the whole project, which are more manageable and definable than the whole. They create a chronological map from one step to the next detailing what needs to be completed at each stage. Each work package includes its own deadline, resource requirements, and supervisor. Work packages can be further broken down into subtasks that can be assigned to an individual worker or group. Collectively, all the work packages of a project combined lead the project team from the start to completion of the project.

5.3 Create a Work Breakdown Structure for a term paper project or another schoolrelated project you are working on. What are the steps in the WBS? Can you identify any sub-steps for each step? This assignment is designed to get students comfortable with deconstructing the project into various levels, including Deliverables, and Work Packages. The goal is identifying the relevant elements in the project, not the sequential nature of those steps. If instructors wish, they can ask students, as a second step, to create some idea of the sequentiality of these WBS elements.

5.4 What are the benefits of developing a Responsibility Assignment Matrix (RAM) for a project? An RAM outlines the team members directly responsible for each task. It also includes a list of supporting organizational members. It is beneficial for following chain of command, solving interdepartmental issues and receiving proper approval. Additionally, an RAM increases the flow of communication throughout the team. By knowing who is in charge of what, members can notify the necessary people of progress or completion of tasks. This keeps the group informed of potential problems and up-to-date on the project’s current status. The process of creating the RAM is also beneficial to the project manager. It requires the identification of team members’ abilities, qualifications, strengths and weaknesses. Tasks and assignments can then be better coordinated to create the highest level of efficiency.

5.5 Develop an argument for scope reporting mechanisms. At a minimum, what types of reports do you consider necessary for document control of a project? Why? Two main concerns of project scope are seeing that the project meets time and budget constraints. Without some sort of limits (i.e., project scope), projects can quickly become expensive, long-term investments of a company’s time and resources. Scope reporting can help reduce the risk of such runaway projects. Reports that update on costs incurred 85 Copyright 2019 Pearson Education, Inc.


using S-curves and variance updates are important for keeping the project within budget constraints. These reports may also serve as control mechanisms. If team members know they will have to publish spending reports frequently to other members in the organization, they may be less likely to approve unscheduled expenditures. Reports related to adherence to the planned time schedule work in a similar fashion. If the project begins to lag behind, team members may have more incentive to look for the cause of the delays if they are required to report variances from the planned timetable.

5.6 What is the chief purpose of configuration management? In your opinion, why has it become increasingly popular in recent years as a part of the project management process? The main purpose of configuration management is to manage and control change within projects. A plan for execution of activities and tasks is agreed upon by those involved in the project. Configuration management then monitors variations from this preestablished plan. Change is an anticipated factor of project management. Configuration management provides a way to deal with changes as they arise, so they do not cause more disruption than necessary. Changes are documented so that all involved in the project may be notified and modifications can be made universally. By identifying necessary changes as early as possible and handling the problem early on, configuration management decreases time delays related to unforeseen conditions and helps reduce obstacles that may result from change later on in the project process.

5.7 What is the logic behind developing a plan for project closeout prior to even beginning the project? Upon completion of the project, certain documentation will be required by management and/or the client involved. The documentation may be used for legal purposes, as training material or in auditing procedures. Therefore, it is important that the closeout information be thorough and accurate. Creating a plan early on is important because proper documentation may need to be performed as the project progresses. By knowing what information will be required at the end of the project, the team can keep proper records at various stages as the project advances. Trying to create this documentation after the fact may be difficult or inaccurate. 5.8 Sustainable project development has many features and can mean many things when beginning a project. If you were to identify three critical messages that explain what sustainable project management really means, what would they be? Sustainability in projects influences project management in a variety of ways. Simply put, four critical issues emerge. First, we are committed to managing sustainable projects; that is, the projects we undertake should cause no harm to the planet or its inhabitants. Second, we should employ sustainable practices when undertaking the projects themselves. See the Project Profile on Nicaragua’s canal to see how there are serious dangers caused by poor project management sustainability. Third, we should 86 Copyright 2019 Pearson Education, Inc.


develop sustainable supplier practices by influencing their behaviors. Fourth, it is important to emphasize sustainability in project design. Students can adapt any three of these four principles when answering this question.

CASE STUDIES Case Study 5.1 Boeing’s Virtual Fence The Virtual Fence project, cancelled in early 2011 by the Federal Government, is a true story of a project that sounded better than it could ever be due to technical problems, unreasonable or overly optimistic projections, and a general lack of understanding of the sheer size of the effort that would be required. Boeing was also given carte blanche to run the project, with governmental oversight notably absent from the decisions regarding vendor selection, technical requirements, and first phase development. In short, the Federal Government signed a blank check to Boeing for initial development and then discovered that “real” costs were projected to be much higher. The project was cancelled when Boeing began failing technical capability tests on initial sections of the virtual fence. Questions 1. What problems do you see emerging from a project such as SBInet where the government allows the contractor to determine scope, manage all contractor relations, and decide how to share project status information with oversight bodies? Oversight, particularly early in the project’s development, is critical. It is during the early phases of the project that most technical decisions are made, that overall project scope is frozen, and that project capabilities and standards for evaluation are determined. Thus, when the government gave this power to the contractor without sufficient checks and balances, they provided the opportunity for overruns, poor technical quality, and loss of accountability. In retrospect, it is actually surprising that the project was cancelled as early as it was. Under similar circumstances, federally-funded projects have been much bigger “black holes” for budget and schedule overruns. 2. Consider the following two arguments: “The failure of SBInet is due to poor scope management,” versus “SBInet failed because of poor oversight and project controls.” Take one side or the other in this argument, and justify your response. A case can be made for either position in this example. The project was poorly scoped, mainly due to an inadequate understanding of the technical requirements for developing and maintaining a virtual fence across a long border. Even the pilot project on the 28mile test section of the border was not technically sound and demonstrated a number of flaws with the system. On the other hand, there was a continued lack of oversight of the 87 Copyright 2019 Pearson Education, Inc.


project through much of the early qualification phases, when Boeing was vetting suppliers, trying to determine the project’s “real” scope, and creating a management structure that seemed designed to promote overruns and poor quality. 3. With the current publicity surrounding the intentions of the Trump administration to begin building a wall along the border with Mexico, identify three lessons from this case that could help with that development process. In other words, if you were to advise the Trump White House on major lessons to absorb as it considers beginning its own fence, what would they be? Students can address this question from several perspectives. The most important point for instructors to note is that this is NOT intended to spark political discussions, opposition to such a wall, etc. Rather, the question requires students to consider the main failures of the SBInet project and make concrete suggestions about how to improve on this failure. Among the answers would be better supplier/contractor management procedures, better scope management (making a more detailed project scope), and so forth. As there are no obvious right or wrong answers here, the primary goal is to encourage students to critically evaluate this project failure and adapt a new approach for the potential border fence project. Case Study 5.2: California’s High-Speed Rail Project A goal of the Obama administration has been to promote high-speed rail across the most populous and geographically-dispersed states in the United States. The idea is to adopt more energy-saving initiatives while also helping to improve state’s infrastructure. It is with this in mind that the Federal Government made billions available to various states in the 2008–2010 budget cycles. After the Fall, 2010 elections, several states that had elected Republican governors refused the grants, suspicious that this seed money would not be sufficient to pay for what they viewed as unnecessary construction based on overoptimistic expectations of the need for and use of high-speed rail. One of the states that accepted the money and has moved forward strongly into high-speed rail has been California, which has already begun work on a 65-mile section in the middle of the state, earning the derisive nickname, the “train to nowhere.” This case details the state’s projections regarding the need for high-speed rail, against the views of infrastructure experts and critics who charge that for a state that is already in a severe budget crisis, this is just the sort of project that makes no sense economically or demographically. Questions 1. Assess the benefits and drawbacks of the high-speed rail project. In your opinion, do benefits outweigh drawbacks, or vice versa? Why? Justify your answer. This question asks the students to take a position either pro or con to the decision to promote high-speed rail service in California. It requires them to consider issues such as the current state of the budget, projected passenger usage rates and actual necessity of 88 Copyright 2019 Pearson Education, Inc.


linking the state’s major cities in this way. Advocates can point to the short-term economic benefits of construction work on the system and the potential for creating more energy-efficient ways to travel from city to city. Opponents can argue that seed money still leaves the already-strapped state on the hook for large future costs as the project continues to be developed. Also, they can point to the obvious responses from airlines covering these routes, which will be to reduce their fares to be competitive with the highspeed rail system. 2. What are the implications of starting a project based on tenuous projections that may or may not come true 10 years from now? The obvious implications are to commit to the project and then develop an “escalation of commitment” mentality that argues since we have already gotten deep into this project, we must now continue work on it regardless of future costs or considerations. Economic circumstances, demographics, and energy issues are all relatively ephemeral and the decision to initiate this project may or may not pay off down the road. That is a major gamble to make for a state that is currently running a $35+ billion deficit. 3. Could you justify the California high-speed rail project from the perspective of a massive public works initiative? In other words, what other factors enter into the decision of whether to pursue a high-speed rail project? Why are they important? Certainly, with the 2011 economy still showing signs of heavy slumping and large unemployment rates, some can make the argument that the California High-Speed Rail Initiative is simply a modern version of the CCC programs that created projects such as the Hoover Dam. The problem is that modern economics suggests the multiplier effect of spending on public works does not necessarily have long-term positive effects on the economy and if this is really a public-works initiative, why not spend the money on refurbishing the thousands of bridges and roads that are decaying at alarming rates around the country?

Case Study 5.3: Project Management at Dotcom.com This case is based on a true story and illustrates some of the key challenges that IT organizations face when they attempt to develop solutions for clients. Many of these clients understand their problems but don’t see how to create an appropriate solution. Others, however, think they understand their needs but find the solutions generated for them to be inadequate or simply addressing the wrong issues. It is a classic story of cope definition that many IT organizations routinely deal with when trying to satisfy the needs of clients. Questions 1. How would you begin redesigning Dotcom.com’s project management processes to minimize the problems they are experiencing with poor scope management? 89 Copyright 2019 Pearson Education, Inc.


One suggestion might be the inclusion on the project team of a representative of the client. It appears that one key problem lies in the fact that project teams talk to the customer and then go away to develop the solution in a vacuum. Then, at the back end of the development cycle, the customer is presented with a project solution that many times does not satisfy them. This lack of communication is a key factor in the problems the company is experiencing with their projects. 2. How do the company’s consulting clients contribute to the problems with “scope creep?” If you were to hold a meeting with a potential customer, what message would you want them to clearly understand? Customers often do not recognize the link between project changes and cost. They may assume that the project contract gives them unlimited rights to suggest or demand any changes after the project was developed, or that the IT solution presented to them is only a working model and subject to modifications. Without good communication between the project organization and their clients, these misunderstandings will escalate as the consulting firm demands more money for the changes and the clients argue that they will not pay for modifications because the project firm did not do it right the first time. “Why should we pay for your mistakes?” is a comment commonly expressed by customers. The key message that the customers must understand is the need to maintain strong lines of communication and develop some milestones that allow for “reality checks” throughout the development cycle. There should be no surprises at the time of delivery. 3. How do you balance the need to involve clients with the equally important need to freeze project scope in order to complete the project in a timely fashion? This may be the key conundrum in managing IT projects and is a great point of departure for in-class discussion. Students are usually quick to point the finger of blame; either at the IT firm for not getting clear information or at the customer for not providing it. Instructors can set up a “point/counter-point” discussion on the causes of IT project failure and how many are linked precisely to this problem and the failure to resolve it as early in the project contract as possible. 4. Why are configuration management and project change control so difficult to perform in the midst of a complex software development project such as those undertaken by Dotcom.com? One of the keys to configuration management is communication between customer and client as both parties observe early versions of the system and identify the needed modifications. The problem for IT organizations is that they are loath to grant too much power to modify the system in mid-development for fear that the initially contracted terms will become meaningless through multiple changes orders. They desire spec freeze precisely for the reason that customers hate it. Hence, configuration management is difficult because it is hard for customers, who are not sophisticated with IT technologies, to identify relevant points when configuration management should occur. Their lack of 90 Copyright 2019 Pearson Education, Inc.


understanding of the development process often makes it hard to create good a priori change controls processes. Instead, they usually simply react (negatively) to perceived inadequacies of the solution, after installation. Case Study 5.4: The Expeditionary Fighting Vehicle This case is an interesting example of how Pentagon procurement policies, misguided strategies, and poor project execution can combine to spend enormous money on a weapon system that is not only not needed, but also does not work reliably. The Expeditionary Fighting Vehicle (EFV) was originally designed for storming beaches, despite the fact that modern warfare calls into serious question the need for such a vehicle, since the last time the U.S. Marines were called on the invade a hostile shoreline was 1951! Further, the system, filled with the most sophisticated technology, has been hampered by all this “gold plating” to the point where it cannot perform any of its functions and is notoriously unreliable. When the project was finally cancelled in 2011, it had cost nearly $3 billion in sunk (nonrecoverable) costs. The case is great for students because there is a wealth of information on the internet about the EFV, including some impressive (but staged) field tests and live fire examples. Questions 1. What does the story of the EFV suggest about the importance of considering what a project’s key mission is supposed to be prior to authorizing it? The EFV demonstrates a variety of mistakes in conception, identification of the project’s key mission, and ultimately, a mistake in scope management. Without a clear sense of what the EFV was expected to do, it quickly became loaded up with extra features that were not well integrated, leading to numerous failures during testing. The EFV bears out the old idea that “simpler is better.” 2. The EFV has been labeled, “The wrong weapon for the wrong war at the wrong time.” Do you agree or disagree with this characterization? Why? This question allows students to consider what mission(s) the EFV was developed to accomplish. Is a fully amphibious armored vehicle needed for the modern Marine Corps? Some would argue that it is, while others are not so sure. The question to be asked here is did the proposed uses for the EFV justify the expense for it. Is a mission of the modern Marine Corps to storm enemy beaches? Some would argue that it is impossible to know, so wouldn’t it be better to have the system in case it is needed. Others would suggest that this idea ignores the nature of modern warfare. Regardless of the position of the students, it is a debatable point. 1) 3. Why does the EFV failure illustrate the dangers of long lead-times for weapon systems? In other words, when a project’s development cycle takes 20 years from start to finish, what dangers do the project developers face when the project is finally operational? 91 Copyright 2019 Pearson Education, Inc.


One of the complaints about EFV development and Pentagon procurement cycles is that they ignore the “now” for the “then;” in other words, the argument that generals are always planning for the last war. In some ways, this is unfair but it illustrates the challenges when it may take over a decade (or longer) to identify, procurement and finance a new weapon system. Look at the B-52, which is still in operation after decades of use, or the A-10 attack aircraft, a veteran of Vietnam. There is no question that long development cycles critically affect the viability of projects like the EFV, leading to soaring costs and long schedule delays.

MS PROJECT EXERCISES Using the information provided below, construct a simple WBS table for the project example. Project Outline—Remodeling an Appliance I Research Phase II Design and Engineering Phase III Testing Phase IV Manufacturing Phase V Sales Phase I

II

III IV V

Research Phase A. Prepare product development proposal 1) Conduct competitive analysis 2) Review field sales reports 3) Conduct technological capabilities assessment B. Develop focus group data C. Conduct telephone surveys D. Identify relevant specification improvements Design and Engineering Phase A. Interface with marketing staff B. And so on Testing Phase Manufacturing Phase Sales Phase

Solution: Entering Tasks Using Custom WBS Codes 1. To view WBS codes, display the sheet view 2. Enter each activity on the MS Project task sheet 3. On the Project menu, point to WBS, and then click Define Code (See Figure 5.13) 92 Copyright 2019 Pearson Education, Inc.


Figure 5.13 – WBS Screen Shot

Microsoft product screen shot(s) reprinted with permission from Microsoft Corporation. 4. As you begin to enter each task and identify it as either a first-level, second-level, or third-level heading, the WBS will demonstrate the organization of the project (See Figure 5.14). Your final output should resemble the following: Figure 5.14 – WBS Codes

Microsoft product screen shot(s) reprinted with permission from Microsoft Corporation.

93 Copyright 2019 Pearson Education, Inc.


CHAPTER SIX Project Team Building, Conflict, and Negotiation Chapter Outline PROJECT PROFILE Engineers without Borders: Project Teams Impacting Lives INTRODUCTION 6.1 BUILDING THE PROJECT TEAM Identify Necessary Skill Sets Identify People Who Match the Skills Talk to Potential Team Members and Negotiate with Functional Heads Build in Fallback Positions Assemble the Team 6.2 CHARACTERISTICS OF EFFECTIVE PROJECT TEAMS A Clear Sense of Mission A Productive Interdependency Cohesiveness Trust Enthusiasm Results Orientation 6.3 REASONS WHY TEAMS FAIL Poorly Developed or Unclear Goals Poorly Defined Project Team Roles and Interdependencies Lack of Project Team Motivation Poor Communication Poor Leadership Turnover Among Project Team Members Dysfunctional Behavior 6.4 STAGES IN GROUP DEVELOPMENT Stage One: Forming Stage Two: Storming Stage Three: Norming Stage Four: Performing Stage Five: Adjourning Punctuated Equilibrium 6.5 ACHIEVING CROSS-FUNCTIONAL COOPERATION Superordinate Goals Rules and Procedures Physical Proximity Accessibility Outcomes of Cooperation: Task and Psychosocial Results 6.6 VIRTUAL PROJECT TEAMS PROJECT PROFILE Tele-Immersion Technology Eases the Use of Virtual Teams 6.7 CONFLICT MANAGEMENT What Is Conflict? 94 Copyright 2019 Pearson Education, Inc.


Sources of Conflict Methods for Resolving Conflict 6.8 NEGOTIATION Questions to Ask Prior to the Negotiation Principled Negotiation Invent Options for Mutual Gain Insist on Using Objective Criteria Summary Key Terms Discussion Questions Case Study 6.1 Columbus Instruments Case Study 6.2 The Bean Counter and the Cowboy Case Study 6.3 Johnson & Rogers Software Engineering, Inc. Exercise in Negotiation Internet Exercises PMP Certification Sample Questions Notes

95 Copyright 2019 Pearson Education, Inc.


TRANSPARENCIES Note for Instructors: To present transparencies in class, please download the PowerPoint Presentations (available on www.pearsonhighered.com) that accompanies this product. The PPT ISBN is 9780134730479.

6.1 CHARACTERISTICS OF EFFECTIVE PROJECT TEAMS  A CLEAR SENSE OF PROJECT MISSION  AN UNDERSTANDING OF TEAM INTERDEPENDENCIES  COHESIVENESS  A HIGH LEVEL OF TRUST  A SHARED SENSE OF ENTHUSIASM  A “RESULTS” ORIENTATION

6.2 STAGES IN GROUP DEVELOPMENT Stage

Defining Characteristics

Forming

Members get to know each other and lay the basis for project and team ground rules.

Storming

Conflict begins as team members begin to resist authority, demonstrate hidden agendas and prejudices.

Norming

Members agree on operating procedures, seeking to work together, developing closer relationships, and committing to the project development process.

Performing

Group members work together to accomplish their tasks.

Adjourning

Groups may disband either following the completion of the project or through significant reassignment of team personnel.

96 Copyright 2019 Pearson Education, Inc.


6.3 STAGES IN TEAM DEVELOPMENT

Adjourn

Productive

Testing

Organized

Infighting

du ct iv i Pr o

n

•Quiet •Polite •Guarded •Impersonal •Business-like •High morale

sio clu In

•Trust •Flexible •Supportive •Confident •Efficient •High morale

•Establish procedures •Develop team skills •Confront issues •Rebuilding morale

C

•Conflict over control •Confrontational •Alienation •Personal agendas •Low morale

n tio

ra

pe

oo

3. Norming

1. Forming

ty

4. Performing

Convene

C

o

l ro t n

2. Storming

97 Copyright 2019 Pearson Education, Inc.


6.4 MODEL OF PUNCTUATED EQUILIBRIUM

High

Completion

Team Performance

Eruption First Meeting

Low

Start

Midpoint

Deadline

Project Timeline

98 Copyright 2019 Pearson Education, Inc.


6.5 PROJECT TEAM CROSS-FUNCTIONAL COOPERATION

Superordinate Goals

Task Outcomes

Rules & Procedures

Cross-functional cooperation Physical Proximity

PsychoSocial Outcomes

Accessibility

Feedback Loop

99 Copyright 2019 Pearson Education, Inc.


6.6 REASONS WHY TEAMS FAIL  POORLY DEVELOPED OR UNCLEAR GOALS  POORLY DEFINED TEAM ROLES AND INTERDEPENCENCIES  LACK OF TEAM MOTIVATION  POOR COMMUNICATION  POOR LEADERSHIP  TURNOVER AMONG TEAM MEMBERS  DYSFUNCTIONAL BEHAVIOR

6.7 METHODS FOR RESOLVING CONFLICT  MEDIATE THE CONFLICT  ARBITRATE THE CONFLICT  CONTROL THE CONFLICT  ACCEPT IT  ELIMINATE IT

6.8 STEPS IN PRINCIPLED NEGOTIATION 1. SEPARATE THE PEOPLE FROM THE PROBLEM 2. FOCUS ON INTERESTS, NOT POSITIONS 3. INVENT OPTIONS FOR MUTUAL GAIN 4. INSIST ON USING OBJECTIVE CRITERIA

DISCUSSION QUESTIONS 6.1 This chapter discussed the characteristics of high-performing project teams. List the factors that characterize these teams and give examples of each one. High-performing teams need to have a clear sense of mission, an understanding of the team’s interdependence and be results oriented. They also need to create cohesiveness, trust, and enthusiasm. A clear sense of mission means that all team members understand and accept the purpose of the project. A strong understanding enables team members to be more effective individually (i.e., without the PM) in solving problems. For example, a team member who understands the goals of the project is in a position to provide input about what to do regarding conflicting resources use, whereas a team member who lacks understanding will not be able provide constructive advice. The team’s interdependence represents the capabilities of individual members and their interrelatedness in relation to accomplishing project goals. It also requires members to set aside preconceived notions about other functional departments and to appreciate the strengths of others. Here, a team member who understands the contributions of others is more likely to ask others for advice in brainstorming or when confronted with a problem. Those who do not have a sense of interdependence may attempt to solve the problem alone or be forced to hassle 100 Copyright 2019 Pearson Education, Inc.


the PM with every problem outside their field of expertise. Being results oriented requires rallying all members around the same project goals and motivating them toward completion of those goals. This goes hand-in-hand with enthusiasm, which simply means getting people excited about the project and raising the energy level around project activities. Teams that possess these two attributes are more driven toward project goals and may work at a faster or more intense pace. Cohesiveness is the desire and attraction that team members have for working with one another. Sometimes rewards are used to develop cohesiveness and to generate the desire for members to devote time to the project. Cohesiveness helps create harmony among team members and develop smoother working conditions than teams that do not demonstrate a cohesive front. Finally, trust is evident when team members feel comfortable working together. They have confidence in one another’s abilities. When trust is developed, members are willing to express differences and handle disagreements upfront. 6.2 “Trust can actually encourage disagreement and conflict among team members.” Explain why this could be the case. This is true because trust enables members to express their differences in opinions, understandings, and ideas. They do so in an open environment that accepts differences and allows team members to work out their differences in a civil manner. By doing so, trust allows teams to bring up issues as they arise rather than allow them to build into insurmountable obstacles.

6.3 Identify the stage of group development. Why is it necessary for project teams to move through these stages in order to be productive? Forming is a sort of orientation stage where members are feeling out each other, their roles in the group. They also begin creating the forms of communication and types of behavior that will exist within the group. Storming is filled with conflict stemming from boundary testing. Given the rules established in the forming stage, team members will begin to push the limits of the team’s structure and newly established practices. This is the period during which conflicts should arise and be addressed; otherwise they may rise to the surface later at more inopportune times. Norming occurs after conflicts have been hashed out in the storming stage. At this point, team members have tested the waters (and each other) and have come to a set of agreed upon group standards. Rules of conduct and expectations are established. Trust and cohesiveness also begin to form at this stage as members gain confidence in one another. Performing is where the work actually takes place. The group has been formed and rules are established so the focus is on visible project progress. Trust and confidence exist at high levels and group performance is optimized. 101 Copyright 2019 Pearson Education, Inc.


Adjourning happens when the project is over. Group members may be separated into their previous or new roles. There are many issues (discussed in other chapters) that relate to this process. In order to aid in the process, team members and the project manager need to be aware of adjourning stage and treat it as a serious transition period. Teams need to complete all stages to become productive because the group development process removes many of the barriers to working as a team. With interpersonal issues out of the way and trust and confidence developed, the team can focus on the accomplishing the goals at hand, rather than on the workings of the group. 6.4 Gersick’s model of punctuated equilibrium offers an alternative view of group development. Why does she suggest that some defining moment (such as an explosion of emotion) often occurs at some midpoint in the project? What does this defining event accomplish for the team? Dissatisfaction and frustration stemming from a lack of progress or unclear operating procedures triggers a mid-life project transition. The event causes a major disruption in group dynamic and standards. As a result, the group revises its norms in a way that facilitates better overall group performance. 6.5 Explain the concepts of “task” and “psychosocial” outcomes for a project. Why are psychosocial outcomes so important for project team members? Task outcomes refer to the successful completion of project goals. Psychosocial outcomes relate to the team members evaluation of the entire project experience. These outcomes are important because members will carry past memories into new projects. This will affect members’ attitudes regarding subsequent projects.

6.6 Distinguish between the traditional, behavioral, and interactionist views of team conflict. How might each explain and treat a project team conflict episode? The traditional view sees conflict as negative. Those who prescribe to this line of thinking try to avoid conflict. When it does arrive, they believe in resolving it as swiftly as possible. The behavioral view is that conflict is natural. Under this thinking, conflict is not avoided, but managed. Instead of suppressing, it is allowed to exist in a controlled atmosphere. Interactionists encourage conflict. They believe there is an optimal level of conflict that drives innovation and productivity. Therefore, conflict is allowed to exist unchecked until it surpasses the optimal level.

6.7 Identify the five major methods for resolving conflict. Give an example of how each might be applied in a hypothetical project team conflict episode.

102 Copyright 2019 Pearson Education, Inc.


The five ways to resolve conflict are mediate, arbitrate, control, accept, and eliminate. A project manager who wishes to mediate the conflict will use either defusion or confrontation. Defusion is used to reach a mutual agreement on the subject without delving into the roots of the conflict. Confrontation attempts to get at the underlying causes of the conflict. This takes more time, but gives each party a chance to state their side of the argument. At the end of which, a common ground on the issue is negotiated. Arbitrating is different in that the project manager renders a decision rather than coming to a mutual decision with those involved. The project manager instead listens to both sides and then hands down a decision. For conflicts that cannot be quickly resolved, managers may choose to control the disagreement. To do so, the parties involve may be separated for a short period or their interaction may be restricted for the duration of the project. When nothing can be done to resolve or control the conflict, the team may just have to accept the fact that the conflict is going to exist and do their best to work around it. The last option is to eliminate the problem. This may be the most extreme action as it may require transferring members of the team to other projects or back to their original departments. 6.8 What are some of the guidelines for adopting a strategy of “principled negotiation”? One guideline is to “separate the people from the problem.” People are emotionally attached to their viewpoints and opinions. In order to reduce the amount of influence of personality, ego, and the like, it is important to limit the personal issues of the problem and concentrate on the actual issue creating the differences. Putting aside preconceived notions, being open and understanding, and being empathetic are good steps to successful negotiation. A second guideline is “focus on interests, not positions.” This gets to the heart of the differences. There may be one or two major motivating factors that are fueling the larger conflict. Getting at these fundamental interests allows the negotiators to formulate a solution for each interest that will satisfy the multiple positions rooted in it. The next guideline is “invent options for mutual gain.” It may difficult at first to recognize potential win–win resolutions to a conflict. However, using certain techniques increases the likelihood of arriving at a mutually beneficial solution. Techniques include positive and inclusive brainstorming, which attempts to identify multiple potential outcomes. These outcomes are then broadened through additional discussion. Another technique is to identify shared interests and generate solutions based on areas of common ground. This begins a process of collaboration, which may lead to an agreeable solution. 6.9 Explain the idea that we should “focus on interests, not positions.” Can you think of an example in which you successfully negotiated with someone else using this principle? Focus on interests, not positions means getting to the underlying fears or desires that are at the root cause of conflicting positions. Positions are more on the surface. Interests are the driving force behind them. The rest of this question asks students to apply it in a personal example from their own experience. The key is to ensure that they understand 103 Copyright 2019 Pearson Education, Inc.


the difference between interests and positions in describing the example that serves as their answer.

CASE STUDIES Case Study 6.1: Columbus Instruments This case is based on a true story of a once-successful organization that had allowed its project management practices to degenerate to the point where assignment to a project team was often a mark of disfavor and a sign of pending termination. The case involves issues of motivation, structural effects on projects, and project team staffing. It offers students an opportunity to see how, if left unchecked, certain behaviors by department heads and others in the organization can work counter to the desires to use project teams to improve organizational profitability and instead make them a dumping ground for malcontents and poor performers. Questions 1. What are the implications of CIC’s approach to staffing project teams? Is the company using project teams as training grounds for talented fast-trackers, or as dumping grounds for poor performers? The response to this question is pretty self-evident to students. Clearly, the company has adopted a dumping ground attitude because there is no downside risk to managers who want to off-load poor performers onto project teams. 2. How would you advise the CEO to correct the problem? Where would you start? One huge problem is the lack of responsibility that pervades the organization. Managers see that project teams are a convenient method for discarding marginal employees for extended periods of time. Because there are no consequences to this behavior, it has become increasingly common and is widely used. The first step is to create some accountability whereby functional managers must respond appropriately to resource requests and assign good personnel to these teams. At the same time, these managers must receive credit for doing so. Where fear of failure or lack of authenticity operate, project team staffing will never be taken seriously and this problem will not be resolved. Another solution would be to begin leveling the playing field by giving the project manager authority to select his or her own team rather than depend upon the kindness of the functional manager to staff the team for him or her. 3. Discuss how issues of organizational structure and power played a role in the manner in which project management declined in effectiveness at CIC.

104 Copyright 2019 Pearson Education, Inc.


As noted, authority must be more evenly spread here. This organization operates with a strict functional structure in which the project manager has no power. All staffing assignments are made by the functional manager and there is no negative consequence attached to poor staffing. When the philosophy of “anyone will do, and the worse person, the better!” is in place, it is not hard to see how the projects are fairing poorly. Case Study 6.2: The Bean Counter and the Cowboy A common theme with multifunctional project teams is a lack of appreciation for the duties of people from other departments. The concept of organizational “differentiation” is key to understanding this malady. It is common for functional “siloing” to create an attitude in which the contributions of other project team members are either not recognized or undervalued. In this case, there are some clear signs of antagonism between Neil, the finance person and Susan, from Marketing. The terms “bean counter” and “cowboy,” have been, in fact, coined by these people to refer to members of the other functions. Questions 1. Was the argument today between Neil and Susan the true conflict or a symptom? What evidence do you have to suggest it is merely a symptom of a larger problem? The interaction is evidence of more deep-rooted antagonisms between Neil and Susan and reflects a fundamental lack of appreciation for what each person brings to the project. Not only does each person view their own contributions as important, they also minimize the value added by the other. Neil clearly resents the activities (including trips) that are part of Susan’s job and feels that she is not committed to the project due to frequent absences. Susan is defensive about this, citing the fact that her job does require extended periods where she is likely to miss meetings. 2. Explain how differentiation plays a large role in the problems that exist between Susan and Neil. Differentiation refers to the idea that people in different parts of the organization (functional groups, geographic locations, operating divisions, etc.) develop their own unique attitudes, goals, time frames, and operating philosophies due to cultural and environmental pressures. Susan, as a marketing professional, contributes to the project team but is also responsible and rewarded for sales and customer-facing activities. They create a different mindset than the one that Neil, from finance, faces in his job. Their different attitudes and reward systems affect their commitment to the project team and how they view each other and the contributions that each can offer to the project.

3. Develop a conflict management procedure for your meeting in 30 minutes. Create a simple script to help you anticipate the comments you are likely to hear from both parties. 105 Copyright 2019 Pearson Education, Inc.


Students can be asked to roleplay the part of either Neil or Susan as they respond to this question. The instructor should assign one person (or group) to serve as the project manager and two others to adopt the conflicting personalities. Because the issue has flared up, it is likely that tempers will still be high when they meet with the project manager; thus, a script should anticipate more give-and-take between the two individuals and consider at what points it makes sense to intervene, how far to allow them to express their opinions, and exactly what should be said to them to get something positive out of the conflict. As it has come to an open confrontation, a script for conflict resolution has to recognize that there are egos involved. 4. Which conflict resolution style is warranted in this case? Why? How might some of the other resolution approaches be inadequate in this situation? This question requires students to think in terms of others and their likely responses to various behaviors. Should the project manager develop an approach that leads to confrontation? If so, what will be the outcome? First, confrontation techniques take time; therefore, a meeting with these two people should be given sufficient time for the issues to come out and then for resolution to occur. Should the project manager arbitrate the conflict? Ignore it? Each of these options must be explored and the downside addressed. Usually, by the time an open confrontation has occurred, the more benign approaches such as ignoring or accepting it may no longer be viable options.

Case Study 6.3: Johnson & Rogers Software Engineering, Inc. This case shows one example of the types of problem that can be encountered with the use of new technologies, such as the Internet, to help link participants on a distributed project team. In this case, a combination of geographically-dispersed project team members, faulty technologies, and other concerns are hampering progress on the project. Further, an additional phenomenon is the fact that distributed project teams do not allow for standard team development stages to occur because team members can only interact in formal channels. Without informal communication, it becomes more difficult to build trust and enthusiasm among members of the project team. Kate’s difficulties here are by no means abnormal, but a managerial plan for handling these communications will go a long way toward helping her and the team get through the awkward “newness” of geographically-dispersed project teams. Questions 1. How would you advise Kate to proceed? Analyze the conversation she had this morning. What went right? What went wrong? Students should consider that Kate does recognize there is a problem here and she is willing to make the tough decision to work to find a way that they can continue to communicate in a live setting. The easiest option would have been to agree with the team 106 Copyright 2019 Pearson Education, Inc.


members and move to either one-on-one conversations or simple e-mails. Kate is to be congratulated on sticking to her guns. On the other hand, she is still new at this technology and has been unaware of time differences among the team. In trying to please everybody on the team, she may end up not pleasing anybody, so at some point she will need to make some tough choices. Like it or not, some team members will need to meet at difficult times. On the other hand, if she does up-front planning prior to these meetings and ensures that everyone is prepared, it may be possible to limit the amount of time spent on the meeting itself. The key is advance preparation by Kate and everyone else on the team. 2. What should Kate’s next steps be? Kate needs to decide whether she is committed to real-time link-ups with the team. If she really values this time, she must get the maximum benefit out of it by making sure these meetings are productive and that everyone is prepared in advance. Further, perhaps she could organize some subgroup meetings among sets of the participants and save the fullblown team meetings for occasional use. Students should be asked to think creatively to come up with some action options for Kate to adopt. 3. How can she use the technology of the Internet and teleconferencing to enhance team development and performance? Teleconferencing can start to create a team out of a group of faceless individuals but it must be used appropriately. The chapter cites several principles that can be used to enhance the teleconferencing experience and instructors should ensure that students address each of them as a means to help Kate resolve some of her problems.

107 Copyright 2019 Pearson Education, Inc.


CHAPTER FOUR Leadership and the Project Manager Chapter Outline PROJECT PROFILE NASA Taps a Leader with the Right Stuff to Run Their Mars 2020 Project Leading by Example for the London Olympics—Sir John Armitt 4.1 INTRODUCTION: SUCCESSFUL PROJECTS NEED LEADERS 4.2 LEADERS VERSUS MANAGERS 4.3 HOW THE PROJECT MANAGER LEADS Acquiring Project Resources Motivating and Building Teams Having a Vision and Fighting Fires Communicating PROJECT MANAGEMENT RESEARCH IN BRIEF Leadership and Emotional Intelligence 4.4 TRAITS OF EFFECTIVE PROJECT LEADERS Conclusions about Project Leaders PROJECT PROFILE Leading by Example for the London Olympics—Sir John Armitt 4.5 PROJECT CHAMPIONS Champions—Who Are They? What Do Champions Do? How to Make a Champion 4.6 THE NEW PROJECT LEADERSHIP PROJECT MANAGERS IN PRACTICE Bill Mowery, CSC PROJECT PROFILE The Challenge of Managing Internationally 4.7 PROJECT MANAGEMENT PROFESSIONALISM 4.8 PROJECT MANAGEMENT AND ETHICS Unethical Behaviors in Project Management PROJECT PROFILE Brazilian Construction Giant Caught in Wide-Spread Corruption Scandal Summary Key Terms Discussion Questions Case Study 4.1 In Search of Effective Project Managers Case Study 4.2 Finding the Emotional Intelligence to Be a Real Leader Case Study 4.3 Volkswagen and Its Project to Cheat Emissions Tests Case Study 4.4 Problems with John Internet Exercises PMP Certification Sample Questions Notes

69 Copyright 2019 Pearson Education, Inc.


TRANSPARENCIES Note for Instructors: To present transparencies in class, please download the PowerPoint Presentations (available on www.pearsonhighered.com) that accompanies this product. The PPT ISBN is 9780134730479.

4.1 DIFFERENCES BETWEEN MANAGERS AND LEADERS Concerns Creation of Purpose

Developing a Network for Achieving the Agenda

Execution

Outcomes

Focus Time-Frame

Managers Focus on plans and budgets; creates steps, timetables for achieving results and looks for resources to support goals. Organizes and staffs; creates structure for achieving the plans; delegates responsibility and authority; develops procedures to guide behavior; creates monitoring systems. Controls and solves problems; monitors results and applies corrective action. Produces a degree of predictability and order; seeks to maintain the status quo. Efficiency of operations Short-term, avoiding risks, maintaining and imitating.

Leaders Establishes direction; creates a vision and the strategies needed to achieve it.

Aligns people with the target; communicates direction by word and deed to those whose cooperation is needed; creates teams that understand and share the project’s vision. Motivates and inspires; energizes people overcome obstacles and show personal initiative. Produces change; challenges the status quo.

Effectiveness of outcomes Long-term; taking risks, innovating and originating.

4.2 DUTIES OF PROJECT MANAGERS 1. ACQUIRING PROJECT RESOURCES 2. TEAM BUILDING AND MOTIVATION 3. HAVING A VISION AND FIGHTING FIRES 4. COMMUNICATING 4.3 SEVEN ESSENTIAL PROJECT MANAGEMENT ABILITIES 1. ORGANIZING UNDER CONFLICT 2. EXPERIENCE 3. DECISION-MAKING 4. PRODUCTIVE CREATIVITY 5. ORGANIZING WITH COOPERATION 70 Copyright 2019 Pearson Education, Inc.


6. COOPERATIVE LEADERSHIP 7. INTEGRATIVE THINKING 4.4 PROJECT CHAMPION ROLES 1. CREATIVE ORIGINATOR 2. ENTREPRENEUR 3. “GODFATHER” OR SPONSOR 4. PROJECT MANAGER 4.5 STEPS TO DEVELOPING PROJECT MANAGEMENT PROFESSIONALS 1. MATCH PERSONALITIES TO PROJECT WORK 2. USE TRAINING PROGRAMS TO FORMALLY COMMIT TO PROJECT MANAGEMENT 3. DEVELOP A UNIQUE AWARD SYSTEM FOR PROJECT MANAGERS 4. IDENTIFY A DISTINCT CAREER PATH FOR PROJECT MANAGERS

DISCUSSION QUESTIONS 4.1 The chapter stressed the idea that project management is a leader-intensive undertaking. Discuss in what sense this statement is true. Managing a project requires a high-level of coordination, communication, motivation and goal-sharing. Project managers are the central entity within the project team and are the contact point for internal (other managers within the company) and external partners (contractors, joint venture partners, etc.). Given their vital role in the project, their ability to lead and motivate others is directly tied to the success of the project. Without a good leader, team members can become confused, conflicting goals may arise and ultimately, projects can end up off-course or behind schedule.

4.2 How do the duties of project managers reinforce the role of leadership? Project managers are in charge of leading a team (often of diverse individuals) toward a common goal. To do so, they must motivate and inspire people toward the project’s end goal. Alignment is one way project managers streamline operations. This is because diffusion of decision-making and innovation is possible when team members are aligned with the overall vision of the project. A key to alignment and to shareholder maintenance is communication. Project managers need to communicate effectively to their team and to outside shareholders during project implementation, when problems arise or when a major project change is required. Project managers must also create interpersonal relationships, not only within their team, but also with other functional and top-level managers. This helps facilitate upper-management commitment, resource sharing, and information flow. 71 Copyright 2019 Pearson Education, Inc.


4.3 What are some key differences between leaders and managers? Leaders develop strategies in line with a vision, while managers are in charge of planning and budgeting resources to implement the strategies. In terms of personnel, leaders align people with their vision ensuring that everyone understands the direction and is motivated toward its accomplishment. Managers on the other hand are primarily concerned with organizing and staffing personnel in appropriate roles and delegating each person specific responsibilities. Overall, managers are geared toward short-term planning, problem solving, and stability. Leaders view things long-term, create challenges, and take risks.

4.4 Discuss the concept of emotional intelligence as it relates to the duties of project managers. Why are the five elements of emotional intelligence so critical to successful project management? Emotional intelligence is important to relationship building and maintenance. The five elements of emotional intelligence are self-awareness, self-regulation, motivation, empathy, and social skills. They involve: an awareness of your own strengths and weaknesses, motives and feelings, the ability to regulate your own behavior to remain under control in typically impulsive situations, an internal motivation to measure progress and set challenging goals, being capable of appreciating others’ backgrounds and feelings, and the ability to manage relationships with others. It can be vital in crisis situations where it is necessary to interpret the state-of-mind of those around you (i.e., members of the project team). Project management involves a high-degree of people management—getting the right people to do the right things. Having a high emotional intelligence helps managers “fight fires” and can create harmonious relationships between the manager and subordinates, peers and superiors.

4.5 Consider the studies on trait theories in leadership. Of the characteristics that emerge as critical to effective leadership, which seem most critical for project managers? Why? Five important traits for project managers are good communication skills, honesty, adaptability, interpersonal skills, and influential. Communication is vital to project management. Project leaders are the contact point for everyone within and outside of the project team. Being able to communicate issues regarding the vision/goal and project progress are key. Honesty is important, as people are unlikely to follow or believe in someone whom they do not trust. By nature, projects incur a high level of risk and thus a great number of changes are likely during the life of a project. A manager who can adapt quickly increases a project’s chance of success. Project managers have to work through others to achieve the goals of the project. This requires a level of interpersonal skill to build and maintain relationships that facilitate goal accomplishment. Lastly, one major role of project managers is to influence. They need to be influential in order to get others onboard and to spread enthusiasm for the project.

72 Copyright 2019 Pearson Education, Inc.


4.6 Consider the profile examples on project leaders Sir John Armitt and Jim Watzin from the chapter. If you were to summarize the leadership keys to their success in running projects, what actions or characteristics would you identify as being critical? Why? What are the implications for you when you are given responsibility to run your own projects? This question gives students a chance to reflect on the characteristics of effective leaders, principally in terms of their decision-making, interpersonal, and stakeholder management behaviors. In fact, the theme of stakeholder management runs deeply through both profiles and would be an excellent question for students to consider: how much of project leadership is appearing decisive and how much is having the ability to be a good listener? Finally, they may offer a number of different answers to the question of what they now consider the keys to being a successful leader when they are given the responsibility to run a project.

4.7 Why are project champions said to be better equipped to handle the nontraditional aspects of leadership? Champions are often personally committed to a project. They are able to identify with the project in ways that traditional managers do not. Being that champions usually head their own pet projects, they are more inclined to develop the project’s vision or become cheerleaders. Additionally, because they are personally tied to the project they often work harder to overcome challenges and to defend the project to the rest of the organization. 4.8 Consider the discussion of “new project leadership.” If you were asked to formulate a principle that could be applied to project leadership, what would it be? Justify your answer. The answer to this question requires students with project experience, or those who are grasping some of the challenges of managing projects, to formulate a principle that identifies a unique aspect of project leadership. The answer given is not as important as having the student justify their reasoning behind the principle. It offers a good method for determining if students comprehend the nature of the project leadership challenge.

4.9 Consider the Odebrecht case (Box 4.3) and its habit of using bribes and kickbacks to secure project contracts. In what ways is this unethical behavior the fault of the firm, and hos is it the fault of the customers? How can an organization develop an ethical culture? Discuss the idea in relation to rewards and punishments. The answer to this question is not clear-cut. Clearly, Odebrecht was engaged in systematic bribery and unethical behavior over an extended period of time. The question is whether or not customers “understood” that this was now business was supposed to be 73 Copyright 2019 Pearson Education, Inc.


transacted. It is useful to discuss ethics from a cultural perspective; for example, business dealings in Japan that would be considered unethical in the U.S. may be thought perfectly appropriate in the other country. Lastly, it is useful to ask students how rewards and punishments send clear messages as to what behaviors our culture really rewards, rather than the ones we claim to reward. Students can be assigned a research assignment to investigate examples of unethical business cultures in local firms to see how these actions began and how they are often supported by promotions and raises going to those who engage in unethical acts.

CASE STUDIES Case Study 4.1—In Search of Effective Project Managers This case involves Pureswing Golf, and illustrates the problems when organizations attempt to locate competent project managers without any systematic plan for identifying and training good potential candidates. They are discovering that the “voluntary approach,” whereby new project managers are solicited seemingly at random from around the company, simply does not work. Many of these individuals likely do not have the skills or a reasonable understanding of what it takes to manage projects effectively. Questions 1. Imagine you are a human resources professional at Pureswing who has been assigned to develop a program for recruiting new project managers. Design a job description for the position. Based on the skills identified in this chapter, it would be possible to develop a job description that highlights several of the key features that strong project leaders possess. The job description must discuss the ability to manage a team, to possess strong time management and organizational skills, to have a sufficient technical background that the individual can understand the core technologies, the ability to work to deadlines, knowledge of cost accounting and finance, and so forth. The instructor can tease out this list for some time, adding additional descriptive skills needed to handle the job. The goal is to highlight the fact that effective project managers must possess a solid, well-rounded set of capabilities that enable them to almost function as “mini-CEOs” within their organizations.

2. What qualities and personal characteristics support a higher likelihood of success as a project manager? The types of personality characteristics that are best associated with project management success, include interpersonal skills, communication skills, technical competency, and so 74 Copyright 2019 Pearson Education, Inc.


forth. As several tables in this chapter suggest, the skills and personality characteristics needed to succeed as a project manager are diverse and quite comprehensive.

3. What qualities and personal characteristics would make it difficult to be a successful project manager? In answering this question, the instructor can have student brainstorm some pathologies (within reason) that make certain people poor project managers. For example, a discussion could occur around the question of whether or not good project managers can delegate. Some would argue against delegation, suggesting that project managers must be on top of everything. The counter-argument can suggest that the purpose of superior team skills requires developing trust in others and the need to delegate. Other examples of qualities that work against becoming a good project manager could be the inability to shift from a “big picture” focus to being detail-oriented and vice versa. It has been suggested that good project managers cannot get bogged down in details nor can they remain above the fray, but require the ability to shift their focus constantly from the forest to the trees and back again.

Case Study 4.2—Finding the Emotional Intelligence to Be a Real Leader As the title suggests, this case is about the concept of emotional intelligence and its role in effective leadership. Kathy was a competent project manager who had been successful in the past, in other settings, but was failing badly in her first effort as head of a large, international project. Her inability to understand her team’s sensibilities and her own domineering style are combining to create a highly negative team environment. The nature of emotional intelligence is identified by possessing: (1) self-awareness, (2) selfregulation, (3) motivation, (4) empathy, and (5) social skill. In this case, it appears that Kathy, though possessing strong motivation and, to some degree, self-regulation, is singularly lacking in empathy and social skills. Further, it could be argued that she is doing a poor job with self-awareness as well, as she does not understand how her own behaviors contributed to these problems. Questions 1. Discuss how Kathy lacked sufficient emotional intelligence to be effective in her new project manager assignment. As noted above, of the five characteristics of emotional intelligence, Kathy appears to be seriously lacking the three of them—self-awareness, empathy, and social skills. Her inability to appreciate and modify her behavior to work with people of different cultures is the key problem with her management style. Further, she is in error by assuming that the same management behaviors that worked well for her in another setting would transfer to this new project with a different workforce and cultural sensibilities. 75 Copyright 2019 Pearson Education, Inc.


2. Of the various dimensions of emotional intelligence, which dimension(s) did she appear to lack most? What evidence can you cite to support this contention? As above, students should quickly note that she lacks empathy and social skills. With a little prompting, they can also see how many of these problems also contribute to and affect her self-awareness. Being caught off-guard by problems in trying to manage a large project in a foreign country suggests a lack of self-awareness on Kathy’s part.

Case Study 4.3—Volkswagen and Its Project to Cheat Emissions Tests This case is based on a current and very timely series of events, relating to the decision by Volkswagen and other car manufacturers to willingly cheat diesel emissions tests by installing a software device that detects the difference between shop testing and actual driving on the roads. Volkswagen actually created a project team of engineers to design this cheat device, in spite of the second-thoughts and objections of several engineers who worked on the software device. Once the evidence for this deception began to be made public, Volkswagen executives engaged in systematic efforts to silence their own engineers, destroy documents, and cover up the fact that they had sanctioned this project in the first place. Since Volkswagen was charged with this deception, other automakers have also been accused and admitted to similar behaviors. Questions 1. How do you think Volkswagen executives could justify this behavior? How do you think the actions of the Japanese automakers influenced VW’s decisionmaking? This question can spark a good debate on the way in which it is easy to create justifications for engaging in unethical or socially-destructive behaviors. “We had to do it to stay competitive.” “Everyone else is doing the same thing.” “It is the U.S. government’s fault for creating unrealistic emissions standards in the first place.” All of these excuses are common ways that people try, after the fact, to explain away or justify what they knew was unsanctioned or illegal behavior. 2. How would you personally respond if you were a member of a project team developing a device that was designed to cheat environmental testing? What if you were the sole support for a large family with three children in college? It is easy, in a classroom, to condemn unethical behavior and most students will easily argue that you should not do these kinds of things. Whistle-blowing is easy in the abstract. When it is personalized to the point where the student is put in the position of having some significant downside penalties for objecting to these practices, it becomes a much more interesting, and unsettling, question for the class to debate. 3. Is there a “moral” to this story for you? What would that moral be? 76 Copyright 2019 Pearson Education, Inc.


This question can be answered in many ways: dealing with pressure to conform to actions demanded by top management, the fact that the cover-up is always worse than the crime, the fact that most such behaviors come to light eventually, and so forth. The point here is to have students personalize this issue and address what lessons they derived from the case and how they can shape behaviors when they are in the workforce or forced to deal with a similar ethical challenge.

Case Study 4.4—Problems with John This is a longer case based on a true story. Students have a lot of fun with this example and with the problems they experience with John because it is a classic problem of motivation and dealing with a dissatisfied employee. John is clearly focused on perceived equity with his peer group and the sense that he is not being fairly treated. Some students will suggest that the key is to continue giving John promotions and other positive strokes so that he starts working to his potential again. The other side will suggest that too much effort has already been spent on John, trying to get him motivated and now is the time to crack down. I have had success in setting this case up as a debate and assigning people to different perspectives on how to deal with John. There are several options for management cited in the case. A sample poll taken prior to the inclass discussion is useful because it forces students to adopt a position on how to deal with John. Once they have made their positions known, it is possible to discuss the case and each of these options (and their potential downside) in more detail.

Questions 1. As the team leader, you have weighed the pros and cons of all options and prepared a presentation to management on how to address this problem. What do you suggest? This is a good question to get discussion started because it allows the instructor to create a chart with pros on one side of the ledger and cons on the other. The case cites five different options that management has identified, so there is a good starting point. Students can be asked to offer both positive and negative comments on the situation and John’s behavior as a first step toward a more complete analysis of what the problems are and how best to deal with them. Next, students can be asked if there are alternative options that top management has not considered and if so, the benefits and drawbacks of each of these options. 2. Consider each of the options, and develop an argument to defend your position for each option. As above, the options developed in the case each have positive and negative elements associated with them. For example, someone might suggest that the key to resolving John’s lack of motivation is to encourage him to finish his degree so that he can be 77 Copyright 2019 Pearson Education, Inc.


promoted more regularly; however, others could note that he has tried that over the past years and never stuck with it. The goal is to get students thinking critically, having them avoid making knee-jerk responses based on a cursory reading of the case, and consider the ramifications of the choices they come up with. 3. What specific leadership behaviors mentioned in this chapter are most relevant to addressing and resolving the problem with John? Several leadership behaviors come to mind, most centered around the need for good interpersonal skills, an understanding of motivation (especially equity and expectancy theories), the leader needs personal credibility, the ability to think of creative solutions (“outside the box” options), and mostly the leader must have the personal integrity to be honest and straightforward with John—to demonstrate an understanding of John’s behavior, a willingness to consider why it is occurring, and the honesty to show him that if it continues, it will necessitate consequences.

78 Copyright 2019 Pearson Education, Inc.


CHAPTER THREE Project Selection and Portfolio Management Chapter Outline PROJECT PROFILE Project Selection Procedures: A Cross-Industry Sampler 3.1 INTRODUCTION: PROJECT SELECTION 3.2 APPROACHES TO PROJECT SCREENING AND SELECTION Method One: Checklist Model Method Two: Simplified Scoring Models Limitations of Scoring Models Method Three: The Analytical Hierarchy Process Method Four: Profile Models 3.3 FINANCIAL MODELS Payback Period Net Present Value Discounted Payback Internal Rate of Return Choosing a Project Selection Approach PROJECT PROFILE Project Selection and Screening at GE: The Tollgate Process 3.4 PROJECT PORTFOLIO MANAGEMENT Objectives and Initiatives The Portfolio Selection Process Developing a Proactive Portfolio Keys to Successful Project Portfolio Management Problems in Implementing Portfolio Management Summary Key Terms Solved Problems Discussion Questions Problems Case Study 3.1 Keflavik Paper Company Case Study 3.2 Project Selection at Nova Western, Inc. Internet Exercises Notes

34 Copyright 2019 © Pearson Education, Inc.


TRANSPARENCIES Note for Instructors: To present transparencies in class, please download the PowerPoint Presentations (available on www.pearsonhighered.com) that accompanies this product. The PPT ISBN is 9780134730479.

3.1 SIMPLIFIED CHECKLIST MODEL FOR PROJECT SELECTION Performance on Criteria High Project

Criteria

Project Alpha

Cost Profit Potential Time to Market Development Risks

Project Beta

Project Gamma

Project Delta

Cost Profit Potential Time to Market Development Risks

Cost Profit Potential Time to Market Development Risks

Cost Profit Potential Time to Market Development Risks

Medium

Low

X X X X

X X X X

X X X X

X X X X

35 Copyright 2019 © Pearson Education, Inc.


3.2 SIMPLE SCORING MODEL

Project

Criteria

(A) Importance Weight

(B) Score

(A) × (B) Weighted Score

Cost

1

3

3

Profit Potential

2

1

2

Development Risk

2

1

2

Time to Market

3

2

6

Project Alpha

Total Score

13

Project Beta Cost

1

2

2

Profit Potential

2

2

4

Development Risk

2

2

4

Time to Market

3

3

9

Total Score

19

36 Copyright 2019 © Pearson Education, Inc.


3.3 PROFILE MODEL

3.4 FINANCIAL MODELS—PAYBACK PERIOD

Comparison of Payback for Projects A and B Project A

Year 0 1 2 3 4 5

Cash flow ($500,000) 50,000 150,000 350,000 600,000 500,000

Cum. Cash Flow ($ 500,000) (450,000) (300,000) 50,000 650,000 1,150,000

Payback = 2.857 years Rate of Return = 35%

37 Copyright 2019 © Pearson Education, Inc.


Project B

Year 0 1 2 3 4 5

Cash flow ($500,000) 75,000 100,000 150,000 150,000 900,000

Cum. Cash Flow ($500,000) (425,000) (325,000) (175,000) (25,000) 875,000

Payback = 4.028 years Rate of Return = 24.8%

3.5 FINANCIAL MODELS—NET PRESENT VALUE ASSUME: $100,000 INITIAL INVESTMENT REQUIRED RATE OF RETURN: 10% INFLATION: 4% USEFUL LIFE: 4 YEARS ANTICIPATED CASH FLOWS: Year 1: Year 2: Year 3: Year 4:

$20,000 $50,000 $50,000 $25,000

EXAMPLE - DISCOUNT FACTOR FOR YEAR ONE Discount factor = (1/(1 + .10 + .04)3) = .6749

Discounted Cash Flows and NPV Year

Inflows

0 1

20,000

Outflows

Net flow

Discount Factor

NPV

100,000

(100,000)

1.000

(100,000)

20,000

0.8772

17,544 38

Copyright 2019 © Pearson Education, Inc.


2

50,000

50,000

0.7695

38,475

3

50,000

50,000

0.6749

33,745

4

25,000

25,000

0.5921

14,803

Total

$4,567

3.6 FINANCIAL MODELS —DISCOUNTED PAYBACK ASSUME: $100,000 INITIAL INVESTMENT REQUIRED RATE OF RETURN: 12.5%

EXAMPLE—DISCOUNT FACTOR FOR YEAR ONE Discount factor = (1/(1 + .125)1) = .89

Year 1 2 3 4 5 Payback Period

Project Cash Flow* Discounted $8,900 7,900 7,000 6,200 5,500 4 Years

Undiscounted $10,000 10,000 10,000 10,000 10,000 3 Years

39 Copyright 2019 © Pearson Education, Inc.


3.7 PROJECT PORTFOLIO MANAGEMENT

Project Portfolio Matrix

40 Copyright 2019 © Pearson Education, Inc.


DISCUSSION QUESTIONS

3.1 If you were to prioritize the criteria for a successful screening model, which criteria would you rank at the top of your priority list? Why? Of the criteria for successful screening models, realism would top the priority list. Realism requires that the model is in line with the constraints and objectives of the organization. The other criteria are based on how usable and applicable the model is. However, these measures would mean little if the model was unable to provide a recommendation that was usable within the resources available to the company (i.e., realistic alternative). 3.2 What are the benefits and drawbacks of checklists as a method for screening project alternatives? Project checklists are easy to use, based on a simplistic visual model with a basic scoring system. Using a checklist enhances the input and discussion during the screening process. Unfortunately, the model also has its shortcomings. The two most significant are the subjectivity of the rating system and the lack of a weighting system. The weighting system is important in establishing trade-offs between criterion. 3.3 How does use of the Analytical Hierarchy Process (AHP) aid in project selection? In particular, what aspects of the screening process does the AHP seem to address and improve directly? The Analytical Hierarchy Process breaks the broad criterion categories of other selection models into smaller, more manageable pieces that have more defined focuses. This allows the AHP to create a more accurate ordering of priorities than other models. It also enables a better demonstration of how potential alternatives meet organizational goals and strategy. Weighting that is absent or inefficient under the checklist and scoring model is improved. AHP allows weighting by main and sub-criterion which eliminates the double counting of the scoring model. Finally, the AHP creates results that are easily compared between projects as well as within in cost/benefit analysis.

3.4 What are the benefits and drawbacks of the profile model for project screening? Be specific about the problems that may arise in identifying the efficient frontier. The Profile Model is beneficial because it clearly outlines the relationship between risk and return of project alternatives. It also establishes a threshold for eliminating or qualifying projects. On the other hand, it may not be as effective for selecting alternatives because it limits discriminating criteria to risk and return only. Additionally, it can be difficult to accurately quantify risk.

41 Copyright 2019 © Pearson Education, Inc.


3.5 How are financial models superior to other screening models? How are they inferior? Financial models are superior to screening models in that they link project alternatives to financial performance. The results of financial models are non-subjective meaning they are not subject to individual interpretation (10% return means 10% return regardless of who is looking at it). Therefore, it becomes easier to compare the benefits of one project alternative versus another. The models do have some drawbacks. Due to the required information in determining NPV and IRR, it may be difficult to make long-term estimates accurately (i.e. would have to estimate future inflation and interest rates). Economic conditions may be unknown or unstable. Determinations are made about the economic future and may turn out to be invalid. 3.6 How does the options model address the problem of non-recoverable investment in a project? The options model looks at alternatives faced when a company has already made investment into a project. Confronted with different possibilities in how to handle the non-recoverable investment (past investment), the options model reveals whether investing into the project in the future will be beneficial or whether the company should simply allow the project to live out its current potential. 3.7 What advantages do you see in the GE Tollgate screening approach? What disadvantages do you see? How would you alter it? The major advantage and disadvantage of GE’s Tollgate screening approach are derived from the same source—multiple checks and reviews. The advantage of the process is the project is under constant review. This means problems are addressed immediately and that time and money is not continually invested in doomed projects. However, this same process may lead to excessive delays in projects due to time spent on checklists, reviews, and waiting for the feedback loop to be completed. Perhaps maintaining the process, but reducing the number of players involved may help to speed up the process while still reaping the benefits of the system and reducing the risk of projects. For example, perhaps having one review board (a cross-functional team headed by a member of senior management) that meets briefly, but frequently with the project team for progress updates would eliminate some of the bureaucratic levels of the system. 3.8 Why is project portfolio management particularly challenging in the pharmaceutical industry? Portfolio management is difficult in the pharmaceutical industry due to the high uncertainty, low success rate, long lead time, and high investment costs of pharmaceutical projects. The uncertainty and high risk associated with the costs and success rates make it difficult to create a balanced, reliable portfolio. The portfolio must be constantly updated and re-evaluated as projects fail, stall, or become obsolete. 42 Copyright 2019 © Pearson Education, Inc.


3.9 What are the keys to successful project portfolio management? The keys to successful project portfolio management are flexibility (freedom from layers of authority) and open communication that allows projects teams to be innovative, using low cost methods to test new markets or product ideas, and smooth, timely transitions between projects. 3.10 What are some of the key difficulties in successfully implementing portfolio management practices? There are several common problems that may hinder successful implementation of portfolio management. To begin with, top management may run into conflicts with technical staff. For instance, engineers may not want to alter or abandon a project that management finds too risky. Another difficulty is investment in projects that do not fall in line with portfolio priorities. Strategy and the portfolio must remain aligned for project portfolios to be successful. A highly detrimental occurrence revolves around unpromising projects. Here, companies continue to pour money into projects even after there is no hope for the project’s success. Finally, project portfolio management may fail due to a lack of resource. Primarily, this comes when there are not enough personnel to support the implementation or when there are not enough resources to initiate the desired set of projects.

CASE STUDIES

Case Study 3.1 Keflavik Paper Company Keflavik Paper is an organization that has lately been facing serious problems with the results of its projects. Specifically, the company’s project development record has been spotty: While some projects have been delivered on time, others have been late. Budgets are routinely overrun, and product performance has been inconsistent, with the results of some projects yielding good returns and others losing money. They have hired a consultant to investigate some of the principal causes that are underlying these problems, and he believes that the primary problem is not how project are run but how they are selected in the first place. Specifically, there is little attention paid to the need to consider strategic fit and portfolio management in selecting new projects. This case is intended to get students thinking of alternative screening measures that could potentially be used when deciding whether or not to invest in a new project. Questions 1. Keflavik Paper presents a good example of the dangers of excessive reliance on one screening technique (in this case, discounted cash flow). How might 43 Copyright 2019 © Pearson Education, Inc.


excessive or exclusive reliance on other screening methods discussed in this chapter lead to similar problems? Some measures that allow us to screen projects may lead to the wrong conclusions; for example, suppose that we selected projects in construction settings for their aesthetic appeal and ability to promote our name across the industry. If insufficient attention was then paid to issues such as cost of the project or safety concerns, we may be selecting projects that will ultimately damage our reputation or drive us out of business. Instructors should probe various screening techniques for their strengths and weaknesses to ultimately demonstrate that effective screening methods usually rely on multiple, complementary measures for selecting projects. 2. Assume that you are responsible for maintaining Keflavik’s project portfolio. Name some key criteria that should be used in evaluating all new projects before they are added to the current portfolio. Students can use this as a brain-storming exercise. Among the criteria they could list are (1) relationship to current projects or products the company carries, (2) new market penetration potential, (3) technological feasibility, and (4) cost of development. This is a short list and students could potentially add several other criteria to it.

3. What does this case demonstrate about the effect of poor project screening methods on a firm’s ability to manage its projects effectively? The firm’s ad hoc approach to project selection demonstrates that even taking on projects that could yield strong cash flows may injure the organization due to its inability to manage them well. Further, it highlights the dangers of using either a single or one heavily weighted criterion for project selection. Successful project portfolios are consciously constructed and managed as a coherent whole, not simply a collection of diverse opportunities.

Case Study 3.2 Project Selection at Nova Western, Inc. This case presents an example that is common, in which different screening methods may yield different findings. In this case, two projects are competing for funding; Project Janus, championed by the organization’s Software Development group and Project Gemini, which as the backing of their Business Applications organization. Using a weighted scoring model, it appears that Project Gemini offers the best alternative in terms of the criteria employed. On the other hand, when a Discounted Cash Flow approach is used, the results suggest that Project Janus will earn greater returns on initial investment. Instructors can use this case to illustrate the fact that many times, selection models will point to conflicting results, particularly when financial models are paired with nonfinancial approaches. 44 Copyright 2019 © Pearson Education, Inc.


Instructors can fashion a debate from this case, in which they assign one team to serve as champions for Project Janus and the other for Project Gemini. It serves as a valuable exercise for requiring students to commit to one approach or another, defend their positions, and also examine these competing models for their strengths and weaknesses. Questions 1. Phyllis has called you into her office to help her make sense of the contradictions in the two project evaluations. How would you explain the reasons for the divergence of opinion from one technique to the next? What are the strengths and weaknesses of each screening method? The chapter notes several strengths and weaknesses of each project screening method and these should be considered in this case. It is not uncommon for financial and nonfinancial screening methods to yield competing information; thus, an argument could be made that using only these two methods is insufficient and in fact, an enhanced screening model should be developed for Nova that considers these factors as part of an overall, larger model of choice. Those instructors familiar with Expert Choice™ software could set up this case as an exercise using the Analytical Hierarchy Process (AHP) discussed in the chapter. 2. Choose the project that you think, based on the two analyses, Nova Western should select. Defend your choice. A reasonable case could be made for selecting either Project Janus or Project Gemini. For example, though Project Janus offers a higher net present value for your initial investment, the payoff period is two years longer than Project Gemini, suggesting that if the firm does not wish to tie its money up too long, Gemini might be a reasonable alternative choice. Likewise, the weighted criteria model seems to favor Project Gemini. Students should be encouraged to consider the criteria Nova employed in project selection. Are they reasonable, or should other factors be considered as well? 3. What does this case suggest to you about the use of project selection methods in organizations? How would you resolve the contradictions found in this example? A successful screening model is often a comprehensive one. Simplistic models typically yield simplistic answers and their consistency from method to method is questionable. As the chapter demonstrates, most effective screening techniques used in organizations today are complex, multi-faceted, and comprehensive in nature. One simple solution to this case might be to use the results of the discounted cash flow analysis as an additional factor in the weighted scoring model, whereby net present value becomes an additional selection criterion to consider along with the other factors already listed. Likewise, students should be asked to consider if any of the criteria in the scoring model represent “must” items that cannot be compromised, such as safety. Finally, student may point out that the factor “Potential Profit” ranks both projects identical. However, the DCF model 45 Copyright 2019 © Pearson Education, Inc.


shows that Project Janus might reasonably be ranked higher. Ask students how this reevaluation might change final results.

PROBLEMS 3.13

Checklist. Suppose that you are trying to choose which of two IT projects to accept. Your company employs three primary selection criteria for evaluating all IT projects: (1) proven technology, (2) ease of transition, and (3) projected cost savings.

One option, Project Demeter, is evaluated as: Technology Ease of transition Projected cost savings

high low high

The second option, Project Cairo, is evaluated as: Technology Ease of transition Projected cost savings

medium high high

Construct a table identifying the projects, their evaluative criteria, and ratings. Based on your analysis, which project would you argue in favor of adopting? Why? Solution:

Performance on Criteria

High Project

Criteria

Project Demeter

Technology Ease of Transition Projected Cost Savings

Project Cairo

Technology Ease of Transition Time to Market

Medium

Low

X X X

X X X

46 Copyright 2019 © Pearson Education, Inc.


On the basis of this checklist, and assuming that all criteria are equally weighted, Project Cairo is marginally superior to Project Demeter. Cairo scores two highs and one medium, while Demeter scores two highs and one low.

3.14

Checklist. For your answer to Problem 3.13, construct an argument as to why this selection process is useful for deciding which project is best. Now, construct a counterargument to criticize this choice. In other words, are you comfortable presenting this project choice to top management? Why or why not?

Solution: The argument in favor of Project Cairo is that based on the three key performance criteria identified; this project choice maximizes the results, with two high and one medium score. If these are the relevant criteria, this is the best option. The counter-arguments include the point that these are simplistic criteria, subject to multiple interpretations as to their definitions. Also, what does “high” or “medium” really mean?

3.15

Checklist. Consider the following information in choosing among the four project alternatives below (labeled A, B, C, and D). Each has been assessed according to four criteria:  Payoff potential  Safety  Lack of risk  Competitive advantage Project A is rated: Payoff potential high Safety high Lack of risk low Competitive advantage medium Project B is rated: Payoff potential Lack of risk

low medium

Project C is rated: Payoff potential Lack of risk

medium medium

Safety Competitive advantage

low low

Safety Competitive advantage

medium medium

47 Copyright 2019 © Pearson Education, Inc.


Project D is rated: Payoff potential Lack of risk Competitive advantage

high high Safety medium

Construct a project checklist model for screening these four alternatives. Based on your model, which project is the best choice for selection? Why? Which is the worst? Why? Solution: Performance on Criteria

High Project

Criteria

Project A

Payoff Potential Lack of Risk Safety Competitive Advantage

Project B

Project C

Project D

Low

X X X X

Payoff Potential Lack of Risk Safety Competitive Advantage

X X X X

Payoff Potential Lack of Risk Safety Competitive Advantage

Payoff Potential Lack of Risk Safety Competitive Advantage

Medium

X X X X

X X X X

48 Copyright 2019 © Pearson Education, Inc.


According to this checklist, it appears that Project D is the best option, as all ratings are either high or medium. Project C is the worst with only two medium and two low ratings.

3.16

Scoring Model. Suppose the information in Problem 3.15 was supplemented by importance weights for each of the four assessment criteria as follows, where 1 = low importance and 4 = high importance: Assessment Criteria Importance Weights 1. Payoff potential 4 2. Lack of risk 3 3. Safety 1 4. Competitive advantage 3 Assume, too, that evaluations of high receive a score of 3, medium 2, and low 1. Recreate your project scoring model and reassess the four project choices (A, B, C, and D). Now which project alternative is the best? Why? Solution:

Project

Criteria

(A) Importance Weight

(B) Score

(A) × (B) Weighted Score

Project A Payoff Potential

4

3

12

Lack of risk

3

1

3

Safety

1

3

3

Competitive Advantage

3

2

6

Total Score

24

49 Copyright 2019 © Pearson Education, Inc.


Project

Criteria

(A) Importance Weight

(B) Score

(A) × (B) Weighted Score

Project B Payoff Potential

4

1

4

Lack of risk

3

2

6

Safety

1

2

2

Competitive Advantage

3

2

6

Total Score

18

Project C Payoff Potential

4

2

8

Lack of risk

3

2

6

Safety

1

1

1

Competitive Advantage

3

1

3

Total Score

18

Project D Payoff Potential

4

3

12

Lack of risk

3

3

9

Safety

1

2

2

Competitive Advantage

3

2

6

Total Score

29

Therefore: Choose Project D, which has the highest total score of 29. 50 Copyright 2019 © Pearson Education, Inc.


3.17

Scoring Model. Now assume that for Problem 3.16, the same importance weights are altered as follows:

Assessment Criteria Importance Weights Payoff potential 1 Lack of risk 1 Safety 4 Competitive advantage 2 How does this new information alter your decision? Which project now looks most attractive? Why? Solution:

Project

Criteria

(A) Importance Weight

(B) Score

(A) × (B) Weighted Score

Project A Payoff Potential

1

3

3

Lack of risk

1

1

1

Safety

4

3

12

Competitive Advantage

2

2

4

Total Score

20

51 Copyright 2019 © Pearson Education, Inc.


Project

Criteria

(A) Importance Weight

(B) Score

(A) × (B) Weighted Score

Project B Payoff Potential

1

1

1

Lack of risk

1

2

2

Safety

4

2

8

Competitive Advantage

2

2

4

Total Score

15

Project C Payoff Potential

1

2

2

Lack of risk

1

2

2

Safety

4

1

4

Competitive Advantage

2

1

2

Total Score

10

Project D Payoff Potential

1

3

3

Lack of risk

1

3

3

Safety

4

2

8

Competitive Advantage

2

2

4

Total Score

18

Project A would be the best choice, with the highest weighted score.

52 Copyright 2019 © Pearson Education, Inc.


3.18

Screening Model. Assume that the following criteria relevant to the process of screening various project opportunities are weighted in importance as follows:

 Quality (5)  Cost (3)  Speed to Market (7)  Visibility (5)  Reliability (1) Our company has four project alternatives that satisfy the key features listed above as follows: Alpha Beta Gamma Delta Quality 1 3 3 5 Cost 7 7 5 3 Speed 5 5 3 5 Visibility 3 1 5 1 Reliability 5 5 7 7 Construct a project screening matrix to identify among these four projects the most likely candidate to be implemented. Solution:

Project

Criteria

(A) Importance Weight

(B) Score

(A) × (B) Weighted Score

Project Alpha Quality

5

1

5

Cost

3

7

21

Speed

7

5

35

Visibility

5

3

15

Reliability

1

5

5

Total Score

81

53 Copyright 2019 © Pearson Education, Inc.


Project Beta Quality

5

3

15

Cost

3

7

21

Speed

7

5

35

Visibility

5

1

5

Reliability

1

5

5

Total Score

Project

Criteria

81

(A) Importance Weight

Project Gamma Quality

(B) Score

(A) × (B) Weighted Score

5

3

15

Cost

3

5

15

Speed

7

3

21

Visibility

5

5

25

Reliability

1

7

7

Total Score

83

54 Copyright 2019 © Pearson Education, Inc.


Project Delta Quality

5

5

25

Cost

3

3

9

Speed

7

5

35

Visibility

5

1

5

Reliability

1

7

7

Total Score

81

Project Gamma represents the best choice, using the weighted scoring model.

3.19

     

Screening Model. Assume that the following criteria relevant to the process of screening various construction project opportunities are weighted in importance as follows:

Safety (10) Speed to completion (5) Sustainable development (4) Infrastructure modifications (3) Traffic congestion (5) Cost (8)

Our company has four project alternatives that satisfy these key features as follows: Midtown Uptown Downtown Suburb Safety 6 5 3 8 Speed 3 4 2 5 Sustainable 7 7 5 4 Infrastructure 4 5 4 1 Traffic 2 4 1 8 Cost 5 5 3 6 Construct a project screening matrix to identify among these four projects the most likely candidate to be implemented.

55 Copyright 2019 © Pearson Education, Inc.


Solution:

Project

Criteria

(A) Importance Weight

(B)

Safety

10

6

60

Speed

5

3

15

Sustainable

4

7

28

Infrastructure

3

4

12

Traffic

5

2

10

Cost

8

5

40

Score

(A) × (B) Weighted Score

Midtown

Total Score

165

Uptown Safety

10

5

50

Speed

5

4

20

Sustainable

4

7

28

Infrastructure

3

5

15

Traffic

5

4

20

Cost

8

5

40

Total Score

173

56 Copyright 2019 © Pearson Education, Inc.


Project

Criteria

(A) Importance Weight

(B)

Safety

10

3

30

Speed

5

2

10

Sustainable

4

5

20

Infrastructure

3

4

12

Traffic

5

1

5

Cost

8

3

24

Score

(A) × (B) Weighted Score

Downtown

Total Score

101

Suburb Safety

10

8

80

Speed

5

5

25

Sustainable

4

4

16

Infrastructure

3

1

3

Traffic

5

8

40

Cost

8

6

48

Total Score

212

The Suburb option represents the best choice, using the weighted scoring model.

57 Copyright 2019 © Pearson Education, Inc.


3.20

Profile Model. Assume the project profile model shown in Figure 3.11.

FIGURE 3.11 Project Profile Model

Define the efficient frontier. The dotted lines represent the minimum return and the maximum risk that the company will accept. Which projects would be suitable for retaining and which should be dropped from the company’s portfolio? Why? Solution: The efficient frontier would be a line running between Project B and Project E. Other project options in the portfolio either offer lower potential returns for similar levels of risk (Projects F and C) or are outside the acceptable boundaries for risk and/or return (Projects A and D).

58 Copyright 2019 © Pearson Education, Inc.


3.21

Using the information from the profile model in Problem 3.20, construct an argument as to why project B is preferable to project C.

Solution: Project B gives a similar level of potential return while offering significantly lower risk; thus, because profit (return) is similar across the two project options, it would make more sense to adopt the project with a lower overall risk level.

3.22

Discounted Payback. Your company is seriously considering investing in a new project opportunity, but cash flow is tight. Top management is concerned about how long it will take for this new project to pay back the initial investment of $50,000. You have determined that the project should generate inflows of $30,000, $30,000, $40,000, $25,000, and $15,000 for the next five years. Your firm’s required rate of return is 15%. How long will it take to pay back the initial investment?

Solution: We can set up a discounted cash flow table to calculate the time needed to pay back the initial $50,000 investment. Remember that: Total outflow = $50,000 Required rate of return = 10% Discount factor = 1/(1+.15)t Year 0 1 2 3 4 5 Payback = 2.1 years

Cash Flow ($50,000) 30,000 30,000 40,000 25,000 15,000

Discount Factor 1.0 .87 .76 .66 .57 .50

Net Inflows ($50,000) 26,100 22,800 26,400 14,250 7,500

Conclusion: You will be able to pay off the initial investment in slightly more than two years.

3.23

Discounted Payback. Your company is considering a high-risk project that could yield strong revenues but will involve a significant up-front investment. Because of this risk, top management is naturally concerned about how long it is likely to take to pay off that investment so that they can begin to realize profits. This project will require an investment of $200,000 and your fiveyear projection for inflows is: Year 1 – $50,000, Year 2 – $75,000, Year 3 – 59 Copyright 2019 © Pearson Education, Inc.


$125,000, Year 4 – $200,000, and Year 5 – $250,000. Your firm’s required rate of return is 18%. How long will it take to pay back your initial investment? Solution: We can set up a discounted cash flow table to calculate the time needed to pay back the initial $200,000 investment. Remember that: Total outflow = $200,000 Required rate of return = 18% Discount factor = 1/(1+.18)t Year Cash Flow 0 ($200,000) 1 50,000 2 75,000 3 125,000 4 200,000 5 250,000 Payback = 4.26 years

3.24

Discount Factor 1.0 .85 .72 .61 .52 .44

Net Inflows ($200,000) 42,500 54,000 76,250 104,000 110,000

Suppose for the previous Problem 3.23, that your firm has decided to raise their required rate of return to 20%. How long will it take to pay back your initial investment now?

Solution: We can set up a discounted cash flow table to calculate the time needed to pay back the initial $200,000 investment. Remember that: Total outflow = $200,000 Required rate of return = 20% Discount factor = 1/(1+.20)t Year Cash Flow 0 ($200,000) 1 50,000 2 75,000 3 125,000 4 200,000 5 250,000 Payback = 4.36 years

Discount Factor 1.0 .83 .69 .58 .48 .40

Net Inflows ($200,000) 41,500 51,750 72,500 96,000 100,000

60 Copyright 2019 © Pearson Education, Inc.


3.25

Suppose for the previous Problem 3.23, that your CEO is highly risk averse and will not invest in any project with a payback longer than three years. Assuming a required rate of return of 15%, what would you tell him about this project opportunity?

Solution: Year 0 1 2 3 4 5

Cash Flow ($200,000) 50,000 75,000 125,000 200,000 250,000

Discount Factor 1.0 .87 .76 .66 .57 .50

New Inflows ($200,000) 43,500 57,000 82,500 114,000 125,000

We can see that adding the discounted inflows from the first three years will not equate to $200,000 so we should not pursue the project.

3.26

Net Present Value. Assume that your firm wants to choose between two project options:

Project A: $500,000 invested today will yield an expected income stream of $150,000 per year for five years, starting in Year 1. Project B: an initial investment of $400,000 is expected to produce this revenue stream: Year 1 = 0, Year 2 = $50,000, Year 3 = $200,000, Year 4 = $300,000, and Year 5 = $200,000. Assume that a required rate of return for your company is 10% and that inflation is expected to remain steady at 3% for the life of the project. Which is the better investment? Why? Solution: First, our required rate of return is 3% + 10% = 13% Next, using the formula from the chapter, we can calculate the discount factors for the time periods: (1/(1 + k + p)t) Then, the formula for net present value allows us to test each option: NPV = I0 + Σ Ft/(1 + r + p)t

We can construct a table that demonstrates projected inflows and outflows for the two projects.

61 Copyright 2019 © Pearson Education, Inc.


Project A Year

Inflows

0

Outflows

Net flow

Discount Factor

NPV

500,000

(500,000)

1.000

(500,000)

1

150,000

150,000

0.88

132,000

2

150,000

150,000

0.78

117,000

3

150,000

150,000

0.69

103,500

4

150,000

150,000

0.61

91,500

5

150,000

150,000

0.54

81,000

Total

$25,000

Project B Year

Inflows

0

Outflows

Net flow

Discount Factor

NPV

400,000

(400,000)

1.000

(400,000)

1

0

0

0.88

0

2

50,000

50,000

0.78

39,000

3

200,000

200,000

0.69

138,000

4

300,000

300,000

0.61

183,000

5

200,000

200,000

0.54

108,000

Total

$68,000

Conclusion: In this case, even though both projects offer a positive NPV, the highest NPV is Project B. Therefore, if we can only select one project to fund, Project B offers higher returns.

62 Copyright 2019 © Pearson Education, Inc.


3.27 Net Present Value. Your vice president of Management Information Systems informs you that she has researched the possibility of automating your organization’s order-entry system. She has projected that the new system will reduce labor costs by $35,000 each year over the next five years. The purchase price (including installation and testing) of the new system is $125,000. What is the net present value of this investment if the discount rate is 10% per year? Solution: Using the formulae shown in Problem 3.26 above, we can construct a discounted cash flow table: Discounted Cash Flows and NPV Year

Inflows

0

Outflows

Net flow

Discount Factor

NPV

125,000

(125,000)

1.000

(125,000)

1

35,000

35,000

0.91

31,818

2

35,000

35,000

0.83

29,050

3

35,000

35,000

0.75

26,250

4

35,000

35,000

0.68

23,800

5

35,000

35,000

0.62

21,700

Total

$7,493

Conclusion: Based on this analysis, the NPV for the project is positive, suggesting that the project would be a good investment.

63 Copyright 2019 © Pearson Education, Inc.


3.28 Net Present Value. A company has four project investment alternatives. The required rate of return on projects is 20%, and inflation is projected to remain at 3% into the foreseeable future. The pertinent information about each alternative is listed in the following chart. Which project should be the firm’s first priority? Why? If the company could invest in more than one project, indicate the order in which it should prioritize these project alternatives. Project Carol

Year 0 1 2 3

Investment $500,000

Revenue Streams 0 50,000 250,000 350,000

Project George

Year 0 1 2 3 4

Investment $250,000

Revenue Streams 0 75,000 75,000 75,000 50,000

Project Thomas Year 0 1 2 3 4 5 6

Investment $1,000,000

Revenue Streams 0 200,000 200,000 200,000 200,000 200,000 200,000

Project Anna Year 0 1 2 3 4 5

Investment $75,000

Revenue Streams 0 15,000 25,000 50,000 50,000 150,000

64 Copyright 2019 © Pearson Education, Inc.


Solution: Project Carol Year

Inflows

0

Outflows

Net flow

Discount Factor

NPV

500,000

(500,000)

1.000

(500,000)

1

50,000

50,000

0.81

40,500

2

250,000

250,000

0.66

165,000

3

350,000

350,000

0.54

189,000

Total

$(105,500)

Project George Year

Inflows

0

Outflows

Net flow

Discount Factor

NPV

250,000

(250,000)

1.000

(250,000)

1

75,000

75,000

0.81

60,750

2

75,000

75,000

0.66

49,500

3

75,000

75,000

0.54

40,500

4

50,000

50,000

0.44

22,000

Total

$(77,250)

65 Copyright 2019 © Pearson Education, Inc.


Project Thomas Year

Inflows

0

Outflows

Net flow

Discount Factor

NPV

1,000,000

(1,000,000)

1.000

(1,000,000)

1

200,000

200,000

0.81

162,000

2

200,000

200,000

0.66

132,000

3

200,000

200,000

0.54

108,000

4

200,000

200,000

0.44

88,000

5

200,000

200,000

0.36

72,000

6

200,000

200,000

0.29

58,000

Total

$(380,000)

Project Anna Year

Inflows

0

Outflows

Net flow

Discount Factor

NPV

75,000

(75,000)

1.000

(75,000)

1

15,000

15,000

0.81

12,150

2

25,000

25,000

0.66

16,500

3

50,000

50,000

0.54

27,000

4

50,000

50,000

0.44

22,000

5

150,000

150,000

0.36

54,000

Total

$56,650

Conclusions: The only project worth investing in is Project Anna, because it is the only project with a positive projected NPV. Each of the other three will not pay back the initial investment due to the high cost of capital (discount rate) the firm uses.

66 Copyright 2019 © Pearson Education, Inc.


3.29 Portfolio Management. Crown Corporation is interested in expanding its project portfolio. This firm, which specializes in water conservation and land reclamation projects, anticipates a huge increase in the demand for home fuel cells as an alternative to current methods of energy generation and usage. Although fuel-cell projects involve different technologies than those in which Crown currently specializes, the profit potential is very large. Develop a list of benefits and drawbacks associated with this potential expansion of Crown’s project portfolio. In your opinion, do the risks outweigh the advantages from such a move? Justify your answer. Solution: This question is intended to get students to think creatively about implications of new project ventures, particularly given the organization’s current project portfolio. Sometimes ideas seem to make sense but instructors should question how these alternative ideas will fit within the current portfolio? What will be the benefits and drawbacks from expanding a portfolio into new areas of technology or outside of the organization’s current range of expertise? Lastly, the TOWS matrix from the chapter can be a useful way of asking students to develop a simple classification matrix for current projects and potential new ones. There are no right or wrong answers to this question; the key is getting students to recognize both benefits and drawbacks from their suggested solution. You could also offer a simple portfolio consisting of some examples of bread and butter, pearls, oysters, and white elephant projects. How would they conceive of the new fuel cell opportunity fitting in to such a portfolio? What type of project category do they feel the fuel cell project would be? Why? These are questions that can encourage students to apply creativity to their answers.

3.30 Project Screening. Assume you are the IT manager for a large urban health care system. Lately you have been bombarded with requests for new projects, including system upgrades, support services, automated record keeping, billing, and so forth. With an average of 50 software and hardware support projects going on at any time, you have decided that you must create a system for screening new project requests from the various departments within the health care system. Develop a project selection and screening system like GE’s Tollgate process. What elements would you include in such a system? How many steps would you recommend? At what points in the process should “gates” be installed? How might a tollgate system for a software development company differ from one used by an architectural firm specializing in the development of commercial office buildings? Solution: This question is intended to get the students to consider how an organization can create decision gates to better screen project opportunities, develop prioritization schemes, and 67 Copyright 2019 © Pearson Education, Inc.


so forth. It is particularly useful to get students to identify a number of reasonable decision gates at which point projects should be reviewed. A key element in this discussion is for students to realize that just because a project was originally accepted, it does not necessarily follow that it should be pursued to completion, depending upon various environmental factors that may render the project no longer necessary. Again, in developing these decision gates, the instructor should try and get students to think of reviews for ongoing projects, as well on new opportunities. What sort of questions should be asked at each stage? What about sunk costs? Should that factor into the decision to maintain the project?

68 Copyright 2019 © Pearson Education, Inc.


CHAPTER TWO The Organizational Context Strategy, Structure, and Culture Chapter Outline PROJECT PROFILE The Airbus A 380: A Failure of Strategy? 2.1 IMPLEMENTING STRATEGY THROUGH PROJECTS 2.2 PROJECTS AND ORGANIZATIONAL STRATEGY 2.3 STAKEHOLDER MANAGEMENT Identifying Project Stakeholders Managing Stakeholders 2.4 ORGANIZATIONAL STRUCTURE FORMS OF ORGANIZATIONAL STRUCTURE Functional Organizations Project Organizations Matrix Organizations Moving to Heavyweight Project Organizations PROJECT MANAGEMENT RESEARCH IN BRIEF The Impact of Organizational Structure on Project Performance 2.5 PROJECT MANAGEMENT OFFICES 2.6 ORGANIZATIONAL CULTURE How Do Cultures Form? Organizational Culture and Project Management PROJECT PROFILE Electronic Arts and the Power of Strong Culture in Design Teams Summary Key Terms Discussion Questions Case Study 2.1 Rolls-Royce Corporation Case Study 2.2 Classic Case: Paradise Lost—The Xerox Alto Case Study 2.3 Project Task Estimation and the Culture of “Gotcha!” Case Study 2.4 Widgets ’R Us Internet Exercises PMP Certification Sample Questions Answers Integrated Project—Building Your Project Plan Notes

17 Copyright © 2019 Pearson Education, Inc.


TRANSPARENCIES Note for Instructors: To present transparencies in class, please download the PowerPoint Presentations (available on www.pearsonhighered.com) that accompanies this product. The PPT ISBN is 9780134730479.

2.1 PROJECTS AND CORPORATE STRATEGY

18 Copyright © 2019 Pearson Education, Inc.


2.2 PROJECT STAKEHOLDER RELATIONSHIPS

Parent Organization Other Functional Managers

External Environment

Project Manager

Clients

Accountant

Top Management

Project Team

19 Copyright © 2019 Pearson Education, Inc.


2.3 PROJECT STAKEHOLDER MANAGEMENT CYCLE

Identify Stakeholders Implement Stakeholder Management Strategy

Predict Stakeholder Behavior

Gather Information on Stakeholders

Project Management Team

Identify Stakeholder Strategy

Identify Stakeholders' Mission

Determine Stakeholder Strengths and Weaknesses

20 Copyright © 2019 Pearson Education, Inc.


2.4 EXAMPLE OF A FUNCTIONAL ORGANIZATIONAL STRUCTURE

Board of Directors

Chief Executive

Vice President of Marketing

Vice President of Production

Market Research Logistics

Vice President of Finance

Accounting Services

Sales

New Product Development Testing

Outsourcing After Market Support

Vice President of Research

Contracting Research Labs

Distribution

Investments

Quality

Advertising Warehousing

Employee Benefits

Manufacturing

21 Copyright © 2019 Pearson Education, Inc.


2.5

EXAMPLE OF A PROJECT ORGANIZATIONAL STRUCTURE

Board of Directors

Chief Executive

Vice President of Projects

Vice President of Marketing

Vice President of Production

Vice President of Finance

Vice President of Research

Project Alpha

Project Beta

22 Copyright © 2019 Pearson Education, Inc.


2.6 EXAMPLE OF A MATRIX ORGANIZATIONAL STRUCTURE

Board of Directors

Chief Executive

Vice President of Projects

Vice President of Marketing

Vice President of Production

Vice President of Finance

Vice President of Research

Project Alpha

2 resources

1.5 resources

1 resource

3 resources

Project Beta

1 resource

2 resources

2 resources

2.5 resources

23 Copyright © 2019 Pearson Education, Inc.


2.7 ALTERNATIVE LEVELS OF PROJECT OFFICES

Chief Operating Officer

Corporate Support

Business Unit

PO

Sales

Delivery

Level 3

Support PO

Level 2

Project A PO Project B PO

Level 1

Project C PO

24 Copyright © 2019 Pearson Education, Inc.


DISCUSSION QUESTIONS 2.1 The chapter suggests that a definition of strategic management includes four components: a. Developing a strategic vision and sense of mission b. Formulating, implementing, and evaluating c. Cross-functional decisions d. Achieving objectives Discuss how each of these four elements is important in understanding the challenge of strategic project management. How do projects serve to allow an organization to realize each of these four components of strategic management? Strategic management involves a complex system of establishing a vision, formulating strategies, and achieving objectives. Strategic management decisions are highly unique to each company—strategy for one company may be in exact opposition to strategies of another. Due to this, there is no predetermined “best way” to implement project management in every organization. Given the variety of corporate size and organization, the main challenge of strategic project management is figuring out how to best implement project management within the specific organizational structure of each company. While it may at first seem difficult to successful integrate project management into an organization, its presence in a corporation may enable effective execution of strategy and objectives. To begin with, projects may be designed around and driven by priorities and objectives derived from corporate mission and vision statements. Beyond the overreaching guidelines of a mission or vision, projects may be used to implement specific strategic initiatives quickly and effectively. Also, by breaking objectives down into projects, progress may be more easily monitored by management. Another aspect of strategic management is that it involves input and resources from various departments throughout the organization. Project teams enable the company to create cross-functional working groups that transcend organizational structure and allow for interdepartmental cooperation. All of the above-mentioned aspects of strategic project management permit organizations to break objectives and strategies into manageable pieces that can be focused on accomplishing specific objectives.

2.2 Discuss the difference between organizational objectives and strategies. Organizational objectives are broader than strategies in that they are derived from the company mission or vision and establish what the company desires to accomplish. On the other hand, strategies are more specific ideas that outline how the company plans to realize these objectives.

25 Copyright © 2019 Pearson Education, Inc.


2.3 Your company is planning to construct a nuclear power plant in Oregon. Why is stakeholder analysis important as a precondition of the decision of whether or not to follow through with such a plan? Conduct a stakeholder analysis for a planned upgrade to a successful software product. Who are the key stakeholders? In the case of building a nuclear plant, stakeholders can not only cause disruptions in the planning and construction, but can altogether block the project from being completed. Very powerful government, environmental, legal, and community stakeholders may intervene in the creation of the plant. Performing a stakeholder analysis could identify potential obstacles and stakeholder objections to building the plant. By identifying these obstacles in advance, it may be possible to prevent them. If prevention is not possible, assessing them beforehand may allow management time to create an alternate plan prior to resources being invested in the current project. Key stakeholders in a software upgrade would include suppliers, competitors, project team members, top and functional management and clients. Suppliers of the software would be influential in successful implementation and maintenance of the system. In the event of successful implementation, competitors would be affected by potential loss of market share. In the event of a failure, competitors would not only possibly gain new business, but may also learn from the shortcomings of the project and avoid such mistakes for themselves. Project team members would have direct impact on the success of the upgrade and as such would also stand to reap benefits or detriments from the outcome. Top management may be evaluated on the outcome of the project and may feel significant pressure to see that the project is a success. Ultimately, clients would stand to gain from a successful implementation in the areas of faster transactions or better service, and so on (depending on the type of software).

2.4 Consider a medium-sized company that has decided to begin using project management in a wide variety of its operations. As part of their operational shift, they are going to adopt a project management office somewhere within the organization. Make an argument for the type of PMO it should adopt (weather station, control tower, or resource pool). What are some of the key decision criteria that will help it determine which model makes most sense? The company should adopt a control tower PMO. Since widespread project management is new to the organizational structure, the control tower will offer it the necessary monitoring (sets standards) and maintenance (improvements and problem solving) for a successful transition into a project organization. It will provide support for employees and will help to focus on improvement and problem solving as the company works through the stages of implementing project management. When determining which model is best for the organization, it is important to consider the structure and size of the current organization, the role of projects within the company, resources available to the PMO and the chain of command.

26 Copyright © 2019 Pearson Education, Inc.


2.5 What are some of the key organizational elements that can affect the development and maintenance of a supportive organizational culture? As a consultant, what advice would you give to a functional organization that was seeking to move from an old, adversarial culture, where the various departments actively resisted helping one another, to one that encourages project thinking and cross-functional cooperation? The key elements that affect a supportive organizational culture are departmental interaction, employee commitment, project planning, and performance evaluation systems. Departmental interaction can create supportive relationships between functional and project managers. It promotes information sharing and increasing likelihood of project success. Employee commitment to goals is important in keeping workers motivated. When employees feel personally committed to company goals, they are willing to work harder (and possibly longer), which leads to success. When planning out resource constraints for a project, it is important to create trust and understanding among managers and employees. Managers are often responsible for approving use of resources from their department and also consult on time requirements for specific tasks. If managers are made an active part of the planning process, they are more willing to allocate resources and give accurate forecasts of time. Workers also need to feel as though they will not be punished if time frames are not met (as long as this is not a persistent problem), otherwise they (or their managers) may exaggerate the forecasted amount of time to complete a task. Finally, a performance evaluation criterion needs to encourage initiative and risk taking in a project environment. Additionally, rewards need to be consistent with the goals of the project. A functional organization that desires to move from an adversarial culture to a supportive, interactive one needs to consider several factors. First, the company should begin by establishing a corporate-wide vision that aims at uniting and motivating workers. Next, they have to create a reward/punishment system in line with that vision. Lastly, they will need to establish unambiguous policies on (short) lines of authority and communication. This will help provide fast and efficient decision-making.

2.6 Compare and contrast the organizational cultures at Amazon and Google. Imagine if you were in charge of a project team at both companies. How might your approach to managing a project, developing your team, and coordinating with different functional departments differ at the two firms? In order to answer this question meaningfully, students have to do some research on the differences in the cultures between the two companies, as reported in the popular press and websites. Generalizing, Amazon is noted for a much more fast-paced and individualistic—even cut-throat—environment. Google emphasizes the power of teams, creativity, independent thinking, and having fun. These characteristics can be used to illustrate how a project would be organized, staffed, how team members would be rewarded for good performance, and so forth. The key is for students to first make a defensible argument for how they would characterize these companies’ cultures and how 27 Copyright © 2019 Pearson Education, Inc.


they would distinguish between them. From these ideas, the means to manage a project would naturally follow. 2.7 You are a member of the senior management staff at XYZ Corporation. You have historically been using a functional structure set up with five departments: finance, human resources, marketing, production, and engineering. a. Create a drawing of your simplified functional structure, identifying the five departments. b. Assume you have decided to move to a project structure. What might be some of the environmental pressures that would contribute to your belief that it is necessary to alter the structure? c. With the project structure, you have four projects currently ongoing: stereo equipment, instrumentation and testing equipment, optical scanners, and defense communications. Draw the new structure that creates these four projects as part of the organizational chart.

Answer to a: Board of Directors CEO

VP of Finance

VP of HR

VP of Marketing

-- Accounting

-- Employment

-- Contracting

-- Training/ Development

-- Marketing Research

VP of Production

VP of Engineering

-- Logistics

-- Product Development

--Manufacturing -- Sales

-- Testing

Answer to b: Pressure may come from within the organization or from environmental or external sources. There may be pressure to be innovative or pressure from a rapidly changing market. Increased consumer demands or competition also put strain on a functional organization. These factors require quick response time, high innovation, speedy development, and risk-taking. Functional organizations may have difficulty meeting these needs, but project management can meet them by decreasing the chain of command and decision-making. Project management is then able to decrease time to make decisions, enable employee freedom to be innovative and take risks and get products/services to market quicker.

28 Copyright © 2019 Pearson Education, Inc.


2.8 Suppose you now wanted to convert the structure from that in Question 2.7 to a matrix, emphasizing dual commitments to function and project. a. Re-create the structural design to show how the matrix would look. b. What behavioral problems could you begin to anticipate through this design? That is, do you see any potential points of friction in the dual hierarchy setup? Answer to a: The conversion of the structure to a matrix is straightforward and involves the addition of a “project” organization along the left side of the structure design. Then, the student could indicate a couple of examples of projects (e.g., “A” and “B”) and how the project managers would link with the functional heads to secure their needed resources. The key is for students to recognize the joint responsibility for project staffing between the project manager and the functional manager. Answer to b: One of the best responses here is recognizing that the balancing of resources between functional department and project will require negotiation and bargaining between the project manager and the functional department head. This is especially the case in “balanced” or “weak” matrix structures, where the project manager may have minimal power to actually get their required resources and must use negotiation, influence, and perhaps the power of connections and using “bargaining chips” to help secure their resources. As the textbook notes, matrix is a constant source of friction between department heads, who want to keep their resources working on their own tasks, and project managers, who are seeking to gain access to these resources to support projects. The people often caught in the middle are the resources themselves: being pulled in multiple directions.

29 Copyright © 2019 Pearson Education, Inc.


CASE STUDIES

Case Study 2.1 Rolls-Royce Corporation Rolls-Royce is an example of a case based on new strategic opportunities and an organization’s desire to capitalize on market and technological developments. As one of the premier manufacturers of jet engines of the commercial and military markets, RollsRoyce is facing an opportunity to “piggy back” off Airbus’s newest airframe design, the A380, an enormous airplane capable of flying up to 750 people. The case also demonstrates the manner in which Rolls-Royce must identify and manage their key stakeholder group for maximum effectiveness. Questions 1. Who are Rolls-Royce’s principal project management stakeholders? How would you design stakeholder management strategies to address their concerns? Among the company’s biggest stakeholders are its direct customers, the commercial airframe manufacturers (Boeing and Airbus), as well as those supplying aircraft for military uses. Rolls-Royce also must work closely with national governments who subsidize their airlines by resorting to creative financing, long-term contracts, or assetbased trading deals. Among Rolls-Royce’s other key stakeholders are its labor force, which must be highly trained, its competitors (technical advances by a competitor must be immediately matched by Rolls-Royce), suppliers of parts and equipment, and so forth. Students discussing this case can create a large and very diverse stakeholder list. It is useful to illustrate how the desires of some stakeholders may be in direct opposition to the needs or expectations of others, making the point that stakeholder management is often a creative juggling act. 2. Given the financial risks inherent in developing a jet engine, make an argument, either pro or con, for Rolls-Royce to develop strategic partnerships with other jet engine manufacturers in a manner similar to Airbus’s consortium arrangement. What are the benefits and drawbacks in such an arrangement? In answering this question, it is helpful to first identify the tremendous barriers to entry and risk factors associated with manufacturing jet engines. What would Rolls-Royce gain from a consortium arrangement? What could they potentially lose? The arguments can add up on both sides of the ledger so the instructor can steer this discussion to include issues of stakeholder management, corporate strategy, and even culture, by highlighting the problems with blending conflicting cultures under a consortium arrangement.

30 Copyright © 2019 Pearson Education, Inc.


Case Study 2.2 Classic Case: Paradise Lost—The Xerox Alto The Xerox Alto is a fascinating story of a large organization fumbling the biggest technological advance in the latter half of the twentieth century. Xerox should have been poised to reap billions. It invested in an advanced research center (PARC), hired the best and brightest talent in this fledgling industry, and was first off the mark with a fully functioning PC, including Ethernet, laser printing, word processing, spreadsheets, and so forth. Instead, this case also details how they managed to squander their opportunity through a moribund culture, and attitude of “playing it safe,” and the inability to think creatively. In short, the Alto was simply too much for Xerox to know how to handle it. Questions 1. Do you see a logical contradiction in Xerox’s willingness to devote millions of dollars to support pure research sites like the PARC and its refusal to commercially introduce the products developed? Absolutely, this contradiction is one of the compelling points in the story. Discuss the difference between research for its own sake and the need to bring it to market. Also, did the Alto and the culture that created it violate Xerox’s strategic mission at the time, which seemed designed to play it safe and stick with simple, incremental products, rather than attempting to take quantum leaps forward. 2. How did Xerox’s strategic vision work in favor of or against the development of radical new technologies such as the Alto? Xerox had allowed their culture to become moribund and hence, their strategic focus was on making incremental improvements. The irony, as instructors may wish to bring up, is that the original Xerox innovation, the model 900 copier, was a radical innovation for its time and led to huge profits for the company. Thus, an organization that made its fortune and reputation on a highly successful and radical innovation could not bring themselves to do the same thing a decade later with the Alto opportunity. 3. What other unforeseeable events contributed to making Xerox’s executives unwilling to take any new risks precisely at the time the Alto was ready to be released? Over the five years after the development of the Alto, a series of ill-timed acquisitions, lawsuits, and reorganizations rendered the PC a casualty of inattention. What division would oversee its development and launch? Whose budget would support it and PARC in general? By leaving those tough decisions unmade, Xerox wasted valuable time and squandered their technological window of opportunity. 4. “Radical innovation cannot be too radical if we want it to be commercially successful.” Argue either in favor of or against this statement. 31 Copyright © 2019 Pearson Education, Inc.


This question can lead to an interesting discussion regarding the advantages and disadvantages of radical innovation. Arguments can be made for both radical change and “logical incrementalism” in new product development and introduction. One important factor to consider is the nature of the industry in which the organization is operating. For example, it could be argued that office products and information technology, which is the setting in which Xerox competed, requires a willingness to make the radical changes that would not be as necessary in other settings, facing less frequent or serious technical changes. Case Study 2.3 Project Task Estimation and the Culture of “Gotcha!” This short case is based on a true and common practice in which the culture of the organization encourages an “inauthentic” relationship to develop between project managers and those who serve on their teams. Authenticity is signaled by the relationship that develops between the leaders and the followers as they develop either a cooperative or combative working relationship. The project manager sets the tone; when they create an atmosphere of distrust, it is much safer for team members to protect themselves by fudging their work estimates. Questions 1. How does the organization’s culture support this sort of behavior? What pressures does the manager face? What pressures does the subordinate face? The organization’s culture has created and, paradoxically, rewarded an attitude of selfpreservation, competitiveness, and unwillingness to be truthful. In this situation, the project manager faces the pressure of getting the project done as quickly as possible. By subordinating everything to the need for speed, the project manager sends out the message that she only wants to hear good news. The subordinates’ pressures are different. If they are likely to be punished for missing their target estimates for the project, they will naturally over-inflate those initials estimates to give themselves sufficient time to complete the assignment. It now becomes a game between the subordinate and the project manager in which neither is willing to provide authentic information to the other. 2. Discuss the statement, “If you don’t take my estimates seriously, I’m not going to give you serious estimates!” How does this statement apply to this example? Subordinates are going to ensure that they protect themselves in the face of a project manager who distrusts them. As noted above, the key lies in authenticity. Where this is lacking, subordinates will assume an attitude of self-preservation. If they cannot trust their boss, they will take necessary steps to protect themselves. Thus, the statement, “If you don’t take my estimates seriously, I’m not going to give you serious estimates!”

32 Copyright © 2019 Pearson Education, Inc.


Case Study 2.4 Widgets ‘R Us This case highlights a company experiencing a number of challenges that are directly related to their willingness to shift to a project-based approach. As the case notes, product life cycles have dramatically shortened; however, at the same time, products are slow to market. Many new innovations have passed right by WRU because the company was slow to pick up signs from the marketplace that they were coming. Internal communication is very poor. These are all signals of an organization that is now facing a very different strategic challenge than one they had been pursuing previously. In the face of these problems, they need to consider how a new, project-based approach will help them. Key to understanding this case is recognizing that the old, functional organizational structure they had used will no longer support their operations within a new, highly complex marketplace. Questions 1. You have been called in as a consultant to analyze the operations at WRU. What would you advise? Students must recognize that many of the problems facing WRU are the result of its functional structure. In discussing the case, it is common for students at first to throw around a number of competing hypotheses as to why WRU is not competing well. Instructors should allow the discussion to continue to a point and then ask the question, “How does the firm’s structure add to the problems they are facing?” 2. What structural design changes might be undertaken to improve the operations at the company? Students may want to consider moving the organization to either a matrix or a project organization. Ask them to draw sample organizational designs reflecting either of these shifts and compare them to see what type of structure seems to make the most sense. 3. What are the strengths and weaknesses of the alternative solutions the company could employ? As the chapter discusses, there are a number of strengths and weaknesses of both the matrix and project organizations that students should consider. Will the overall result be positive in light of the new operating environment WRU finds itself facing? This is the key question that instructors should elicit from their students.

33 Copyright © 2019 Pearson Education, Inc.


CHAPTER ONE Introduction Why Project Management? Chapter Outline PROJECT PROFILE Case—Development Projects that are Transforming Africa 1.1 THE IMPORTANCE OF PROJECTS 1.2 WHAT IS A PROJECT? General Project Characteristics PROJECT PROFILE President Obama Signs the Program Management Improvement and Accountability Act 1.3 WHY ARE PROJECTS IMPORTANT? PROJECT PROFILE London’s Crossrail: Europe’s Largest Construction Project 1.4 PROJECT LIFE CYCLES PROJECT MANAGERS IN PRACTICE Theresa Hinkler, R. Conrader Company 1.5 DETERMINANTS OF PROJECT SUCCESS PROJECT MANAGEMENT RESEARCH IN BRIEF Assessing Information Technology (IT) Project Success 1.6 DEVELOPING PROJECT MANAGEMENT MATURITY 1.7 EMPLOYABILITY SKILLS 1.8 PROJECT ELEMENTS AND TEXT ORGANIZATION Summary Key Terms Discussion Questions Case Study 1.1 MegaTech, Inc. Case Study 1.2 The IT Department at Hamelin Hospital Case Study 1.3 Disney’s Expedition Everest Case Study 1.4 “Throwing Good Money after Bad”: the BBC’s Digital Media Initiative Internet Exercises PMP Certification Sample Questions Answers Notes

1 Copyright © 2019 Pearson Education, Inc.


TRANSPARENCIES Note for Instructors: To present transparencies in class, please download the PowerPoint Presentations (available on www.pearsonhighered.com) that accompanies this product. The PPT ISBN is 9780134730479.

1.1 GENERAL PROJECT CHARACTERISTICS 1. Projects are ad hoc endeavors with a clear life cycle. 2. Projects are building blocks in the design and execution of organizational strategies. 3. Projects are responsible for the newest and most improved products, services, and organizational processes. 4. Projects provide a philosophy and strategy for the management of change. 5. Project management entails crossing functional and organizational boundaries. 6. The traditional management functions of planning, organizing, motivation, directing, and control apply to project management. 7. The principal outcomes of a project are the satisfaction of customer requirements within the constraints of technical, cost, and schedule objectives. 8. Projects are terminated upon successful completion of performance objectives.

1.2 DIFFERENCES BETWEEN PROCESS AND PROJECT MANAGEMENT Process Repeat process or product Several objectives Ongoing People are homogenous Well established systems in place to integrate efforts Greater certainty of performance, cost, schedule Part of line organization Bastions of established practice Supports status quo

Project New process or product One objective One shot—limited life More heterogeneous Systems must be created to integrate efforts Greater uncertainty of performance, cost, schedule Outside of line organization Violates established practice Upsets status quo

2 Copyright © 2019 Pearson Education, Inc.


1.3 WHY ARE PROJECTS IMPORTANT? 1. Shortened product life cycles. 2. Narrow product launch windows. 3. Increasingly complex and technical products. 4. Global markets. 5. An economic period marked by low inflation.

1.4 PROJECT LIFE CYCLES

Project Life Cycle Stages

Manhours

Conceptualization

Planning

Execution

Termination

Pekka Rouhiainen March 26, 2000

3 Copyright © 2019 Pearson Education, Inc.


4 Copyright © 2019 Pearson Education, Inc.


1.5 PROJECT SUCCESS—THE NEW QUADRUPLE CONSTRAINT

Budget

Client Acceptance

Success

Schedule

Performance

5 Copyright © 2019 Pearson Education, Inc.


1.6 PROJECT MANAGEMENT MATURITY—A GENERIC MODEL

High Maturity Institutionalized, seeks continuous improvement Moderate Maturity Defined practices, training programs, organizational support Low Maturity Ad hoc process, no common language, little support

1.7 EMPLOYABILITY SKILLS Mastering project management will contribute to key skills employers are looking for: 1. Communication 2. Critical Thinking 3. Collaboration 4. Knowledge Application and Analysis 5. Business Ethics and Social Responsibility 6. Information Technology Application and Computing Skills 7. Data Literacy

6 Copyright © 2019 Pearson Education, Inc.


1.8 PMBoK KNOWLEDGE AREAS

7 Copyright © 2019 Pearson Education, Inc.


DISCUSSION QUESTIONS

1.1 What are some of the principal reasons why project management has become such a popular business tool in recent years? In today’s market, the length of product life cycles is shortening. This means businesses are under pressure to produce new or improved products at an increasingly rapid pace. Growing global markets, consumer tastes, and competition demand that products constantly be improved to be better, faster, and sleeker and offer more features. Most organizations are planning their next product or product improvement as their latest innovation is just on its way out the door. Under conventional business practices, keeping up with this demand for innovation can be difficult. Project management offers companies a manner in which to become more innovative and to develop products at a faster pace. 1.2 What do you see as the primary challenges to introducing a project management philosophy to most organizations? That is, why is it difficult to shift to a project-based approach in many companies? Many companies encounter a resistance to change within their personnel that makes implementing a new approach, such as project based, difficult. Employees have to be trained in the new processes and learn to implement it into their current role. Oftentimes, employees are averse to a large shift in current practices due to uncertainty of the outcome. 1.3 What are the advantages and disadvantages of using project management? Advantages:  Innovative, produce new ideas and new products  Geared toward accomplishing a specific goal  Aimed at customer satisfaction Disadvantages:  Inaccurate cost estimates during initial stages may cause project to fail due to lack of resources  Low success rate in some industries  Requires heavy commitment by staff 1.4 What key characteristics do all projects possess? Projects:  are temporary operations with a defined life span  help develop and execute organizational strategies and goals  are sources of innovation and progress  stimulate internal collaboration between members of various functional areas 8 Copyright © 2019 Pearson Education, Inc.


 

are limited by resource and time constraints end when objectives are successfully reached

1.5 Describe the basic elements of the project life cycle. Why is an understanding of the life cycle relevant for our understanding of projects? The project life cycle includes the stages of the project’s development. The basic elements of the cycle include:  conceptualization: outlines project goal, scope of work, identifies required resources and stakeholders  planning: specifications, timetables and other plans are created, work packages are broken out, assignments are made and process for completion is defined  execution: actual work of project takes place, majority of teamwork is performed and, characteristically, majority of costs are incurred  termination: project is completed and passed on to customer, resources are reassigned and team members disbanded Life cycles provide a guiding point for determining the scope and resource requirements of specific projects. By outlining a project’s life cycle, many challenges and potential pitfalls can be pinpointed. More generally, an understanding of life cycles lends itself to a better understanding of how projects function within an organization and how they differ from conventional forms of corporate process. 1.6 Think of a successful project and an unsuccessful project with which you are familiar. What distinguishes the two, both in terms of the process used to develop them and their outcomes? This question is intended for classes with students who have had some experience with projects in the past. It seeks to get them to examine the causes of success and failure from their own experience. Instructors should then begin developing a list of the various causes of success and failure as a point of discussion. 1.7 Consider the Expedition Everest case at the end of the chapter: what elements in Disney’s approach to developing its theme rides do you find particularly impressive? How can a firm like Disney balance the need for efficiency and smooth development of projects with the desire to be innovative and creative? Based on this case, what principles appear to guide its development process? This case lets students comment on the particularly appealing elements in Disney’s project management approach; for example, their attention to detail and willingness not to cut corners in terms of cost or schedule to make sure that the ride offers a memorable experience. The need to balance efficiency and creativity is an interesting one because it gets to the heart of project trade-offs. There are always more trips to be taken, more time to be spent, more artifacts that can be gathered to continuously “tweak” the ride; however, ultimately, they must also adhere to a roll-out schedule that gets the project completed. How much is enough? How much is too much? These form the basis of 9 Copyright © 2019 Pearson Education, Inc.


great in-class discussions. Finally, it is important to get the class to consider other factors that must weigh into project development decisions, like safety and general appeal. For example, creating a ride that is too intense for young children would violate Disney’s “kid friendly” philosophy. Likewise, all new rides must first be completely safe for the passengers, so any design issues always must be subordinated to safety.

1.8 Consider the six criteria for successful IT projects. Why is IT project success often so difficult to assess? Make a case for some factors being more important than others. IT project success is often difficult to assess because the criteria for success—system quality, information quality, use, user satisfaction, individual impact, and organizational impact—are not easy to accurately measure. Customer feedback related to user satisfaction, system quality, and impact may vary from user to user. For instance, while someone in insurance claims may find the system user friendly and beneficial to everyday tasks, an employee in actuary may find it cumbersome and difficult to navigate. When it comes to IT projects and the criterion above, user background, training, and experience could greatly affect the success rate of the project. These factors may not be fully known during initial planning and implementation stages. However, criteria such as system/information quality and use may be easier to assess. The team should be able to determine whether the designed system meets the specifications of the customer. All specifications should have been determined from the beginning, so upon completion test runs should determine if the system meets quality standards. In the area of use, following implementation, in most cases, it is possible to track use. Due to their ability to be more concretely measured, these factors, combined with the overall satisfaction of the customer, may be more important in determining success of the project than other more arbitrary measures. 1.9 As organizations seek to become better at managing projects, they often engage in benchmarking with other companies in similar industries. Discuss the concept of benchmarking. What are its goals? How does benchmarking work? Benchmarking compares the performance of a company to that of industry competitors and in some cases, for instance, where procedures or functions are similar, to that of superior performers in other industries. To set benchmarks for a company, first a leader in the industry is selected. Then, the company gathers data of that leader’s performance measures. The data is analyzed and gaps between the leader/benchmarks and the company are noted. The company then sets goals and strives to meet the benchmarking standards. The goal of benchmarking is, therefore, to seek out weak performance areas within the company and set goals for improvement.

10 Copyright © 2019 Pearson Education, Inc.


1.10 Explain the concept of a project management maturity model. What purpose does it serve? Implementing project management occurs in phases over time. Companies evolve through stages of project management. Project management maturity models are a way to help ensure that companies do so in the correct method and at a competitive pace. Maturity models provide a starting point for companies new to project management. Project maturity models offer businesses a way to map out necessary steps to becoming competitive through project-based work. Maturity models assess a specific company’s current practices (related to projects), establish the company’s position in relation to its competitors, and provide guidelines for improvement. They use industry data to establish a series of benchmarks. Based on industry competitors, they can then determine stages required as well as how quickly a company should develop. The company can then follow the model to achieve the highest level of ability in each pertinent project management area. 1.11 Compare and contrast the four project management maturity models shown in Table 1.3. What strengths and weaknesses do you perceive in each of the models? The four models each use five levels beginning with an initial ad hoc or sporadic use of project management and ending with a fully integrated project management system with emphasis on innovation and continuous improvement. Other similarities among the models include an element of benchmarking or use of industry standards to measure project management performance. The models do vary on the relative pace of innovation. For instance, the ESI’s International Project Framework develops more slowly in early stages than that of Kerzner’s Project Management Maturity Model. In addition, some models focus more on learning while others are more directed at control. Kerzner’s discusses training and curriculum while SEI’s Capability Maturity Model Integration outlines steps for control and assessment of results.

Center for Business Practices Strengths: It is mapped out at an appropriate pace; there are no broad leaps from one stage to the next. Also, it emphasized the role of project management as corporate processes, which means project management becomes part of the working firm, not just part of the job duties of a specific group or team. Weaknesses: This model lacks direction in management training. It refers to management awareness and support, but does not mention training or formal training.

11 Copyright © 2019 Pearson Education, Inc.


Kerzner’s Project Management Maturity Model Strengths: Kerzner’s does a much better job of designating at what levels managers need to be trained or curriculum developed. Weaknesses: Benchmarking does not come in until level 4, which maybe a little late. A firm in this model would have already integrated project management processes; trying to make any significant adjustments (in accordance with benchmarking figures) after this integration may be difficult. ESI International’s Project Framework Strengths: This model has two strong qualities. The first is its overt emphasis on innovation and continuous improvement. Secondly, the model emphasizes the need for integration and understanding throughout the firm. Weaknesses: The movement between levels 1 through 3 may cause problems for a firm. In level 1, processes are ill-defined and have little organizational support; this changes little as the corporation moves to level 2, which has no project control processes. Then, in level 3 processes are tailored. Given the undefined nature of processes prior to level 3, it may be hard to reach this goal initially. This may cause companies to become stalled in level 3. SEI’s Capability Maturity Model Integration Strengths: Quality is a top concern even in early stages of this model. Analysis and insurance procedures are developed at different stages to ensure standards are met. Weaknesses: On the other hand, the attention to testing may also hinder project management integration. There may be an overabundance of measures to control, analyze, and qualify in this system. While quality is of importance, the level of time commitment to those procedures may be the crux of this model. Team members may become frustrated with the project process if they (or their work) are constantly being measured, tested, and re-measured.

12 Copyright © 2019 Pearson Education, Inc.


CASE STUDIES

Case Study 1.1 MegaTech, Inc. MegaTech, Inc. is designed to highlight some of the reasons why an organization that had operated in a relatively stable and predictable environment would seek to move to an emphasis on project-based work. The trigger event, in this case is the advent of the NAFTA treaty, which opened up competition on a more price-competitive basis. Questions 1. What is it about project management that offers MegaTech a competitive advantage in its industry? Project management techniques will allow the firm to combine the advantages of internal efficiency with external (environmental) responsiveness. For example, it was determined that successful firms offer frequent product updates, which MegaTech’s move has allowed them to exploit. It has also promoted a team-based atmosphere that is encouraging cooperation and unity of effort among the different functional departments. 2. What elements of the marketplace in which MegaTech operates led the firm to believe that project management would improve its operations? The intense, new competitive nature of the marketplace impels companies to find new methods for competitive advantage. With many new competitors and serious price pressure, success will require firms to be fast to market, hold the lid on costs, offer frequent upgrades and new products—all while encouraging an atmosphere of risk taking and cooperation.

Case Study 1.2 The IT Department at Hamelin Hospital The IT Department case shows the prevalence of projects in settings that are perhaps not as obvious (in this case, a large hospital). The case is designed to get students to understand the ubiquitous nature of project-based work in our modern public and private organizations. It also demonstrates career paths and how successful work on projects is often rewarded with corporate success. Projects are not a distraction or a side-line; they are the principal means by which the IT department’s operations are demonstrated. Questions 1. What are the benefits and drawbacks of starting most new hires at the help desk function?

13 Copyright © 2019 Pearson Education, Inc.


Most new hires start at the help desk function, where they can become familiar with the system, learn about problem areas, become sensitive to user’s frustrations and concerns, and understand how the IT department affects all hospital operations. Students may also note that though mundane, working at the help desk forces people to “pay their dues” by demonstrating their commitment to the organization prior to being trusted with enhanced responsibilities. 2. What are the potential problems with requiring project team members to be involved in multiple projects at the same time? What are the potential advantages? One serious disadvantage is that it stretches departmental resources very thin; people can be involved in multiple projects and are likely to start letting commitments slip if they are not careful. Also, it is often difficult to move from assignment to assignment quickly and seamlessly. Instead, team members experience times when they are not productive as they try and multitask across several projects at once. Among the advantages are that this configuration allows the project team members to work with many different people, including several project managers, experiencing different managerial styles and interpersonal relationships. It also keeps team members interest high because their involvement in multiple teams and projects ensures that they do not become bored by routine. 3. What signals does the department send by making “project manager” the highest position in the department? The main signal is the idea that the career path for successful IT professionals runs directly through project-based work. They cannot be successful in this organization unless they are competent at first serving in and then running projects. Case Study 1.3 Disney’s Expedition Everest The Expedition Everest case is an example of the extreme attention to detail that Disney pays in all of their rides. Their management is a combination of careful planning coupled with the imagination and knack for visual effects for which the company is well known. The case tells the story of the development of the ride, the numerous steps Disney went through to get every detail as accurate as possible, and reflects on their overall approach to project management. Questions 1. Suppose you were a project manager for Disney. Based on the information in this case, what do you think the company uses when designing a new ride? That is, how would you prioritize the needs for addressing project cost, schedule, quality, and client acceptance? What evidence supports your answer?

14 Copyright © 2019 Pearson Education, Inc.


The case clearly shows that Disney makes its priorities quality and client acceptance. Given the industry they are in, they must first ensure that all their rides are safe and of the highest quality (customers expect nothing less from the Disney name). Because these issues are paramount, concerns with cost and schedule are secondary considerations. One way to see that this is the case is to consider the multiple trips that Disney Imagineers took to Nepal to gather local artifacts, check the topography, and building styles, and other steps to ensure accuracy. These come at a cost but to Disney, the overall effect is worth it. 2. Why is Disney’s attention to detail in its rides unique? How do they use the “atmosphere” discussed in the case to maximize the experience while minimizing complaints about length of wait for the ride? Disney is interested in creating more than simply a ride; they seek to provide their customers with an experience. In order to maximize this effect, the attention to detail, including the ancillary buildings they construct and the way in which the grounds are prepared, are all designed to distract the customer from the sometime lengthy wait for the ride. The better able Disney is to develop this sense of overall atmosphere, the more their customers will find the ride memorable and their overall satisfaction with Disney will be that much higher. Case Study 1.4 “Throwing Good Money after Bad”: the BBC’s Digital Media Initiative The BBC’s disastrous efforts to digitize their broadcast operations resulted in a loss of millions; a system that was riddled with technical flaws, and resulted in the loss of original material and archived records. This case allows students to explore many of the causes of this disaster, occurring at a well-known organization—the British Broadcasting Corporation. This case can be used to illustrate the wide number of ways in which a project can go wrong—through poor technical support, over-optimistic estimates and the ways in which different stakeholders viewed the project, constantly assuming that the problems were due to other reasons, and not through their missteps. The case is quite comprehensive and addresses both causes of the project disaster as well as outcomes from the aftermath of the failure. Questions 1. What does the story of the BBC’s failed Digital Media Initiative suggest to you about the importance of carefully managing not only the project, but the “message” of the project? That is, why is “benefits management” critical for project success? This question is intended to demonstrate to students the way that project management requires us to carefully plan the project front-end, develop a coherent goal or set of goals for the project, and make sure that these goals are widely understood. One of the problems with this project was a lack of understanding of exactly what digitizing the 15 Copyright © 2019 Pearson Education, Inc.


BBC’s system would do. What were the benefits from this project? Without a clear understanding of the goals, it is impossible to craft a coherent message about its benefits, leading to misunderstandings, faulty interpretations, and finger-pointing when problems arise. 2. Successful project management requires clear organization, careful planning, and good execution. How was the absence of each of these traits shown in this example? The BBC Digital Media case offers great examples of the need to carefully organize a variety of different groups and teams, including contractors and other stakeholders, to ensure that everyone is working toward the same goals. The missteps and technological failures of the project illustrate the need to manage not only the core technologies of the project but all relationships among members of the larger project team. Once the problems started occurring, the project organization also failed to effectively manage the message to the larger public, insisting the technology was fine in the face of multiple examples of failure. All in all, this is a great case example illustrating why project management is such a challenge and the myriad issues and groups that effective leaders must manage for project success.

16 Copyright © 2019 Pearson Education, Inc.


Turn static files into dynamic content formats.

Create a flipbook
Issuu converts static files into: digital portfolios, online yearbooks, online catalogs, digital photo albums and more. Sign up and create your flipbook.